P1 2011 Test 5 Answ

Télécharger au format pdf ou txt
Télécharger au format pdf ou txt
Vous êtes sur la page 1sur 73

Acquérir une certification

> Préparation au CIA, partie 1

Questionnaire
Eléments de restitution

Restitution

© IFACI – CIA1 V.5.1 page 1/73


Acquérir une certification
> Préparation au CIA, partie 1

P1 - 1 -- Quel type d'activités peuvent être effectuées dans le cadre de l'audit P1 - 1 -- Internal audit activities may involve which of the following?
interne ?
A. Des activités d'assurance. A. Assurance services.
B. Des activités de conseil. B. Consulting services.
C. Des activités d'assurance et de conseil. C. Both assurance and consulting services.
D. Ni des activités d'assurance, ni des activités de conseil. D. Neither assurance nor consulting services.
La réponse (A) est fausse. L'audit interne peut effectuer aussi bien des Answer (A) is incorrect because internal audit activities may involve
activités d'assurance que de conseil. both assurance and consulting activities.
La réponse (B) est fausse. L'audit interne peut effectuer aussi bien des Answer (B) is incorrect because internal audit activities may involve
activités d'assurance que de conseil. both assurance and consulting activities.
La réponse (C) est juste. L'audit interne peut effectuer aussi bien des Answer (C) is correct. Internal audit activities may involve
activités d'assurance que de conseil. Les activités d'assurance sont un both assurance and consulting services. Assurance services
examen objectif d'éléments probants, effectué en vue de fournir à are objective examinations of evidence for the purpose of
l'organisation une évaluation indépendante des processus de providing an independent assessment on risk management,
management des risques, de contrôle ou de gouvernement control, or governance processes for the organization.
d'entreprise. Les activités de conseil sont les conseils et services y Consulting services are advisory and related client service
afférents rendus au client donneur d'ordre, dont la nature et le champ activities, the nature and scope of which are agreed upon with
sont convenus au préalable avec lui. Ces activités ont pour objectifs de the client and are intended to add value and improve an
créer de la valeur ajoutée et d'améliorer le fonctionnement de organization's operations.
l'organisation. Answer (D) is incorrect because internal audit activities may involve
La réponse (D) est fausse. L'audit interne peut effectuer aussi bien des both assurance and consulting activities.
activités d'assurance que de conseil.

P1 - 2 -- Laquelle des propositions suivantes décrit le mieux l'objectif de l'audit P1 - 2 -- Which of the following best describes the purpose of the
interne ? internal audit activity?
A. Créer de la valeur ajoutée et améliorer les opérations d'une organisation. A. To add value and improve an organization's operations.
B. Aider la direction dans la conception et la mise en oeuvre des systèmes de B. To assist management with the design and implementation of
gestion des risques et de contrôle. risk management and control systems.
C. Examiner et évaluer le système comptable d'une organisation, à titre de C. To examine and evaluate an organization's accounting system
service rendu à la direction. as a service to management.
D. Superviser le système de contrôle interne d'une organisation pour le D. To monitor the organization's internal control system for the
compte d'auditeurs externes. external auditors.
La réponse (A) est juste. D'après sa définition, l'audit interne est “une Answer (A) is correct. According to the definition of internal
activité indépendante et objective qui donne à une organisation une auditing, "Internal auditing is an independent, objective
assurance sur le degré de maîtrise de ses opérations, lui apporte des assurance and consulting activity designed to add value and
conseils pour les améliorer et contribue à créer de la valeur. L'audit improve an organization's operations. It helps an organization
interne aide cette organisation à atteindre ses objectifs en évaluant, par accomplish its objectives by bringing a systematic,
une approche systématique et méthodique, ses processus de disciplined approach to evaluate and improve the
management des risques, de contrôle et de gouvernement d'entreprise, effectiveness of risk management, control, and governance
et en faisant des propositions pour renforcer leur efficacité.” processes."
La réponse (B) est fausse. Ces fonctions de conception et de mise en oeuvre Answer (B) is incorrect because performing the functions of design
des systèmes de gestion des risques et de contrôle entraveraient l'objectivité and implementation of risk management and control systems
des auditeurs internes. Un auditeur interne peut recommander des normes would impair the objectivity of the internal auditors. An internal
de contrôle et examiner les procédures avant leur mise en oeuvre. auditor may recommend control standards and review procedures
La réponse (C) est fausse. L'audit interne est bien plus vaste que les simples prior to their implementation.
missions d'examen et d'évaluation d'un système comptable. Answer (C) is incorrect because internal auditing is much broader
La réponse (D) est fausse. L'audit interne sert l'organisation, et non les than examining and evaluating an organization's accounting
auditeurs externes. system.
Answer (D) is incorrect because internal auditing serves the
organization, not the external auditors.

Restitution

© IFACI – CIA1 V.5.1 page 2/73


Acquérir une certification
> Préparation au CIA, partie 1
P1 - 3 -- Les auditeurs internes doivent définir dans quelle mesure le P1 - 3 -- Internal auditors need to ascertain the extent to which
management a fixé des critères adéquats pour déterminer si les buts et management has established adequate criteria to determine
objectifs ont été atteints. À cette fin, laquelle (lesquelles) des actions whether goals and objectives have been accomplished. For this
suivantes peut (peuvent) être appropriée(s) ? purpose, which of the following actions may be appropriate?
I. Déterminer si les buts et objectifs des opérations et projets sont I. Determining whether operating and program goals and
conformes à ceux de l'organisation. objectives conform to those of the organization
II. Revoir les opérations pour définir dans quelle mesure les résultats II. Reviewing operations to ascertain the extent to which results
correspondent aux buts et objectifs fixés. are consistent with established goals and objectives
III. Œuvrer avec l'encadrement à l'élaboration de critères d'évaluation du III. Working with management to develop appropriate control
contrôle appropriés. evaluation criteria
A. I uniquement. A. I only.
B. I et II uniquement. B. I and II only.
C. I, II et III. C. I, II, and III.
D. II uniquement. D. II only.
La réponse (A) est fausse. Les auditeurs internes peuvent entreprendre Answer (A) is incorrect because the internal auditors may take any
n'importe laquelle des tâches décrites en I, II et III. of the actions described in statements I, II, and III.
La réponse (B) est fausse. Les auditeurs internes peuvent entreprendre Answer (B) is incorrect because the internal auditors may take any
n'importe laquelle des tâches décrites en I, II et III. of the actions described in statements I, II, and III.
La réponse (C) est juste. Conformément aux Normes, « Les auditeurs Answer (C) is correct. According to the Standards, "Internal
internes devraient déterminer dans quelle mesure des buts et objectifs auditors should ascertain the extent to which operating and
concernant les opérations et programmes ont été définis et si ces buts program goals and objectives have been established and
et objectifs sont conformes à ceux de l’organisation ». Les normes conform to those of the organization." Besides, Standards
indiquent par ailleurs que “Les auditeurs internes devraient passer en state, "Internal auditors should review operations and
revue les opérations et les programmes afin de déterminer dans quelle programs to ascertain the extent to which results are
mesure les résultats suivent les buts et objectifs établis et si ces consistent with established goals and objectives to determine
opérations et programmes sont mis en oeuvre ou réalisés comme whether operations and programs are being implemented or
prévu.” De plus, “des critères adéquats sont nécessaires pour évaluer performed as intended." Furthermore, "Adequate criteria are
le dispositif de contrôle. Les auditeurs internes doivent déterminer needed to evaluate controls. Internal auditors should
dans quelle mesure le management a défini des critères adéquats pour ascertain the extent to which management has established
apprécier si les objectifs et les buts ont été atteints. Si ces critères sont adequate criteria to determine whether objectives and goals
adéquats, les auditeurs internes doivent les utiliser dans leur have been accomplished. If adequate, internal auditors should
évaluation. S'ils sont inadéquats, les auditeurs internes doivent use such criteria in their evaluation. If inadequate, internal
travailler avec le management pour élaborer des critères d'évaluation auditors should work with management to develop
appropriés”. appropriate evaluation criteria".
La réponse (D) est fausse. Les auditeurs internes peut entreprendre Answer (D) is incorrect because the internal auditors may take any
n'importe laquelle des tâches décrites en I, II et III. of the actions described in statements I, II, and III.

P1 - 4 -- Lequel des postes suivants est responsable du développement et du P1 - 4 -- Which of the following is responsible for developing and
maintien d’un programme d’assurance et d’amélioration de la qualité qui maintaining a quality assurance and improvement program that
couvre tous les aspects de l’audit interne et qui permet un suivi permanent de covers all aspects of the internal audit activity and continuously
son efficacité ? monitors its effectiveness?
A. La direction générale. A. Senior management.
B. Le responsable de l’audit interne. B. Chief audit executive.
C. Le conseil d’administration. C. The Board of Directors.
D. Le comité d’audit. D. Audit committee.
La réponse (A) est fausse. Ils ne sont pas responsables du développement et Answer (A) is incorrect because they are not responsible for
du maintien d’un programme d’assurance et d’amélioration de la qualité qui developing and maintaining a quality assurance and improvement
couvre tous les aspects de l’activité de l’audit interne et qui permet un suivi program that covers all aspects of the internal audit activity and
permanent de son efficacité. continuously monitors its effectiveness.
La réponse (B) est juste. Le responsable de l’audit interne doit élaborer et Answer (B) is correct. The chief audit executive should develop
tenir à jour un programme d’assurance et d’amélioration qualité portant and maintain a quality assurance and improvement program
sur tous les aspects de l’audit interne et permettant un contrôle continu that covers all aspects of the internal audit activity and
de son efficacité. L’objectif d’un tel programme est d’aider l’audit interne à continuously monitors its effectiveness. The program should
apporter une valeur ajoutée à améliorer le fonctionnement de be designed to help the internal auditing activity add value and
l’organisation et de garantir que l’audit interne est en conformité avec les improve the organization's operation and to provide assurance
Normes et Le Code de Déontologie. that the internal audit activity is in conformity with the
La réponse (C) est fausse. Ils ne sont pas responsables du développement Standards and the Code of Ethics.
et du maintien d’un programme d’assurance et d’amélioration de la qualité Answer (C) is incorrect because they are not responsible for
qui couvre tous les aspects de l’activité de l’audit interne et qui permet un developing and maintaining a quality assurance and improvement
suivi permanent de son efficacité. program that covers all aspects of the internal audit activity and
La réponse (D) est fausse. Ils ne sont pas responsables du développement continuously monitors its effectiveness.
et du maintien d’un programme d’assurance et d’amélioration de la qualité Answer (D) is incorrect because they are not responsible for
qui couvre tous les aspects de l’activité de l’audit interne et qui permet un developing and maintaining a quality assurance and improvement
suivi permanent de son efficacité. program that covers all aspects of the internal audit activity and
continuously monitors its effectiveness.

Restitution

© IFACI – CIA1 V.5.1 page 3/73


Acquérir une certification
> Préparation au CIA, partie 1
P1 - 5 -- Afin d'éviter d'être la source apparente d'un conflit entre la direction P1 - 5 -- To avoid being the apparent cause of conflict between an
générale d'une organisation et le comité d'audit, le responsable de l'audit organization's senior management and the audit committee, the
interne doit : chief audit executive should
A. Transmettre l'ensemble des résultats d'une mission à la fois à la direction A. Communicate all engagement results to both senior
générale et au comité d'audit. management and the audit committee.
B. Renforcer l'indépendance de l'audit interne à travers sa position dans B. Strengthen the independence of the internal audit activity
l'organisation. through organizational status.
C. Commencer par discuter avec le comité d'audit de l'ensemble des C. Discuss all reports to senior management with the audit
conclusions qu'il entend présenter à la direction générale. committee first.
D. Demander que le conseil d'administration valide les mesures qui traitent D. Request board approval of policies that include internal audit
des relations entre l'audit interne et le comité d'audit. activity relationships with the audit committee.
La réponse (A) est fausse. Il n'est pas nécessaire, et il serait inefficient, que la Answer (A) is incorrect because receipt of all engagement results
direction générale et le comité d'audit reçoivent l'ensemble des résultats by senior management and the audit committee is unnecessary
d'une mission. and inefficient.
La réponse (B) est fausse. La position dans l'organisation aide le service d'audit Answer (B) is incorrect because organizational status helps the
interne à être indépendant, mais ne suffit pas, à elle seule, à éviter les conflits. internal audit activity to achieve independence but is not, by itself,
La réponse (C) est fausse. Le comité d'audit exerce avant tout un rôle de enough to avoid conflict.
supervision et non une fonction opérationnelle. Answer (C) is incorrect because the audit committee essentially
La réponse (D) est juste. La mission, les pouvoirs et les responsabilités has an oversight rather than an operational role.
de l'audit interne doivent être formellement définis dans une Charte, Answer (D) is correct. The purpose, authority, and responsibility
être cohérents avec les Normes et dûment approuvés par le conseil. La of the internal audit activity should be formally defined in a
Charte doit établir la position du service d'audit interne dans charter, consistent with the Standards, and approved by the
l'organisation, autoriser l'accès aux documents, aux personnes et aux board. The charter should establish the internal audit activity's
biens physiques, nécessaire à la bonne réalisation des missions et position within the organization; authorize access to records,
définir le champ de l'audit interne. personnel, and physical properties relevant to the performance
of engagements; and define the scope of internal audit
activities.

P1 - 6 -- Selon le Cadre de Référence International des Pratiques P1 - 6 -- According to the International Professional Practices
Professionnelles, l'indépendance de l'audit interne passe par : Framework, the independence of the internal audit activity is
achieved through:

A. La dotation en personnel et la supervision. A. Staffing and supervision.


B. La formation professionnelle continue et la conscience professionnelle. B. Continuing professional development and due professional care.
C. Les relations humaines et les communications. C. Human relations and communications.
D. La position dans l'organisation et l'objectivité. D. Organizational status and objectivity.

La réponse (A) est fausse. La dotation en personnel et la supervision sont Answer (A) is incorrect. Staffing and supervision relate to the
liées à la compétence professionnelle de l'audit interne. professional proficiency of the internal audit activity.
La réponse (B) est fausse. la formation professionnelle continue et la Answer (B) is incorrect. Continuing professional development and
conscience professionnelle sont liées à la compétence professionnelle de due professional care relate to the professional proficiency of the
l'audit interne. internal auditor.
La réponse (C) est fausse. Les relations humaines et les communications Answer (C) is incorrect. Human relations and communications
sont liées à la compétence professionnelle de l'audit interne. relate to the professional proficiency of the internal auditor.
La réponse (D) est juste. Selon le CRIPP, la position dans l'organisation Answer (D) is correct. According to IPPF, organizational
et l'objectivité permettent aux auditeurs internes de porter des status and objectivity permit members of the internal audit
jugements sans partialité et sans préjugé, ce qui est essentiel à la activity to render the impartial and unbiased judgments
bonne conduite des audits. essential to the proper conduct of engagements.

Restitution

© IFACI – CIA1 V.5.1 page 4/73


Acquérir une certification
> Préparation au CIA, partie 1
P1 - 7 -- Laquelle des actions suivantes nuit à l'indépendance de l'auditeur ? P1 - 7 -- Which of the following actions would be a violation of
auditor independence?
A. Poursuivre une mission dans un service dont l'auditeur sera bientôt A. Continuing on an audit assignment at a division for which the
responsable en raison d'une promotion. auditor will soon be responsible as the result of a promotion.
B. Réduire le champ d'une mission en raison de restrictions budgétaires. B. Reducing the scope of an engagement due to budget
C. Participer à un groupe de travail qui recommande des normes de contrôle restrictions.
pour un nouveau système de distribution. C. Participating on a task force which recommends standards of
D. Examiner la version préliminaire des contrats d'un préposé aux achats control for a new distribution system.
avant leur signature. D. Reviewing a purchasing agent's contract drafts prior to their
execution.
La réponse (A) est juste. Un auditeur qui rejoint un service à la suite Answer (A) is correct. An auditor who has been promoted to
d'une promotion ne doit pas poursuivre l'audit de ce service. Selon le an operating department should not continue on an audit of
CRIPP, le responsable de l'audit interne doit réaffecter les auditeurs that department. According to IPPF, the chief audit executive
dans les situations où l'on peut raisonnablement suspecter un conflit should reassign auditors if a conflict of interest or bias may
d'intérêt ou un parti-pris. be reasonably inferred.
La réponse (B) est fausse. les restrictions budgétaires ne nuisent pas à Answer (B) is incorrect. Budget restrictions do not constitute a
l'indépendance d'un auditeur. violation of an auditor's independence.
La réponse (C) est fausse. Selon le CRIPP, un auditeur peut recommander Answer (C) is incorrect. The IPPF states that an auditor may
la mise en place de normes de contrôle pour de nouveaux systèmes recommend standards of control for new systems. However,
d'information. Par contre, son objectivité est présumée altérée s'il conçoit, met designing, installing, or operating such systems might impair
en place, rédige les procédures y afférentes ou exploite de tels systèmes. objectivity.
La réponse (D) est fausse. Un auditeur peut examiner les contrats avant leur Answer (D) is incorrect. An auditor may review contracts prior to
signature. their execution.

P1 - 8 -- Dans laquelle des situations suivantes l'auditeur risque-t-il de P1 - 8 -- In which of the following situations would an auditor
manquer d'objectivité ? potentially lack objectivity?
A. Un auditeur examine les procédures relatives à une nouvelle connexion A. An auditor reviews the procedures for a new electronic data
pour l'échange de données électronique avec un client important avant sa interchange connection to a major customer before it is
mise en service. implemented.
B. Quatre mois après avoir été transféré dans le service de l'audit interne, un B. A former purchasing assistant performs a review of internal
ancien responsable des achats analyse les contrôles internes appliqués aux controls over purchasing four months after being transferred to the
achats. internal audit activity.
C. Un auditeur recommande la mise en place de normes de contrôle et de C. An auditor recommends standards of control and performance
mesures de la performance pour un contrat avec une société de services measures for a contract with a service organization for the
concernant le traitement des fiches de paie et des avantages sociaux. processing of payroll and employee benefits.
D. Un membre du service de la paie aide un auditeur à vérifier le stock D. A payroll accounting employee assists an auditor in verifying the
physique de petits moteurs. physical inventory of small motors.
La réponse (A) est fausse. Selon le CRIPP, “l'objectivité de l'auditeur interne Answer (A) is incorrect. The IPPF states that the internal auditor's
n'est pas compromise lorsqu'il est amené à examiner des procédures avant objectivity is not adversely affected when the auditor reviews
leur mise en application.” procedures before they are implemented.
La réponse (B) est juste. Selon le CRIPP, “les membres du personnel Answer (B) is correct. The IPPF states that persons
qui sont affectés au service d'audit interne ne doivent pas participer transferred to the internal audit activity should not be
avant un délai raisonnable (au moins un an) à l'audit de leurs assigned to audit those activities that they previously
précédentes activités.” performed until a reasonable period of time (at least one year)
La réponse (C) est fausse. Selon le CRIPP, “l'objectivité de l'auditeur interne has elapsed.
n'est pas compromise lorsqu'il est amené à recommander la mise en place Answer (C) is incorrect. The IPPF states that the internal auditor's
de normes de contrôle pour les systèmes d'information avant leur mise en objectivity is not adversely affected when the auditor recommends
application.” standards of control for systems before they are implemented.
La réponse (D) est fausse. Faire appel à des membres d'autres services pour Answer (D) is incorrect. Use of staff from other areas to assist the
aider l'auditeur interne ne compromet pas l'objectivité de ce dernier, en internal auditor does not impair objectivity, especially when the staff
particulier si ces personnes n'émanent pas du service audité. is from outside the area being audited.

Restitution

© IFACI – CIA1 V.5.1 page 5/73


Acquérir une certification
> Préparation au CIA, partie 1
P1 - 9 -- Le responsable de l'audit interne a demandé à un auditeur interne P1 - 9 -- The chief audit executive has assigned an internal auditor
d'effectuer une mission de fin d'exercice afin d'évaluer les données de la to perform a year-end engagement to evaluate payroll records. The
paye. L'auditeur interne a contacté le directeur du service de la paye et s'est internal auditor has contacted the director of compensation and has
vu refuser l'accès aux documents nécessaires. Pour éviter un tel problème, been refused access to necessary documents. To avoid this
problem,
A. L'accès aux documents nécessaire à la bonne réalisation des missions
doit être inscrit dans la charte de l'audit interne. A. Access to records relevant to performance of engagements
B. L'audit interne doit être obligé de rendre des comptes au directeur de should be specified in the internal audit activity's charter.
l'organisation. B. Internal auditing should be required to report to the CEO of the
C. Si l'on suit le processus de planification à long terme, l'accès à tous les organization.
documents pertinents doit être garanti. C. By following the long-range planning process, access to all
D. Toutes les limitations du champ d'intervention doivent recevoir l'aval du relevant records should be guaranteed.
comité d'audit. D. Audit committee approval should be required for all scope
limitations.
La réponse (A) est juste. “Les auditeurs internes doivent bénéficier du Answer (A) is correct. The IAA should have the support of
soutien de la direction générale et du Conseil afin d'obtenir la management and the board in gaining cooperation from all
coopération des audités et de pouvoir exercer leur activité sans engagement clients (IPPF). Specific guidelines should be
entrave” (CRIPP). La Charte doit contenir des principes directeurs written in its charter authorizing access to records, personnel,
spécifiques autorisant “l'accès aux documents, aux personnes et aux and physical properties relevant to the performance of
biens physiques, nécessaire à la bonne réalisation des missions” engagements (IPPF). Such provisions reduce the likelihood of
(CRIPP). Ces dispositions réduisent la possibilité de rencontrer des scope limitations.
limitations du champ d'intervention. Answer (B) is incorrect because the IAA need not report to a
La réponse (B) est fausse. L'audit interne n'a pas à rendre de comptes à un specific individual in the organization, although reporting
individu précis de l'organisation, même si un rattachement administratif au administratively to the CEO is desirable.
directeur est souhaitable. Answer (C) is incorrect because following the long-range planning
La réponse (C) est fausse, car suivre le processus de planification à long process provides no guarantee of access.
terme ne donne aucune garantie concernant l'accès. Answer (D) is incorrect because the IAA should inform the board of
La réponse (D) est fausse. L'audit interne doit informer le conseil de toute any scope limitations, but its approval is not required.
limitation du champ d'intervention, mais l'aval de ce dernier n'est pas
obligatoire.

P1 - 10 -- Un auditeur interne chargé de vérifier qu'un fournisseur respecte P1 - 10 -- An internal auditor assigned to audit a vendor's
bien les normes de qualité des produits se trouve être le frère du contrôleur compliance with product quality standards is the brother of the
de gestion de ce fournisseur. Quelle attitude l'auditeur interne doit-il adopter ? vendor's controller. The auditor should:
A. Accepter la mission, mais éviter les contacts avec le contrôleur de gestion A. Accept the assignment, but avoid contact with the controller
pendant le travail sur le terrain. during fieldwork.
B. Accepter la mission, mais faire part de son lien de parenté dans les B. Accept the assignment, but disclose the relationship in the
communications finales de la mission. engagement final communication.
C. Avertir le fournisseur qu'il y a un risque de conflit d'intérêt. C. Notify the vendor of the potential conflict of interest.
D. Avertir le responsable de l'audit interne qu'il y a un risque de conflit D. Notify the chief audit executive of the potential conflict of interest.
d'intérêt.
La réponse (A) est fausse. Même si l'auditeur évite les contacts avec le Answer (A) is incorrect. Even if the auditor avoided contact with the
contrôleur de gestion, on aura tout de même l'impression qu'il y a un conflit controller, there would still be the appearance of conflict of interest.
d'intérêt. Answer (B) is incorrect. Situations of potential conflict of interest or
La réponse (B) est fausse. Les situations de conflits d'intérêt ou de préjugés bias should be avoided, not merely disclosed.
potentiels doivent être évitées, et non simplement mentionnées. Answer (C) is incorrect. Conflicts of interest should be reported to
La réponse (C) est fausse. Les conflits d'intérêt doivent être signalés au the chief audit executive, not the vendor or engagement client.
responsable de l'audit interne, et non au fournisseur ou au client de la Answer (D) is correct. The IPPF states that internal auditors
mission. should report to the chief audit executive any situations in
La réponse (D) est juste. Selon le CRIPP, les auditeurs internes doivent which a conflict of interest or bias is present or may
signaler au responsable de l'audit interne toutes les situations où l'on reasonably be inferred.
peut raisonnablement suspecter l'existence d'un conflit d'intérêt ou
d'un parti-pris.

Restitution

© IFACI – CIA1 V.5.1 page 6/73


Acquérir une certification
> Préparation au CIA, partie 1
P1 - 11 -- Au cours d'une mission, un auditeur interne établit, dans une P1 - 11 -- During the course of an engagement, an internal auditor
évaluation préliminaire, qu'un important service a financé de manière makes a preliminary determination that a major division has been
inappropriée des dépenses de recherche et développement. La mission n'est inappropriately capitalizing research and development expense.
pas encore achevée et l'auditeur interne n'a pas consigné ce problème dans The engagement is not yet completed, and the internal auditor has
des documents, ni déterminé si cela constitue réellement un problème. not documented the problem or determined that it really is a
Cependant, l'auditeur est informé que le responsable de l'audit interne a reçu problem. However, the internal auditor is informed that the chief
du président de l'organisation la communication suivante : « Le contrôleur de audit executive has received the following communication from the
la Division B m'informe que vous avez décelé une classification comptable president of the organization: The controller of Division B informs
suspecte ayant trait aux dépenses de recherche et développement. J'ai me that you have discovered a questionable account classification
connaissance de ce problème. Je vous demande de ne pas procéder à de dealing with research and development expense. We are aware of
plus amples investigations sur ce sujet jusqu'à nouvel ordre. En vertu des the issue. You are directed to discontinue any further investigation
règles de confidentialité en cours dans votre profession, je vous demande of this matter until informed by me to proceed. Under the
également de ne pas diffuser d'informations concernant ce problème à des confidentiality standard of your profession, I also direct you not to
auditeurs extérieurs.” Que doit faire le responsable de l'audit interne à propos communicate with the outside auditors regarding this issue. Which
de cet élément suspect ? of the following is an appropriate action for the CAE to take
regarding the questionable item?

A. Transmettre immédiatement cette communication à l'Institut de l'Audit A. Immediately report the communication to The Institute of Internal
Interne, lui en demander une interprétation déontologique et l'interroger sur la Auditors and ask for an ethical interpretation and guidance.
marche à suivre. B. Inform the president that this scope limitation will need to be
B. Informer le président que cette limitation du champ d'intervention devra reported to the board.
être notifiée au Conseil. C. Continue to investigate the area until all the facts are determined
C. Continuer d'enquêter sur cette affaire jusqu'à ce que les faits soient and document all the relevant facts in the engagement records.
confirmés ou infirmés et consigner tous les éléments pertinents dans le D. Immediately notify the external auditors of the problem to avoid
dossier de la mission. aiding and abetting a potential crime by the organization.
D. Informer immédiatement les auditeurs externes du problème, afin d'éviter
d'encourager un délit potentiel de l'organisation et de s'en rendre complice.
La réponse (A) est fausse. L'IIA n'a pas autorité en la matière. Answer (A) is incorrect because The IIA has no authority in this
La réponse (B) est juste. Une limitation du champ d'intervention et ses matter.
répercussions possibles doivent être communiquées, de préférence Answer (B) is correct. A scope limitation along with its
par écrit, au Conseil, au comité d'audit ou à tout autre organe délibérant potential effect should be communicated, preferably in
approprié (CRIPP). writing, to the board, audit committee, or other appropriate
La réponse (C) est fausse. Le responsable de l'audit interne doit au préalable governing authority (IPPF).
consulter le conseil. Il crée de la valeur ajoutée en servant l'organisation et le Answer (C) is incorrect because the CAE should first consult the
conseil peut, en fait, être pleinement conscient du problème mais ne pas board. The CAE adds value by serving the organization, and the
vouloir encourir de frais supplémentaires. board may, in fact, be fully aware of the problem and may not want
La réponse (D) est fausse. Le travail est préliminaire, et l'auditeur interne n'a to incur additional costs.
pas encore étayé son opinion. Il serait donc prématuré de contacter les Answer (D) is incorrect because the engagement work is preliminary,
auditeurs externes. Néanmoins, si ces derniers procèdent à une enquête, les and the internal auditor has not yet formed a basis for an opinion.
auditeurs internes doivent leur communiquer les travaux réalisés à ce jour. Thus, contacting the external auditors is premature. However, if an
inquiry is made by the external auditors, the internal auditors should
share the work done to date.

Restitution

© IFACI – CIA1 V.5.1 page 7/73


Acquérir une certification
> Préparation au CIA, partie 1
P1 - 12 -- Il existe un risque d'altération de l'indépendance et de l'objectivité P1 - 12 -- Independence and objectivity may be impaired if
lorsque les missions d'assurance sont réalisées au cours de l'année qui suit assurance services are provided within one year after a formal
une mission de conseil formelle. Quelles mesures peuvent être prises pour consulting engagement. What steps can be taken to minimize the
limiter le plus possible les conséquences de cette altération ? effects of this kind of impairment?
I. Affecter des auditeurs distincts entre les missions de conseil et d'assurance. I. Assigning different auditors to perform each of the services.
II. Désigner des managers et des superviseurs différents. II. Establishing independent management and supervision.
III. Définir des responsabilités distinctes pour les résultats des projets. III. Defining separate accountability for the results of projects.
IV. Informer d'une possible altération de l'indépendance et de l'objectivité. IV. Disclosing the presumed impairment.
A. I et III. A. I and III.
B. I, III et IV. B. I, III, and IV.
C. II, III et IV. C. II, III, and IV.
D. I, II, III et IV. D. I, II, III, and IV.
La réponse (A) est fausse, car toutes ces mesures peuvent être prises pour Answer (A) is incorrect because all of the items are steps that can
limiter le plus possible les conséquences de l'altération de l'indépendance et be taken to minimize the effects of impairment.
de l'objectivité. Answer (B) is incorrect because all of the items are steps that can
La réponse (B) est fausse, car toutes ces mesures peuvent être prises pour be taken to minimize the effects of impairment.
limiter le plus possible les conséquences de l'altération de l'indépendance et Answer (C) is incorrect because all of the items are steps that can
de l'objectivité. be taken to minimize the effects of impairment.
La réponse (C) est fausse, car toutes ces mesures peuvent être prises pour Answer (D) is correct. Independence and objectivity may be
limiter le plus possible les conséquences de l'altération de l'indépendance et impaired if assurance services are provided within one year
de l'objectivité. after a formal consulting engagement. Steps can be taken to
La réponse (D) est juste. Il existe un risque d'altération de minimize the effects of impairment by assigning different
l'indépendance et de l'objectivité lorsque des missions d'assurance auditors to perform each of the services, establishing
sont réalisées au cours de l'année qui suit une mission de conseil independent management and supervision, defining separate
formelle. Pour limiter le plus possible les conséquences de cette accountability for the results of the projects, and disclosing
altération, on peut affecter des auditeurs distincts entre les missions de the presumed impairment. Management should be
conseil et d'assurance, désigner des managers et des superviseurs responsible for accepting and implementing
différents, instaurer des responsabilités distinctes pour les résultats recommendations (IPPF).
des projets et informer d'une possible altération de l'indépendance et
de l'objectivité. C'est à la direction de valider et de mettre en oeuvre les
recommandations (CRIPP).

P1 - 13 -- La place de l'audit interne dans l'organisation : P1 - 13 -- The organizational status of the internal audit activity
A. Doit être suffisante pour lui permettre de remplir ses fonctions. A. Should be sufficient to permit the accomplishment of its
B. Est optimale lorsqu'il rend directement compte au conseil d'administration. responsibilities.
C. Nécessite, annuellement, l'approbation des programmes de travail de B. Is best when the reporting relationship is direct to the board of
l'audit interne, des prévisions d'effectifs et du budget financier par le conseil. directors.
D. Est garantie lorsque la charte définit spécifiquement son indépendance. C. Requires the board's annual approval of the engagement work
schedule, staffing plan, and financial budget.
D. Is guaranteed when the charter specifically defines its
independence.

La réponse (A) est juste. “L'audit interne doit être indépendant” Answer (A) is correct. The IAA should be independent (IPPF).
(CRIPP). “Les auditeurs internes sont indépendants lorsqu'ils peuvent Internal auditors are independent when they can carry out
exercer leur activité librement et de façon objective. Cet objectif est their work freely and objectively, and independence is
atteint grâce à leur position dans l'organisation et à leur objectivité.” achieved through organizational status and objectivity (IPPF).
(CRIPP). Ainsi, “le responsable de l'audit interne doit relever d'un Thus, the chief audit executive should report to a level within
niveau hiérarchique permettant aux auditeurs internes d'exercer leurs the organization that allows the IAA to accomplish its
responsabilités” (CRIPP). “Les auditeurs internes doivent bénéficier du responsibilities (IPPF). Internal auditors should have the
soutien de la direction générale et du Conseil afin d'obtenir la support of management and of the board so that they can
coopération des audités et de pouvoir exercer leur activité sans gain the cooperation of engagement clients and perform their
entrave” (CRIPP). work free from interference (IPPF).
La réponse (B) est fausse. L'audit interne doit bénéficier d'un soutien Answer (B) is incorrect because the IAA requires day-to-day
quotidien qui ne peut lui être fourni par le conseil. Il doit tout de même rendre support that cannot be provided by the board. It should still report to
compte à la direction. Dans l'idéal, l'audit interne doit être rattaché management. Ideally, the IAA should report administratively to the
administrativement au directeur général de l'organisation. CEO of the organization.
La réponse (C) est fausse, car “le responsable de l'audit interne doit Answer (C) is incorrect because the CAE should submit annually to
soumettre, annuellement, à la direction générale pour approbation et au senior management for approval, and to the board for its
Conseil pour information, les programmes de travail de l'audit interne, les information, a summary of the IAA's work schedule, staffing plan,
prévisions d'effectifs et le budget financier” (CRIPP). and financial budget (IPPF).
La réponse (D) est fausse. Un fait énoncé dans la charte ne garantit pas Answer (D) is incorrect because a statement in the charter does
l'indépendance. not guarantee independence.

Restitution

© IFACI – CIA1 V.5.1 page 8/73


Acquérir une certification
> Préparation au CIA, partie 1
P1 - 14 -- L’audit interne rencontre une limitation à son champ d’action P1 - 14 -- The internal auditing activity encounters a scope
imposée par la direction générale qui l’empêchera de remplir sa mission et limitation from senior management that will affect its ability to meet
d’atteindre ses objectifs pour un client de mission potentiel. La nature de cette its goals and objectives for a potential engagement client. The
limitation du champ d’action devra être : nature of the scope limitation should be
A. Notée dans les papiers de travail de la mission mais la mission devra être A. Noted in the engagement working papers, but the engagement
effectuée comme prévu tout en respectant dans la mesure du possible la should be carried out as scheduled and the scope limitation worked
limitation au champ d’action. around, if possible.
B. Communiquée aux auditeurs externes pour qu’ils puissent examiner ce B. Communicated to the external auditors so they can investigate
domaine plus en détail. the area in more detail.
C. Communiquée de préférence par écrit au conseil d’administration. C. Communicated, preferably in writing, to the board.
D. Communiquée à la direction en disant que la limitation ne sera pas D. Communicated to management stating that the limitation will not
acceptée car elle porterait atteinte à l’indépendance de l’audit interne. be accepted because it would impair the internal audit activity's
independence.

La réponse (A) est fausse. La limitation devra être communiquée en premier Answer (A) is incorrect because the limitation should be
lieu au conseil d’administration. communicated first to the board.
La réponse (B) est fausse. Il n’y a aucune obligation ni aucun besoin de Answer (B) is incorrect because there is no requirement or need to
communiquer la limitation à l’auditeur externe. communicate the limitation to the external auditor.
La réponse (C) est juste. Les plannings des travaux de mission les Answer (C) is correct. Engagement work schedules, staffing
plannings des effectifs ainsi que les budgets devraient faire état vis-à- plans, and financial budgets should inform senior
vis de la direction générale et du conseil d’administration de l’étendue management and the board of the scope of internal auditing
du travail de l’audit interne et de toute limitation imposée sur ce champ work and of any limitations placed on that scope (IPPF).
d’action (CRIPP). De plus une limitation à un champ d’action avec son Furthermore, a scope limitation along with its potential effect
impact éventuel devra être signalé de préférence par écrit au conseil should be communicated, preferably in writing, to the board,
d’administration au comité d’audit ou à toute autre organe délibérant audit committee, or other appropriate governing authority
(CRIPP). (IPPF).
La réponse (D) est fausse. L’AI existe afin d’aider l’organisation à atteindre Answer (D) is incorrect because the IAA exists to help the
ses objectifs. Les auditeurs internes devront donc communiquer ses conflits organization achieve its objectives. Thus, the internal auditors
avec la direction au conseil d’administration. should communicate with the board about conflicts with
management.

P1 - 15 -- À partir du moment où le Responsable de l’Audit Interne reçoit P1 - 15 -- After the Chief Audit Executive gets approval from the
l’approbation du Conseil d’Administration pour fournir des services de conseil Board of Directors to offer consulting services, what should be
quelle action doit être entreprise ? done?
A. Le RAI devrait commencer à fournir les services de conseil. A. The CAE should begin performing consulting services.
B. Le RAI devrait obtenir l’approbation du Comité d’Audit. B. The CAE should get approval from the audit committee.
C. La Charte de l’audit interne devrait être modifiée. C. The internal audit charter should be amended.
D. Le Conseil d’Administration devrait établir les politiques et les procédures D. The board should develop appropriate policies and procedures
adéquates pour pouvoir mener de telles missions. for conducting such engagements.
La réponse (A) est fausse. Après avoir obtenu l’approbation du Conseil Answer (A) is incorrect because, after the CAE gets board
d’administration, la charte de l’audit interne devra être modifiée et l’activité de approval, the audit charter should be amended and the internal
l’audit interne devra définir les politiques et les procédures appropriées pour audit activity should develop appropriate policies and procedures
mener de telles missions. for conducting such engagements.
La réponse (B) est fausse. Le RAI n’a pas besoin d’obtenir l’approbation du Answer (B) is incorrect because the CAE does not need to get
comité d’audit. L’approbation du conseil d’administration suffit. additional approval from the audit committee; only board approval
La réponse (C) est juste. De temps à autre on demande aux auditeurs is required.
internes d’effectuer des services de conseil dans des domaines où Answer (C) is correct. Internal auditors are sometimes
précédemment ils ont eu des positions de responsabilité ou pour requested to provide consulting services relating to
lesquels ils ont conduit une mission d’assurance. Avant de proposer un operations for which they had previous responsibilities or had
service de conseil, le RAI devra s’assurer que le Conseil d’Administration conducted assurance services. Prior to offering consulting
comprenne et donne son approbation au concept de fournir des services, the Chief Audit Executive should confirm that the
prestations de conseil. Une fois cette approbation obtenue la charte de board understands and approves the concept of providing
l’audit interne devra être modifiée afin d’inclure l’autorisation et les consulting services. Once approved, the internal audit charter
responsabilités pour les activités de conseil et le département d’audit should be amended to include authority and responsibilities
interne devra définir les politiques et les procédures nécessaires pour for consulting activities, and the internal audit activity should
mener de telles missions. (CRIPP) develop appropriate policies and procedures for conducting
La réponse (D) est fausse. L’audit interne devra définir les politiques et les such engagements. (IPPF)
procédures nécessaires pour mener de telles missions. Answer (D) is incorrect because the internal audit activity should
develop appropriate policies and procedures for conducting such
engagements.

Restitution

© IFACI – CIA1 V.5.1 page 9/73


Acquérir une certification
> Préparation au CIA, partie 1
P1 - 16 -- Quel type de normes s'applique aux organisations et aux individus P1 - 16 -- Which type of standards applies to organizations and
réalisant les missions d'audit interne spécifiques définies par les Normes ? individuals performing the specific internal auditing services
delineated by these Standards?
A. Les Normes pour la pratique professionnelle de l'audit interne - Normes de
fonctionnement. A. Standards for the Professional Practice of Internal Auditing -
B. Les Normes pour la pratique professionnelle de l'audit interne - Normes de Performance Standards.
qualification. B. Standards for the Professional Practice of Internal Auditing -
C. Les Normes pour la pratique professionnelle de l'audit interne - Normes de Attribute Standards.
mise en oeuvre. C. Standards for the Professional Practice of Internal Auditing -
D. Toutes ces catégories de normes. Implementation Standards.
D. All of these types of standards.

La réponse (A) est fausse. Ces Normes définissent les normes de Answer (A) is incorrect because they apply to Standards for
fonctionnement des organisations et des individus réalisant des activités Performance of organizations and individuals performing internal
d'audit interne. auditing services.
La réponse (B) est fausse. Ces Normes définissent les normes de qualification Answer (B) is incorrect because they apply to Standards for
des organisations et des individus réalisant des activités d'audit interne. Attributes of organizations and individuals performing internal
La réponse (C) est juste. D'après le Cadre de référence des pratiques auditing services.
professionnelles, les Normes pour la pratique professionnelle de l'audit Answer (C) is correct. According to the Professional Practices
interne - Normes de mise en oeuvre s'appliquent aux organisations et Framework, Standards for the Professional Practice of
aux personnes réalisant des types de mission d'audit interne Internal Auditing - Implementation Standards apply to
spécifiques définies dans ces Normes, à savoir essentiellement des organizations and individuals performing the specific internal
missions d'assurance et de conseil. Elles se divisent en deux grandes auditing services delineated by these Standards. Board
catégories : les Normes de mise en oeuvre pour les missions categories are assurance services and consulting services
d'assurance et celles pour les missions de conseil. Implementation Standards.
La réponse (D) est fausse, car seules les Normes de mise en oeuvre Answer (D) is incorrect because only Implementation standards
s'appliquent à des organisations et personnes réalisant des catégories de apply to organizations and individuals performing the specific
missions d'audit interne spécifiques définies dans ces Normes. internal auditing services delineated by these Standards.

P1 - 17 -- Un service d'audit interne a planifié une mission portant sur un P1 - 17 -- An internal audit activity has scheduled an engagement
contrat de BTP. Une partie de cette mission consistera à comparer les relating to a construction contract. One portion of this engagement
matériaux achetés à ceux spécifiés dans les plans et dessins techniques. Le will include comparing materials purchased with those specified in
service de l'audit interne ne dispose pas d'une personne ayant des the engineering drawings. The IAA does not have anyone on staff
compétences suffisantes pour mener à bien cette tâche. Son responsable with sufficient expertise to complete this procedure. The chief audit
doit donc : executive should
A. Annuler la mission. A. Delete the engagement from the schedule.
B. Faire appel au personnel existant pour réaliser l'ensemble de la mission. B. Perform the entire engagement using current staff.
C. Recourir à un consultant en ingénierie pour qu'il procède à la comparaison C. Engage an engineering consultant to perform the comparison.
nécessaire. D. Accept the contractor's written representations.
D. Accepter les déclarations écrites de l'entrepreneur.
La réponse (A) est fausse. Cette mission fait partie du champ de l'audit interne. Answer (A) is incorrect because the engagement is within the scope of
La réponse (B) est fausse. Il n'est pas approprié de faire appel au personnel the IAA.
existant (non qualifié) pour réaliser cette mission. Answer (B) is incorrect because performing the engagement using
La réponse (C) est juste. Le service d'audit interne doit disposer d'un the current (unqualified) staff is inappropriate.
personnel qualifié ou recourir à des prestataires extérieurs qualifiés Answer (C) is correct. The IAA should have employees or use
dans des disciplines telles que la comptabilité, l'audit, l'économie, la outside service providers who are qualified in such
finance, les statistiques, les technologies de l'information, l'ingénierie, disciplines as accounting, auditing, economics, finance,
la fiscalité, le droit ou l'environnement, et dans d'autres domaines, afin statistics, information technology, engineering, taxation, law,
de pouvoir exercer ses responsabilités. Chaque membre de ce service environmental affairs, and such other areas as needed to
n'a cependant pas besoin d'être qualifié dans toutes ces disciplines meet the IAA's responsibilities. Each member of the IAA,
(CRIPP). Par conséquent, pour faire preuve de conscience however, need not be qualified in all of these disciplines
professionnelle, il est nécessaire de recourir à un prestataire extérieur. (IPPF). Thus, hiring an outside service provider is also
La réponse (D) est fausse. Il n'est pas approprié d'accepter les déclarations necessary as an exercise of due professional care.
écrites de l'entrepreneur sans procéder à une vérification adéquate. Answer (D) is incorrect because accepting the contractor's
representations without adequate testing is inappropriate.

Restitution

© IFACI – CIA1 V.5.1 page 10/73


Acquérir une certification
> Préparation au CIA, partie 1
P1 - 18 -- Afin qu'une mission soit menée d'un bout à l'autre avec conscience P1 - 18 -- To ensure that due professional care has been taken at
professionnelle, l'auditeur interne doit toujours : all times during an engagement, the internal auditor should always:
A. Veiller à ce que toutes les informations financières liées à l'audit soient A. Ensure that all financial information related to the audit is
incluses dans le plan d'audit et examinées afin de repérer les déviations ou included in the audit plan and examined for nonconformance or
les irrégularités. irregularities.
B. Veiller à ce que tous les tests d'audit soient pleinement documentés. B. Ensure that all audit tests are fully documented.
C. Envisager la possibilité de déviations ou d'irrégularités à tout moment au C. Consider the possibility of nonconformance or irregularities at all
cours de la mission. times during an engagement.
D. Communiquer rapidement au comité d'audit tous les cas de déviations ou D. Communicate any noncompliance or irregularity discovered
d'irrégularités apparus au cours de la mission. during an engagement promptly to the audit committee.
La réponse (A) est fausse. L'inclusion automatique d'informations financières Answer (A) is incorrect. The automatic inclusion of financial
dans un audit ne garantit pas que les auditeurs font preuve de conscience information in an audit does not guarantee that due professional
professionnelle pour l'ensemble de la mission. care has been achieved for the audit as a whole.
La réponse (B) est fausse. Le fait de conserver des papiers de travail détaillés Answer (B) is incorrect. Keeping detailed working papers does not
ne garantit pas que l'auditeur fait preuve de conscience professionnelle pendant ensure that due professional care has been taken during the tests.
les contrôles. Answer (C) is correct. Considering the possibility of
La réponse (C) est juste. Selon le CRIPP, envisager la possibilité de nonconformance or material irregularities at all times during
déviations ou d'irrégularités à tout moment au cours d'une mission an engagement is the only way of demonstrating that due
constitue la seule manière de démontrer que les auditeurs font preuve professional care has been taken in an internal audit
de conscience professionnelle lors de leur mission. assignment, according to IPPF.
La réponse (D) est fausse. La conscience professionnelle n'oblige pas Answer (D) is incorrect. Due professional care does not require that
l'auditeur à signaler au comité d'audit toutes les déviations ou irrégularités. all instances of noncompliance or irregularity be reported to the audit
committee.

Restitution

© IFACI – CIA1 V.5.1 page 11/73


Acquérir une certification
> Préparation au CIA, partie 1
P1 - 19 -- Jacques mène une mission de conseil formelle pour la société P1 - 19 -- Jack is conducting a formal consulting engagement for
XYZ. Outre les conditions d'indépendance et d'objectivité, ainsi que la XYZ Corp. In addition to the independence and objectivity
conscience professionnelle requise, laquelle des tâches suivantes doit-il evaluation and due professional care considerations, Jack should
exécuter ? do which of the following?
I. Document general terms, understandings, deliverables, and
I. Formaliser, dans un accord ou plan écrit, les termes généraux, la other key factors of the formal consulting engagement in a
compréhension, les résultats attendus et les autres facteurs clés de la written agreement or plan.
mission de conseil. II. Evaluate the consulting engagement for comparability with the
II. Évaluer la mission de conseil pour permettre la comparaison avec le internal audit activity's overall plan of engagements.
programme global de missions du service d'audit interne. III. Conduct appropriate meetings and gather necessary
III. Organiser les réunions appropriées et recueillir les informations information to assess the nature and extent of the services to
nécessaires pour évaluer la nature et l'étendue des prestations à fournir. be provided.
A. I et II. A. I and II.
B. II et III. B. II and III.
C. I et III. C. I and III.
D. I, II et III. D. I, II, and III.

La réponse (A) est fausse, car tous les éléments cités sont des tâches que Answer (A) is incorrect because all of the items are things that the
l'auditeur interne doit exécuter afin de mener à bien une mission de conseil internal auditor should do when performing a formal consulting
formelle, outre les conditions d'indépendance et d'objectivité ainsi que la engagement in addition to the independence and objectivity
conscience professionnelle requise. evaluation and due professional care considerations.
La réponse (B) est fausse, car tous les éléments cités sont des tâches que Answer (B) is incorrect because all of the items are things that the
l'auditeur interne doit exécuter afin de mener à bien une mission de conseil internal auditor should do when performing a formal consulting
formelle, outre les conditions d'indépendance et d'objectivité ainsi que la engagement in addition to the independence and objectivity
conscience professionnelle requise. evaluation and due professional care considerations.
La réponse (C) est fausse, car tous les éléments cités sont des tâches que Answer (C) is incorrect because all of the items are things that the
l'auditeur interne doit exécuter afin de mener à bien une mission de conseil internal auditor should do when performing a formal consulting
formelle, outre les conditions d'indépendance et d'objectivité ainsi que la engagement in addition to the independence and objectivity
conscience professionnelle requise. evaluation and due professional care considerations.
La réponse (D) est juste. Outre les conditions d'indépendance et Answer (D) is correct. In addition to the independence and
d'objectivité ainsi que la conscience professionnelle requise, l'auditeur objectivity evaluation and due professional care
interne doit : organiser les réunions appropriées et recueillir les considerations, the internal auditor should: Conduct
informations nécessaires pour évaluer la nature et l'étendue des appropriate meetings and gather necessary information to
prestations à fournir ; s'assurer que les bénéficiaires des prestations assess the nature and extent of the service to be provided.
adhèrent aux dispositions de la Charte d'audit interne, aux règles et Confirm that those receiving the service understand and
procédures d'audit interne et aux autres dispositions régissant la agree with the relevant guidance contained in the internal
conduite des missions de conseil. L'auditeur interne doit refuser les audit charter, internal audit activity's policies and procedures,
missions de conseil qui sont prohibées par la Charte d'audit interne, and other related guidance governing the conduct of
qui sont contraires aux règles et procédures du service d'audit interne, consulting engagements. The internal auditor should decline
ou qui n'apportent pas de valeur ajoutée ni ne répondent aux intérêts to perform consulting engagements that are prohibited by the
de l'organisation ; s'assurer de la compatibilité de la mission de conseil terms of the internal audit charter, conflict with the policies
avec le plan d'audit. Ce plan, fondé sur une analyse des risques, peut and procedures of the internal audit activity, or do not add
intégrer et s'appuyer sur des missions de conseil, dans la mesure où value and promote the best interests of the organization.
elles contribuent à assurer la couverture d'audit nécessaire à Evaluate the consulting engagement for compatibility with the
l'organisation ; formaliser, dans un accord ou plan écrit, les termes internal audit activity's overall plan of engagements. The
généraux, la compréhension, les produits attendus et les autres internal audit activity's risk-based plan of engagements may
facteurs clés de la mission de conseil. Il est essentiel que l'auditeur incorporate and rely on consulting engagements, to the
interne et les bénéficiaires des prestations de conseil définissent d'un extent deemed appropriate, to provide necessary audit
commun accord les exigences en termes de rapport et de coverage to the organization. Document general terms,
communication. (CRIPP) understandings, deliverables, and other key factors of the
formal consulting engagement in a written agreement or plan.
It is essential that both the internal auditor and those
receiving the consulting engagement understand and agree
with the reporting and communication requirements. (IPPF)

Restitution

© IFACI – CIA1 V.5.1 page 12/73


Acquérir une certification
> Préparation au CIA, partie 1
P1 - 20 -- Alors qu'il est sur le point de terminer une mission, un auditeur P1 - 20 -- An auditor, nearly finished with an engagement,
découvre que le directeur du marketing aime le jeu. Cette question n'est pas discovers that the director of marketing has a gambling habit. The
directement liée à la mission en cours et l'auditeur doit terminer sa mission au gambling issue is not directly related to the existing engagement
plus vite. L'auditeur note le problème et transmet l'information au responsable and there is pressure to complete the current engagement. The
de l'audit interne, mais n'effectue aucun suivi sur ce point. L'auditeur : auditor notes the problem and forwards the information to the chief
audit executive but performs no further follow-up. The auditor's
actions would:
A. Enfreint le Code de déontologie de l'IIA, car il fait de la rétention A. Be in violation of the IIA Code of Ethics for withholding
d'informations importantes. meaningful information.
B. Enfreint les Normes car il n'a pas effectué un suivi correct après un signal B. Be in violation of the Standards because the auditor did not
d'alerte susceptible d'indiquer l'existence d'une fraude. properly follow up on a red flag that might indicate the existence of
C. N'enfreint ni le Code de déontologie de l'IIA ni les Normes. fraud.
D. A et B. C. Not be in violation of either the IIA Code of Ethics or Standards.
D. Both A and B.

La réponse (A) est fausse. L'auditeur ne fait pas de la rétention d'information. Answer (A) is incorrect. The auditor is not withholding information
L'information a en effet bien été communiquée au responsable de l'audit because the information has been forwarded to the chief audit
interne. Elle peut se révéler utile lors d'une mission ultérieure dans le service executive. The information may be useful in a subsequent
marketing. engagement in the marketing area.
La réponse (B) est fausse. L'auditeur a noté un signal d'alerte qui peut se Answer (B) is incorrect. The auditor has documented a red flag that
révéler important lors d'une mission ultérieure. Il n'enfreint pas les Normes. may be important in a subsequent engagement. This does not violate
La réponse (C) est juste. L'auditeur n'enfreint ni le Code de déontologie the Standards.
ni les Normes. Voir réponses A et B. Answer (C) is correct. There is no violation of either the Code
La réponse (D) est fausse. Voir réponses A et B. of Ethics or the Standards. See answers "a" and "b".
Answer (D) is incorrect. See answers "a" and "b".

P1 - 21 -- Laquelle des activités suivantes procure un retour d'information sur P1 - 21 -- Which of the following activities are designed to provide
l'efficacité d'un service d'audit interne ? feedback on the effectiveness of an internal audit function?
I. Une supervision appropriée. I. Proper supervision.
II. Une bonne formation. II. Proper training.
III. Des évaluations internes. III. Internal assessments.
IV. Des évaluations externes. IV. External assessments.
A. I, II et III uniquement. A. I, II, and III only.
B. I, II et IV uniquement. B. I, II, and IV only.
C. I, III et IV uniquement. C. I, III, and IV only.
D. II, III et IV uniquement. D. II, III, and IV only.
La réponse (A) est fausse. Une bonne formation est importante, mais elle ne Answer (A) is incorrect. Proper training is important but it does not
procure pas de retour d'information. provide feedback.
La réponse (B) est fausse. Une bonne formation est importante, mais elle ne Answer (B) is incorrect. Proper training is important but it does not
procure pas de retour d'information. provide feedback.
La réponse (C) est juste. Un programme d'assurance qualité est destiné Answer (C) is correct. Quality assurance programs are
à procurer un retour d'information sur l'efficacité de l'audit interne. Il designed to provide feedback on the effectiveness of an
doit inclure la supervision, qui apporte un retour d'information au jour internal audit function. A quality assurance program should
le jour. Un programme d'assurance qualité doit comporter des include supervision, which provides day-to-day feedback. A
évaluations internes et externes. quality assurance program should include internal and
La réponse (D) est fausse. Une bonne formation est importante, mais elle ne external assessments.
procure pas de retour d'information. Answer (D) is incorrect. Proper training is important but it does not
provide feedback.

Restitution

© IFACI – CIA1 V.5.1 page 13/73


Acquérir une certification
> Préparation au CIA, partie 1
P1 - 22 -- Laquelle des propositions suivantes fait partie du programme P1 - 22 -- Which of the following is part of an internal audit activity's
d'assurance qualité de l'audit interne, et non des autres attributions du quality assurance program, rather than being included as part of
responsable de l'audit interne ? other responsibilities of the chief audit executive (CAE)?
A. Le responsable de l'audit interne procure aux auditeurs externes des A. The CAE provides information about and access to internal audit
informations sur les papiers de travail de l'audit interne et leur donne accès à workpapers to the external auditors to enable them to understand
ces papiers afin de leur permettre de comprendre et de déterminer dans and determine the degree to which they may rely on the internal
quelle mesure ils peuvent s'appuyer sur les travaux des auditeurs internes. auditors' work.
B. La direction approuve une charte formelle qui définit l'objectif, les pouvoirs B. Management approves a formal charter establishing the
et les responsabilités de l'audit interne. purpose, authority, and responsibility of the internal audit activity.
C. Les performances de chaque auditeur interne sont évaluées au moins une C. Each individual internal auditor's performance is appraised at
fois par an. least annually.
D. Le travail d'un auditeur interne est supervisé du début à la fin de chaque D. Supervision of an internal auditor's work is performed throughout
mission. each audit engagement.
La réponse (A) est fausse. Elle concerne la responsabilité du responsable de Answer (A) is incorrect. This statement relates to the responsibility
l'audit interne qui est d'assurer la coordination avec les auditeurs externes of the chief audit executive (CAE) to coordinate with external
(CRIPP). auditors (IPPF).
La réponse (B) est fausse. La fonction du responsable de l'audit interne est Answer (B) is incorrect. A CAE's responsibility to seek approval of a
de faire adopter une charte qui définit la mission, les pouvoirs et les charter which establishes authority, purpose, and responsibility
responsabilités (CRIPP) ne fait pas partie d'un programme d'assurance (IPPF) is not part of a quality assurance program.
qualité. Answer (C) is incorrect. Individual performance appraisals are part
La réponse (C) est fausse. L'évaluation des performances individuelles fait of a CAE's responsibility toward personnel management and
partie des responsabilités du responsable de l'audit interne en termes de development (IPPF).
gestion et de développement du personnel (CRIPP). Answer (D) is correct. Supervision is one method of ongoing
La réponse (D) est juste. La supervision est une méthode d'évaluation review, which is part of the internal assessment aspect of
interne continue, qui fait partie du volet évaluation interne de quality assurance (IPPF).
l'assurance qualité (CRIPP).
P1 - 23 -- Au cours d'une mission d'assurance, un auditeur interne ne P1 - 23 -- An internal auditor fails to discover an employee fraud
découvre pas une fraude perpétrée par un salarié. Le fait de n'avoir pas pu during an assurance engagement. The non discovery is most likely
découvrir cette fraude provient vraisemblablement d'une violation des to suggest a violation of the Standards if it was the result of a
Normes s'il découle du fait que :
A. Failure to perform a detailed review of all transactions in the
A. L'auditeur n'a pas effectué une analyse détaillée de toutes les opérations area.
réalisées dans ce service. B. Determination that any possible fraud in the area would not
B. L'auditeur a conclu que toute fraude possible dans ce service n'impliquerait involve a material amount.
pas de montant conséquent. C. Determination that the cost of extending procedures in the area
C. L'auditeur a conclu que le coût de l'élargissement des procédures dans ce would exceed the potential benefits.
service dépasserait les avantages attendus. D. Presumption that the internal controls in the area were adequate
D. L'auditeur a supposé que les contrôles internes dans ce service étaient and effective.
adéquats et efficaces.
Answer (A) is incorrect because due professional care does not
La réponse (A) est fausse. La conscience professionnelle n'impose pas de require detailed reviews of all transactions (IPPF).
procéder à un examen détaillé et systématique de toutes les opérations Answer (B) is incorrect because the relative complexity, materiality,
(CRIPP). or significance of matters to which assurance procedures are
La réponse (B) est fausse. La complexité relative, l'importance ou le applied should be considered.
caractère significatif des domaines auxquels sont appliquées les procédures Answer (C) is incorrect because the internal auditor should
propres aux missions d'assurance doivent être pris en considération. consider the cost of assurance in relation to potential benefits.
La réponse (C) est fausse. L'auditeur interne doit prendre en considération le Answer (D) is correct. Exercising due professional care
coût des contrôles d'assurance par rapport aux avantages escomptés. entails considering the adequacy and effectiveness of risk
La réponse (D) est juste. L'exercice de la conscience professionnelle management, control, and governance processes (IPPF).
implique de prendre en considération la pertinence et l'efficacité des
processus de management des risques, de contrôle et de
gouvernement d'entreprise (CRIPP).

Restitution

© IFACI – CIA1 V.5.1 page 14/73


Acquérir une certification
> Préparation au CIA, partie 1
P1 - 24 -- En ce qui concerne le soin apporté à sa pratique professionnelle, P1 - 24 -- With regard to the exercise of due professional care, an
un auditeur interne doit: internal auditor should
A. Prendre en considération la matérialité ou le caractère significatif des A. Consider the relative materiality or significance of matters to
domaines auxquels les procédures, propres aux missions d’assurance, which assurance procedures are applied.
s’applique. B. Emphasize the potential benefits of an engagement without
B. Insister sur les avantages potentiels d’une mission sans se préoccuper regard to the cost.
des coûts. C. Consider whether established operating standards are being
C. Considérer si les critères opérationnels établis sont effectivement atteints met and not whether those standards are acceptable.
sans se préoccuper de savoir si ces critères sont acceptables. D. Select procedures that are likely to provide absolute assurance
D. Opter pour des procédures susceptibles de fournir une assurance that irregularities do not exist.
infaillible qu’aucune irrégularité ne puisse exister.
La réponse (A) est juste. Le soin apporté à sa pratique professionnelle Answer (A) is correct. Exercising due professional care
signifie la diligence et le savoir-faire que l’on peut attendre d’un means applying the care and skill expected of a reasonably
auditeur interne raisonnablement averti et compétent (CRIPP). Selon le prudent and competent internal auditor (IPPF). According to
CRIPP, l’auditeur interne doit prendre en considération l’étendue du IPPF, the internal auditor should consider the Extent of work
travail nécessaire pour atteindre les objectifs de la mission. La needed to achieve the engagement's objectives. Relative
complexité relative, la matérialité, ou le caractère significatif des complexity, materiality, or significance of matters to which
domaines auxquels sont appliquées les procédures propres aux assurance procedures are applied. Adequacy and
missions d’assurance. La pertinence et l’efficacité des processus de effectiveness of risk management, control, and governance
management des risques, de contrôle et de gouvernement d’entreprise. processes. Probability of significant errors, irregularities, or
La probabilité d’erreurs, d’irrégularités ou de non-conformités noncompliance. Cost of assurance in relation to potential
significatives. Le coût de la mise en place des contrôles par rapport aux benefits.
avantages escomptés. Answer (B) is incorrect because the internal auditor should
La réponse (B) est fausse. L’auditeur interne doit prendre en compte les consider the cost in relation to the benefits before beginning an
coûts par rapport aux bénéfices avant d’entamer une mission. engagement.
La réponse (C) est fausse. Les auditeurs internes doivent évaluer les objectifs Answer (C) is incorrect because internal auditors should evaluate
et les buts qui ont été établis et ils doivent déterminer si ceux-ci sont adéquats established operating targets and expectations and determine
et réalisés (CRIPP). whether those standards are acceptable and are being met (IPPF).
La réponse (D) est fausse. Les auditeurs internes ne peuvent pas fournir une Answer (D) is incorrect because internal auditors cannot give
assurance absolue que la non-conformité, les erreurs, ou les irrégularités absolute assurance that noncompliance, errors, or irregularities do
n’existent pas (CRIPP). not exist (IPPF).

P1 - 25 -- Le responsable de l'audit interne doit élaborer et tenir à jour un P1 - 25-- The chief audit executive should develop and maintain a
programme d'assurance et d'amélioration de la qualité qui couvre tous les quality assurance and improvement program that covers all
aspects de l'audit interne et effectue un suivi continu de son efficacité. Tous aspects of the internal audit activity and continuously monitors its
les éléments suivants sont compris dans un programme qualité, sauf effectiveness. All of the following are included in a quality program
except
A. L'évaluation annuelle des performances de chaque auditeur interne.
B. L'évaluation interne périodique. A. Annual appraisals of individual internal auditors' performance.
C. La supervision. B. Periodic internal assessment.
D. Les évaluations externes périodiques. C. Supervision.
D. Periodic external assessments.
La réponse (A) est juste. Un programme d'assurance et d'amélioration Answer (A) is correct. A quality assurance and improvement
qualité doit être conçu de manière à fournir aux différentes parties program should be designed to provide reasonable
prenantes de l'audit interne l'assurance raisonnable (1) qu'il respecte sa assurance to the various stakeholders of the IAA that it (1)
charte, qui doit elle-même se conformer aux Normes et au Code de performs in accordance with its charter, which should be
Déontologie, (2) qu'il est efficient et efficace et (3) qu'il est perçu par les consistent with the Standards and the Code of Ethics; (2)
parties prenantes comme contribuant à la création de valeur ajoutée et operates effectively and efficiently; and (3) is perceived by the
à l'amélioration du fonctionnement de l'organisation. Le programme stakeholders as adding value and improving operations. The
doit comporter une supervision adéquate, une évaluation interne program should include appropriate supervision, periodic
périodique et un suivi continu de l'assurance qualité, ainsi que des internal assessment and ongoing monitoring of quality
évaluations externes périodiques (CRIPP). Le programme de assurance, and periodic external assessments (IPPF). The
recrutement et de développement des ressources humaines de l'audit program for selecting and developing the human resources of
interne doit prévoir, entre autres, l'évaluation des performances de the IAA provides for, among other things, appraising each
chaque auditeur interne, au moins une fois par an (CRIPP). internal auditor's work at least annually (IPPF).
La réponse (B) est fausse. L'évaluation interne constitue un élément d'un Answer (B) is incorrect because internal assessment is an element
programme qualité. of a quality program.
La réponse (C) est fausse. La supervision constitue un élément d'un Answer (C) is incorrect because supervision is an element of a
programme qualité. Les examens continus sont des évaluations internes quality program. Ongoing reviews are internal assessments that
comprenant la supervision de la mission. include engagement supervision.
La réponse (D) est fausse. L'évaluation externe constitue un élément d'un Answer (D) is incorrect because external assessment is an
programme qualité. element of a quality program.

Restitution

© IFACI – CIA1 V.5.1 page 15/73


Acquérir une certification
> Préparation au CIA, partie 1
P1 - 26 -- Un responsable de l’audit interne examine la liste ci-après des P1 - 26 -- A chief audit executive is reviewing the following
risques de l’entreprise : enterprise-wide risk map:

PROBABILITÉ LIKELIHOOD
Lointaine Possible Probable Remote Possible Likely
IMPACT Critique Risque A Risque B IMPACT Critical Risk A Risk B
Majeur Risque D Major Risk D
Mineur Risque C Minor Risk C

Compte tenu des moyens limités dont dispose l’audit interne, quelle est la Which of the following is the correct prioritization of risks,
hiérarchie correcte des risques ? considering limited resources in the internal audit activity?
A. Risque B, risque C, risque A, risque D. A. Risk B, Risk C, Risk A, Risk D.
B. Risque A, risque B, risque C, risque D. B. Risk A, Risk B, Risk C, Risk D.
C. Risque D, risque B, risque A, risque C. C. Risk D, Risk B, Risk A, Risk C.
D. Risque B, risque C, risque D, risque A. D. Risk B, Risk C, Risk D, Risk A.
La réponse (A) est fausse. Le risque D serait prioritaire par rapport au risque A, en Answer (A) is incorrect. Risk D would take precedence over risk A,
raison de sa plus forte probabilité de survenue et malgré son moindre impact. as it has a higher probability of occurring despite the lower impact.
La réponse (B) est fausse. Ce n'est pas le bon ordre. Answer (B) is incorrect. This is not the correct order.
La réponse (C) est juste. Elle classifie les risques en combinant Answer (C) is correct. This order ranks the risk by a
probabilité et impact. combination of probability and impact.
La réponse (D) est fausse. Le risque D devrait être prioritaire par rapport au Answer (D) is incorrect. Risk D should be rated higher than risk C,
risque C, en raison de sa probabilité et de son impact supérieurs. due to probability and impact.

[Enoncé #20] Pendant la phase de planification, un responsable de l’audit [Fact Pattern #20] During the planning phase, a chief audit
interne évalue quatre missions d’après les facteurs suivants : la capacité à executive (CAE) is evaluating four audit engagements based on
atténuer le risque pour l’organisation, l’aptitude à préserver les ressources the following factors: the engagement’s ability to reduce risk to the
financières de l’organisation et l’ampleur des changements mis en œuvre organization, the engagement’s ability to save the organization
depuis la dernière mission. Pour chaque mission, le responsable de l’audit money, and the extent of change in the area since the last
interne assigne de 1 à 3 points par aspect examiné, et calcule un score engagement. The CAE has scored the engagements for each
global. Voici les résultats obtenus (nombre de points entre parenthèses) : factor from low to high, assigned points, and calculated an overall
ranking. The results are shown below with the points in
Atténuation du parenthesis:
Mission Économies Changements
risque
1 Forte (3) Moyennes (2) Peu nombreux (1) Audit Risk Reduction Cost Savings Changes
2 Forte (3) Faibles (1) Nombreux (3) 1 High (3) Medium (2) Low (1)
3 Faible (1) Fortes (3) Moyens (2) 2 High (3) Low (1) High (3)
4 Moyenne (2) Moyennes (2) Nombreux (3) 3 Low (1) High (3) Medium (2)
4 Medium (2) Medium (2) High (3)
P1 - 27 -- (Voir énoncé 20) Si l'organisation lui a demandé de considérer que
les économies sont deux fois plus importantes que tout autre facteur, quelles P1 - 27 -- (Refers to Fact Pattern #20) If the organization has asked
missions le responsable de l'audit interne doit-il continuer à mettre en œuvre? the CAE to consider the cost savings factor to be twice as
important as any other factor, which engagements should the CAE
A. 1 et 2 uniquement. pursue?
B. 1 et 3 uniquement.
C. 2 et 4 uniquement. A. 1 and 2 only.
D. 3 et 4 uniquement. B. 1 and 3 only.
C. 2 and 4 only.
La réponse (A) est fausse. Ce choix correspond au score total le moins élevé. D. 3 and 4 only.
La réponse (B) est fausse. Le score total est inférieur à celui des missions 3 et 4. Answer (A) is incorrect. This choice involves the least total
La réponse (C) est fausse. Le score total est inférieur à celui des missions 3 et 4. points.
La réponse (D) est juste. Ce choix correspond au score total le plus Answer (B) is incorrect. The total points are less than for engagements
élevé, et les missions concernées ont une capacité moyenne ou élevée 3 and 4.
à permettre des économies. Answer (C) is incorrect. The total points are less than for engagements
3 and 4.
Answer (D) is correct. This has the highest total points, and
the engagements have medium and high potentials for cost
savings.

Restitution

© IFACI – CIA1 V.5.1 page 16/73


Acquérir une certification
> Préparation au CIA, partie 1
P1 - 28 -- Pour planifier les missions, le responsable de l'audit interne est le P1 - 28 -- A chief audit executive would most likely use risk
plus susceptible de recourir à une évaluation des risques, car celle-ci : assessment for audit planning because it provides:
A. Constitue un processus systématique d'évaluation et de prise en compte A. A systematic process for assessing and integrating professional
d'un jugement professionnel sur des facteurs délétères probables. judgment about probable adverse conditions.
B. Établit une liste des effets potentiellement délétères pour l'organisation. B. A listing of potentially adverse effects on the organization.
C. Établit une liste des activités de l'organisation susceptibles d'être auditées. C. A list of auditable activities in the organization.
D. Indique la probabilité qu'un événement ou une action ait des effets D The probability that an event or action may adversely affect the
délétères sur l'organisation. organization.
La réponse (A) est juste. Cette raison est appropriée. Answer (A) is correct. This is an appropriate rationale.
La réponse (B) est fausse. Une telle liste pourrait convaincre le responsable Answer (B) is incorrect. Such a listing might convince the chief
de l'audit interne de la nécessité d'une évaluation des risques, mais elle n'est audit executive of the need for risk assessment but is not provided
pas établie dans le cadre de cette évaluation. by the process.
La réponse (C) est fausse. Une telle liste est utilisée dans le processus Answer (C) is incorrect. This is used in the risk assessment
d'évaluation des risques, mais ne constitue pas la raison pour laquelle on process but is not the rationale for using risk assessment.
recourt à une évaluation des risques. Answer (D) is incorrect. This is one definition of risk.
La réponse (D) est fausse. C'est une définition du risque.
P1 - 29 -- Lorsqu'un auditeur interne évalue les risques liés à une activité, il doit : P1 - 29 -- When assessing the risk associated with an activity, an
internal auditor should:
A. Déterminer le meilleur mode de gestion des risques.
B. Donner l'assurance que les risques sont gérés. A. Determine how the risk should best be managed.
C. Adapter le processus de gestion des risques aux expositions. B. Provide assurance on the management of the risk.
D. Élaborer des contrôles destinés à atténuer les risques identifiés. C. Update the risk management process based on risk exposures.
D. Design controls to mitigate the identified risks.
La réponse (A) est fausse. C'est à la direction de déterminer comment gérer
un risque inacceptable. Answer (A) is incorrect. Determining how unacceptable risk should
La réponse (B) est juste. L'assurance consiste, pour l'auditeur interne, be managed is the role of management.
en une évaluation objective des activités de gestion des risques par la Answer (B) is correct. Assurance services involve the internal
direction et de leur efficacité. auditor's objective assessment of management's risk
La réponse (C) est fausse. C'est à la direction d'élaborer et d'adapter le management activities and the degree to which they are
processus de gestion des risques. effective.
La réponse (D) est fausse. Si l'auditeur interne élaborait lui-même les Answer (C) is incorrect. Designing and updating the risk
contrôles, cela altèrerait son indépendance. management process is the role of management.
Answer (D) is incorrect. Designing controls would impair the
internal auditor's independence.

P1- 30 -- Le travail de l'audit interne comprend l’évaluation des P1- 30 -- The work of the internal audit activity includes
systèmes de management des risques et la participation à son evaluating and contributing to the improvement of risk
amélioration. Le risque est : management systems. Risk is

I. L'impact négatif d'événements dont on est sûr qu'ils se produiront. I. The negative effect of events certain to occur
II. Mesuré en termes de conséquences. II. Measured in terms of consequences
III. Mesuré en termes de probabilité. III. Measured in terms of likelihood
A. I uniquement.
B. I et II uniquement. A. I only.
C. II et III uniquement. B. I and II only.
D. I, II et III. C. II and III only.
D. I, II, and III.
La réponse (A) est fausse. Le risque suppose une notion d'incertitude
et les événements n'auront pas nécessairement d'impact négatif. Answer (A) is incorrect because risk involves uncertainty, and
La réponse (B) est fausse. Le risque suppose une notion d'incertitude the effects of events are not necessarily negative.
et les événements n'auront pas nécessairement d'impact négatif. Answer (B) is incorrect because risk involves uncertainty, and
La réponse (C) est juste. L'audit interne doit aider l'organisation the effects of events are not necessarily negative.
en identifiant et en évaluant les risques significatifs et contribuer Answer (C) is correct. The IAA should assist the
à l'amélioration des systèmes de management des risques et de organization by identifying and evaluating significant
contrôle (CRIPP). Le risque est la possibilité que se produise un exposures to risk and contributing to the improvement of
événement qui aura un impact sur la réalisation des objectifs. Il risk management and control systems (IPPF). Risk is the
se mesure en termes de conséquences et de probabilité uncertainty of an event the occurrence of which could
(Glossaire). have an effect on the achievement of organizational
La réponse (D) est fausse. Le risque suppose une notion d'incertitude objectives. It is measured in terms of consequences and
et les événements n'auront pas nécessairement d'impact négatif. likelihood (Glossary).
Answer (D) is incorrect because risk involves uncertainty, and
the effects of events are not necessarily negative.

Restitution

© IFACI – CIA1 V.5.1 page 17/73


Acquérir une certification
> Préparation au CIA, partie 1
P1- 31 -- Que sont les politiques, procédures et activités faisant partie P1- 31 -- What are the policies, procedures, and activities that
d'un cadre de contrôle, conçues pour veiller à ce que les risques soient are part of a control framework, designed to ensure that risks
contenus dans les limites fixées ? are contained within the established risk tolerances?
A. Les processus de management des risques. A. Risk management process.
B. Les processus de contrôle interne. B. Internal control processes.
C. Les processus de contrôle. C. Control processes.
D. Les processus de gouvernement d'entreprise. D. Governance process.
La réponse (A) est fausse. Les processus de management des risques Answer (A) is incorrect because this is not the definition of a
ne répondent pas à cette définition. risk management process.
La réponse (B) est fausse. Les processus de contrôle interne ne Answer (B) is incorrect because this is not the definition of an
répondent pas à cette définition. internal control process.
La réponse (C) est juste. Les processus de contrôle regroupent Answer (C) is correct. Control processes are the policies,
les politiques, procédures et activités faisant partie d'un cadre de procedures, and activities that are part of a control
contrôle, conçues pour veiller à ce que les risques soient framework, designed to ensure that risks are contained
contenus dans les limites fixées par le processus de management within the risk tolerances established by the risk
des risques. management process.
La réponse (D) est fausse. Les processus de gouvernement Answer (D) is incorrect because this is not the definition of a
d'entreprise ne répondent pas à cette définition. governance process.

P1-32 -- Il y a pertinence des processus de management des risques, P1- 32 -- Adequacy of risk management, control, and
de contrôle et de gouvernement d’entreprise si le management les a governance processes is present if management has planned
conçus et organisés de telle manière qu’ils apportent une assurance and designed these processes in a manner that provides
raisonnable que les objectifs et les buts fixés seront efficacement reasonable assurance that the organization's objectives and
économiquement atteints. Laquelle des assertions suivantes n’est pas goals will be achieved efficiently and economically. Which of
vraie au regard de l’atteinte efficace et économique des objectifs et the following statements is not true regarding the efficient
des buts de l’organisation? and economical achievement of the organization's objectives
and goals?
A. Un fonctionnement économique permet d’atteindre les objectifs et
buts avec une utilisation minimale de ressources quels que soient les A. Economical performance accomplishes objectives and
risques encourus. goals with minimal use of resources with no regard to risk
B. Un fonctionnement efficient signifie que les buts et les objectifs sont exposure.
atteints à bonne date. B. Efficient performance accomplishes objectives and goals
C. Un fonctionnement économique permet d’atteindre les objectifs et in a timely manner.
buts avec une utilisation minimale de ressources en proportion du C. Economical performance accomplishes objectives and
risque encouru. goals with minimal use of resources commensurate with the
D. Un fonctionnement efficient permet d’atteindre avec précision les risk exposure.
objectifs et les buts fixés au moindre coût. D. Efficient performance accomplishes objectives and goals
in an accurate and economical manner.

La réponse (A) est correcte. Un fonctionnement efficace permet Answer (A) is correct. Efficient performance
d’atteindre avec précision les objectifs et les buts fixés, à bonne accomplishes objectives and goals in an accurate,
date et au moindre coût. Un fonctionnement économique permet timely, and economical fashion. Economical performance
d’atteindre les objectifs et buts avec une utilisation minimale des accomplishes objectives and goals with minimal use of
ressources (c’est à dire à des coûts moindres) en proportion du resources (i.e., cost) commensurate with risk exposure
risque encouru. Pour un fonctionnement économique selon les (IPPF). Thus, in order to achieve economical
objectifs et buts fixés, la réduction des coûts doit être rapprochée performance when accomplishing objectives and goals,
du degré d’exposition au risque. the minimal cost should correspond to the degree of risk
La réponse (B) est fausse. Cette assertion est vraie au regard de exposure.
l’atteinte efficace et économique des objectifs et des buts de Answer (B) is incorrect because this is a true statement
l’organisation. regarding the efficient and economic achievement of the
La réponse (C) est fausse. Cette assertion est vraie au regard de organization's objectives and goals.
l’atteinte efficace et économique des objectifs et des buts de Answer (C) is incorrect because this is a true statement
l’organisation. regarding the efficient and economic achievement of the
La réponse (D) est fauss. Cette assertion est vraie au regard de organization's objectives and goals.
l’atteinte efficace et économique des objectifs et des buts de Answer (D) is incorrect because this is a true statement
l’organisation. regarding the efficient and economic achievement of the
organization's objectives and goals.

Restitution

© IFACI – CIA1 V.5.1 page 18/73


Acquérir une certification
> Préparation au CIA, partie 1
P1-33 – Lequel des éléments suivants ne fait pas partie des 7 P1- 33 -- Which of the following is not one of the seven
composantes du risque ? elements of risk?
A. La fréquence. A. Frequency.
B. Le délai. B. Timing.
C. L’analyse du retour sur investissement (ROI). C. ROI analysis.
D. L’incertitude. D. Uncertainty.
La réponse (A) est fausse car il s’agit d’une composante du risque. Answer (A) is incorrect because this is a valid element of risk.
La réponse (B) est juste. Les 7 composantes du risque sont les Answer (B) is correct. The seven elements of risk include
évènements menaçants, l’impact financier, la fréquence, threat events, single loss exposure value, frequency,
l’incertitude, les dispositifs de protection et de contrôle, le coût uncertainty, safeguards and controls, safeguard and
des dispositifs de protection et de contrôle, le rapport control costs, and cost/benefit or ROI analysis. Timing,
coût/avantages ou analyse de la rentabilité de l’investissment. Le which is not a risk element, is when a risk may occur. It
délai ne fait pas partie des éléments constitutifs du risque. Il is different from frequency, which is how often an
intervient lors de la survenance du rique. Il diffère de la fréquence uncertain event might occur.
qui évalue le nombre de fois qu’un évènement incertain est Answer (C) is incorrect because this is a valid element of risk.
suceptible de se produire. Answer (D) is incorrect because this is a valid element of risk.
La réponse (C) est fausse car il s’agit d’une composante du risque.
La réponse (D) est fausse car il s’agit d’une composante du risque.

P1- 34 – Lequel (lesquels) des éléments suivants constitue(nt) des P1- 34 -- Which of the following are elements of risk
éléments relatifs à la gestion et à la réduction des risques ? management and mitigation?
I. Les analyses coût / bénéfice des menaces. I. Threat events and cost/benefit analysis.
II. Les protections, les contrôles, et les analyses du retour sur II. Safeguards, controls, and ROI analysis.
investissement. III. Frequency and uncertainty.
III. La fréquence et l’incertitude. IV. Safeguard and control costs.
IV. Les coûts de la protection et du contrôle.
A. I only.
A. I uniquement. B. II only.
B. II uniquement. C. I and III only.
C. I et III uniquement. D. II and IV only.
D. II et IV uniquement.

La réponse (A) est fausse. Les menaces sont des éléments relatifs à Answer (A) is incorrect because threat events is a risk
l’identification et à l’évaluation des risques. element associated with risk identification and quantification.
La réponse (B) est fausse. Les coûts de la protection et du contrôle Answer (B) is incorrect because safeguard and control costs--
(choix IV) constituent également des éléments relatifs à la gestion et à choice IV--is also a risk element that relates to risk
la réduction des risques. management and mitigation.
La réponse (C) est fausse. Les menaces, la fréquence et l’incertitude Answer (C) is incorrect because threat events, frequency,
sont tous des éléments relatifs à l’identification et à l’évaluation des and uncertainty are all risk elements associated with risk
risques. identification and quantification.
La réponse (D) est juste. Les éléments relatifs à la gestion et à la Answer (D) is correct. The elements relating to risk
réduction des risques comprennent les protections et les management and mitigation include safeguards and
contrôles, les coûts de la protection et du contrôle, et les controls, safeguard and control costs, and cost/benefit
analyses coût / bénéfice et retour sur investissement. Les autres or ROI analysis. The other elements of risk-threat events,
éléments liés aux menaces, l’impact, la fréquence et l’incertitude single loss exposure value, frequency, and uncertainty-
font partie de l’identification et à l’évaluation des risques. comprise risk identification and quantification.

Restitution

© IFACI – CIA1 V.5.1 page 19/73


Acquérir une certification
> Préparation au CIA, partie 1
P1- 35 -- Laquelle des questions suivantes est la mieux associée aux P1- 35 -- Which of the following questions is best associated
dispositifs de protection et de contrôle du risque ? with the safeguards and controls risk element?
A. Quels événements pourraient affecter la capacité de l'organisation à A. What could happen that would adversely affect the
atteindre ses objectifs et à mettre en oeuvre ses stratégies ? organization's ability to achieve its objectives and execute its
B. Comment peut-on prévenir, éviter, atténuer et détecter les risques strategies?
et communiquer sur cet aspect ? B. What can be done to prevent and avoid, mitigate, and
C. Quelle est l'incidence financière potentielle d'un événement dont la detect risks and provide notification?
probabilité est incertaine ? C. What is the potential financial impact of the occurrence of
D. À quelle fréquence un événement dont la probabilité est incertaine an uncertain event?
est-il susceptible de se produire ? D. How often might an uncertain event occur?
La réponse (A) est fausse. C'est une question clé associée à la
composante du risque constituée par les événements menaçants. Answer (A) is incorrect because it is a key question
La réponse (B) est juste. Pour l'encadrement, il existe sept associated with the threat events element of risk.
questions clés pouvant servir à identifier les risques de Answer (B) is correct. For the purposes of management,
l'organisation et cibler les moyens potentiels destinés à les there are seven key questions that can serve to identify
maîtriser ou à les atténuer. Outre les composantes du risque qui y organizational risk and target potential ways to control or
sont associées (entre crochets), ces questions sont notamment mitigate the exposures. These questions, along with the
les suivantes : Quel événement pourrait affecter la capacité de risk elements associated with them (in brackets), include
l'organisation à atteindre ses objectifs et à mettre en oeuvre ses the following: What could happen that would adversely
stratégies ? [événements menaçants]. Si un tel événement affect the organization's ability to achieve its objectives
survient, quelle serait son incidence financière potentielle ? and execute its strategies? [Threat Events] If it happens,
[impact financier]. À quelle fréquence est-il susceptible de se what is the potential financial impact? [Single Loss
produire ? [fréquence]. Quelle est la probabilité des réponses aux Exposure Value] How often might it happen? [Frequency]
trois premières questions ? [incertitude]. Que peut-on faire pour How probable are the answers to the first three
prévenir, éviter, atténuer et détecter les risques et à communiquer questions? [Uncertainty] What can be done to prevent
sur cet aspect ? [dispositifs de protection et de contrôle]. Quel en and avoid, mitigate, and detect risks and provide
sera le coût ? [coût des dispositifs de protection et de contrôle]. notification? [Safeguards and Controls] How much will it
Quelle serait l'efficience de ces mesures ? [rapport cost? [Safeguard and Control Costs] How efficient would
coût/avantages ou analyse de la rentabilité de l'investissement]. that be? [Cost/Benefit or ROI Analysis]
La réponse (C) est fausse. C'est une question clé associée à la Answer (C) is incorrect because it is a key question
composante du risque constitué par l'impact financier. associated with the single loss exposure value element of
La réponse (D) est fausse. C'est une question clé associée à la risk.
composante du risque constitué par la fréquence d'un événement. Answer (D) is incorrect because it is a key question
associated with the frequency element of risk.

Restitution

© IFACI – CIA1 V.5.1 page 20/73


Acquérir une certification
> Préparation au CIA, partie 1
P1- 36 -- Quelles sont les problématiques particulièrement critiques P1- 36 -- Which of the following are critical risk and control
liées aux risques et aux contrôles que doit examiner l'auditeur interne ? issues that an internal auditor must address?
I. Les progrès technologiques rapides.
II. Le maintien de l'intégrité des transactions. I. Rapid technology changes.
III. L'examen et la validation du contenu du site Internet. II. Maintenance of transaction integrity.
IV. Les modifications apportées aux structures organisationnelles. III. Website content review and approval.
IV. Changes to organizational structures.
A. I et II uniquement.
B. I et III uniquement. A. I and II only.
C. II et III uniquement. B. I and III only.
D. I, II, III et IV. C. II and III only.
D. I, II, III, and IV.
La réponse (A) est fausse, car tous les points cités sont des Answer (A) is incorrect because all of the items are critical
problématiques critiques liées aux risques et aux contrôles que doit risk and control issues that an internal auditor must address.
examiner l'auditeur interne. Answer (B) is incorrect because all of the items are critical
La réponse (B) est fausse, car tous les points cités sont des risk and control issues that an internal auditor must address.
problématiques critiques liées aux risques et aux contrôles que doit Answer (C) is incorrect because all of the items are critical
examiner l'auditeur interne. risk and control issues that an internal auditor must address.
La réponse (C) est fausse, car tous les points cités sont des Answer (D) is correct. Some of the more critical risk and
problématiques critiques liées aux risques et aux contrôles que doit control issues to be addressed by the internal auditor
examiner l'auditeur interne. are: General project management risks. Specific security
La réponse (D) est juste. Parmi les problématiques liées aux threats, such as denial of service, physical attacks,
risques et aux contrôles que doit examiner l'auditeur interne, les viruses, identity theft, and unauthorized access or
points suivants sont particulièrement critiques : risques généraux disclosure of data. Maintenance of transaction integrity
afférents à la gestion de projets ; menaces spécifiques liées à la under a complex network of links to legacy systems and
sécurité, notamment déni de service, attaques physiques, virus, data warehouses. Website content review and approval
usurpation d'identité, accès non autorisé ou divulgation de when there are frequent changes and sophisticated
données ; maintien de l'intégrité des transactions dans le cadre customer features and capabilities that offer around-the-
d'un réseau complexe de liens avec les systèmes précédents et clock service. Rapid technology changes. Legal issues,
avec les entrepôts de données (data warehouses) ; examen et such as increasing regulations throughout the world to
approbation du contenu du site Internet en cas de modifications protect individual privacy; enforceability of contracts
fréquentes et de fonctionnalités clients élaborées et offrant un outside of the organization's country; and tax and
service 24 heures sur 24 ; évolutions technologiques rapides ; accounting issues. Changes to surrounding business
aspects juridiques, notamment augmentation à travers le monde processes and organizational structures.
du nombre de réglementations visant à protéger la vie privée ;
caractère exécutoire des contrats à l'étranger ; problématiques
d'ordre fiscal et comptable ; modifications apportées aux
processus environnants et aux structures organisationnelles.

Restitution

© IFACI – CIA1 V.5.1 page 21/73


Acquérir une certification
> Préparation au CIA, partie 1
P1- 37 -- Laquelle des propositions suivantes constitue un objectif clé P1- 37 -- Which of the following are key objectives of a risk
d'un processus de gestion des risques ? management process?
I. Les risques découlant des stratégies et des activités de l'organisation I. Risks arising from business strategies and activities are
sont identifiés et hiérarchisés. identified and prioritized.
II. Un suivi permanent des activités est effectué afin de réévaluer II. Ongoing monitoring activities are conducted to periodically
périodiquement les risques et l'efficacité des contrôles permettant de reassess risk and the effectiveness of controls to manage
les gérer. risk.
III. L'examen des précédents rapports d'évaluation des risques par la III. Review of previous risk evaluation reports by
direction, les auditeurs internes, les auditeurs externes et toutes les management, internal auditors, external auditors, and any
autres sources qui peuvent avoir publié ce type de rapports. other sources that may have issued such reports.
A. I et II uniquement. A. I and II only.
B. I et III uniquement. B. I and III only.
C. II et III uniquement. C. II and III only.
D. I, II et III. D. I, II, and III.

La réponse (A) est juste. Pour formuler une opinion globale sur Answer (A) is correct. Internal auditors must determine
l'adéquation du dispositif de management des risques, les that the organization's risk management processes
auditeurs internes doivent également s'assurer que le processus address five key objectives to formulate an opinion on
de management des risques de l'organisation répond à cinq the overall adequacy of the risk management processes.
objectifs principaux. Les cinq principaux objectifs d'un dispositif The five key objectives of a risk management process
de management des risques sont les suivants : les risques are: Risks arising from business strategies and activities
découlant des stratégies et des activités de l'organisation sont are identified and prioritized. Management and the board
identifiés et hiérarchisés ; le management et le Conseil ont have determined the level of risks acceptable to the
déterminé un niveau de risques acceptable pour l'organisation, en organization, including the acceptance of risks designed
tenant compte des risques liés à la mise en oeuvre des plans to accomplish the organization's strategic plans. Risk
stratégiques de l'organisation ; des mesures d'atténuation des mitigation activities are designed and implemented to
risques sont définies et mises en place afin de réduire ou de gérer reduce, or otherwise manage, risk at levels that were
les risques, compte tenu des seuils jugés acceptables par le determined to be acceptable to management and the
management et le Conseil ; un suivi permanent des activités est board. Ongoing monitoring activities are conducted to
effectué afin de réévaluer périodiquement les risques et periodically reassess risk and the effectiveness of
l'efficacité des contrôles permettant de les gérer ; des rapports controls to manage risk. The board and management
concernant les résultats des processus de management des receive periodic reports of the results of the risk
risques sont adressés périodiquement au Conseil et au management processes. The corporate governance
management. Le processus de gouvernement d'entreprise doit processes of the organization should provide periodic
fournir une présentation périodique des risques, des stratégies communication of risks, risk strategies, and controls to
liées aux risques, et des contrôles, destinée aux parties stakeholders.
prenantes de l'organisation. Answer (B) is incorrect because review of previous risk
La réponse (B) est fausse. L'examen des précédents rapports evaluation reports by management, internal auditors, external
d'évaluation des risques par la direction, les auditeurs internes, les auditors, and any other sources that may have issued such
auditeurs externes et toutes autres sources qui peuvent avoir publié ce reports is an engagement procedure, not a key objective, that
type de rapports constitue une procédure de mission, et non un objectif the internal auditor should consider when gathering sufficient
clé, que l'auditeur interne doit prendre en compte lorsqu'il collecte information regarding the five key objectives of the risk
suffisamment d'informations sur les cinq objectifs clés du processus de management process.
gestion des risques. Answer (C) is incorrect because review of previous risk
La réponse (C) est fausse. L'examen des précédents rapports evaluation reports by management, internal auditors, external
d'évaluation des risques par la direction, les auditeurs internes, les auditors, and any other sources that may have issued such
auditeurs externes et toutes autres sources qui peuvent avoir publié ce reports is an engagement procedure, not a key objective, that
type de rapports constitue une procédure de mission, et non un objectif the internal auditor should consider when gathering sufficient
clé, que l'auditeur interne doit prendre en compte lorsqu'il collecte information regarding the five key objectives of the risk
suffisamment d'informations sur les cinq objectifs clés du processus de management process.
gestion des risques. Answer (D) is incorrect because review of previous risk
La réponse (D) est fausse. L'examen des précédents rapports evaluation reports by management, internal auditors, external
d'évaluation des risques par la direction, les auditeurs internes, les auditors, and any other sources that may have issued such
auditeurs externes et toutes autres sources qui peuvent avoir publié ce reports is an engagement procedure, not a key objective, that
type de rapports constitue une procédure de mission, et non un objectif the internal auditor should consider when gathering sufficient
clé, que l'auditeur interne doit prendre en compte lorsqu'il collecte information regarding the five key objectives of the risk
suffisamment d'informations sur les cinq objectifs clés du processus de management process.
gestion des risques.

Restitution

© IFACI – CIA1 V.5.1 page 22/73


Acquérir une certification
> Préparation au CIA, partie 1
P1- 38 -- Laquelle des propositions suivantes définit le mieux le P1- 38 -- Which of the following best defines control?
contrôle ?
A. Control is the result of proper planning, organizing, and
A. Le contrôle est le résultat de la mise en œuvre de mesures directing by management.
planifiées, organisées et orientées par l'encadrement. B. Controls are statements of what the organization chooses
B. Les contrôles sont l'énoncé de ce que l'organisation choisit to accomplish.
d'accomplir. C. Control is provided when cost-effective measures are
C. Le contrôle est en place lorsque des mesures présentant un bon taken to restrict deviations to a tolerable level.
rapport coût-efficacité sont prises afin de limiter les écarts à un niveau D. Control accomplishes objectives and goals in an accurate,
tolérable. timely, and economical fashion.
D. Le contrôle permet d'atteindre les objectifs et les buts de manière
précise, rapide et économique.
La réponse (A) est juste. D'après le Glossaire annexé aux Normes, Answer (A) is correct. According to the Glossary
un contrôle est « toute mesure prise par le management, le appended to the Standards, a control is "any action taken
Conseil et d'autres parties afin de gérer les risques et d'accroître by management, the board, and other parties to enhance
la probabilité que les buts et objectifs fixés seront atteints. Les risk management and increase the likelihood that
managers planifient, organisent et dirigent la mise en œuvre de established objectives and goals will be achieved.
mesures suffisantes pour donner une assurance raisonnable que Management plans, organizes, and directs the
les buts et objectifs seront atteints ». Ainsi, le contrôle est le performance of sufficient actions to provide reasonable
résultat d'une planification, d'une organisation et d'une direction assurance that objectives and goals will be achieved."
adéquates de la part de l'encadrement. Thus, control is the result of proper planning, organizing,
La réponse (B) est fausse. Les objectifs et buts établis sont ce que and directing by management.
l'organisation choisit d'accomplir. Answer (B) is incorrect because established objectives and
La réponse (C) est fausse. Lors de l'élaboration de processus de goals are what the organization chooses to accomplish.
gestion des risques, de contrôle et de gouvernement d'entreprise, « Answer (C) is incorrect because, during the development of
une assurance raisonnable est apportée dès lors que les mesures les risk management, control, and governance processes,
plus efficaces et économiques possible sont prises dès la conception reasonable assurance of achieving objectives and goals
et la mise en place des processus pour réduire les risques et limiter les efficiently and economically is provided when the most cost-
écarts attendus à un niveau tolérable » (CRIPP). effective measures are taken in the design and
La réponse (D) est fausse, car « un fonctionnement efficace permet implementation stages to reduce risks and restrict deviations
d'atteindre avec précision les objectifs et les buts fixés, à bonne date et to a tolerable level (IPPF).
au moindre coût » (CRIPP). Answer (D) is incorrect because efficient performance
accomplishes objectives and goals in an accurate, timely, and
economical fashion (IPPF).
P1- 39 -- Le management a un rôle à jouer dans le contrôle. En fait, la P1- 39 -- Management has a role in the maintenance of
direction est parfois elle-même un contrôle. Laquelle des propositions control. In fact, management sometimes is a control. Which of
suivantes concerne des fonctions managériales représentant un the following involves managerial functions as a control?
contrôle ?
A. Monitoring performance.
A. Le suivi des performances. B. Use of an organizational policies manual.
B. L'utilisation d'un manuel des politiques organisationnelles. C. Maintenance of a quality assurance program.
C. L'application d'un programme d'assurance qualité. D. Establishment of an internal audit activity.
D. La mise en place d'un service d'audit interne.
La réponse (A) est juste. « Il y a efficacité des processus de Answer (A) is correct. Risk management, control, and
management des risques, de contrôle et de gouvernement governance processes are effective when management
d'entreprise si le management dirige ces processus de telle sorte directs processes in such a manner as to provide
qu'ils apportent une assurance raisonnable que les objectifs et reasonable assurance that the organization's objectives
les buts de l'organisation seront atteints. Diriger implique, en plus and goals will be achieved. In addition to accomplishing
de la réalisation des objectifs et des activités prévue, the objectives and planned activities, management
l'autorisation des transactions et des activités, la surveillance des directs by authorizing activities and transactions,
résultats obtenus et la vérification que les processus de monitoring resulting performance, and verifying that the
l'organisation fonctionnent comme prévu. » (CRIPP) organization's processes are operating as designed
La réponse (B) est fausse. Le manuel donne des conseils mais ne (IPPF).
constitue pas un moyen de contrôle. Answer (B) is incorrect because the manual advises but does
La réponse (C) est fausse. Un programme d'assurance qualité est une not control.
forme d'évaluation interne. Le responsable du programme doit être Answer (C) is incorrect because a quality assurance program
indépendant par rapport aux opérations évaluées. is a form of internal assessment. The manager of the
La réponse (D) est fausse. L'audit interne doit être indépendant par program should be independent of the operations assessed.
rapport aux opérations évaluées et ne constitue pas une fonction Answer (D) is incorrect because an internal audit activity
managériale. should be independent of the operations reviewed and is not
a managerial function.

Restitution

© IFACI – CIA1 V.5.1 page 23/73


Acquérir une certification
> Préparation au CIA, partie 1
P1- 40 -- Les contrôles peuvent être classés d'après la fonction qu'ils P1- 40 -- Controls may be classified according to the function
doivent remplir : ils peuvent être détectifs, préventifs ou directifs. they are intended to perform, for example, as detective,
Lequel des contrôles suivants est directif ? preventive, or directive. Which of the following is a directive
control?
A. Des rapprochements mensuels des relevés bancaires. A. Monthly bank statement reconciliations.
B. La double signature pour tous les décaissements dépassant un B. Dual signatures on all disbursements over a specific
certain montant. amount.
C. L'enregistrement de toutes les transactions le jour même où elles C. Recording every transaction on the day it occurs.
sont effectuées. D. Requiring all members of the internal audit activity to be
D. L'exigence que tous les membres de l'audit interne soient des CIAs.
auditeurs certifiés.
La réponse (A) est fausse. Le rapprochement mensuel des relevés Answer (A) is incorrect because monthly bank statement
bancaires constitue un contrôle détectif. Les événements observés se reconciliation is a detective control. The events under scrutiny
sont déjà produits. have already occurred.
La réponse (B) est fausse. La double signature pour tous les Answer (B) is incorrect because dual signatures on all
décaissements dépassant un certain montant constitue un contrôle disbursements over a specific amount is a preventive control.
préventif. Ce type de contrôle vise à prévenir la survenance d'un The control is designed to deter an undesirable event.
événement non souhaité. Answer (C) is incorrect because recording every transaction
La réponse (C) est fausse. L'enregistrement de toutes les transactions on the day it occurs is a preventive control. The control is
le jour même où elles sont effectuées constitue un contrôle préventif. designed to deter an undesirable event.
Ce type de contrôle vise à prévenir la survenance de tout événement Answer (D) is correct. Requiring all members of the
non souhaité. internal audit activity to be CIAs is a directive control.
La réponse (D) est juste. Exiger que tous les membres de l'audit The control is designed to cause or encourage a
interne soient des auditeurs certifiés constitue un contrôle desirable event to occur (IPPF). The requirement
directif. Ce type de contrôle a pour but de provoquer ou enhances the professionalism and level of expertise of
d'encourager la survenance d'un événement souhaité (CRIPP). the internal audit activity.
Cette exigence renforce le professionnalisme et le niveau
d'expertise de l'audit interne.

P1- 41 -- Lesquels des éléments suivants font partie de P1- 41 -- Which of the following are elements included in the
l'environnement de contrôle ? control environment?
A. La structure organisationnelle, la philosophie de direction et la A. Organizational structure, management philosophy, and
planification. planning.
B. L'intégrité et les valeurs éthiques, l'attribution des pouvoirs, les B. Integrity and ethical values, assignment of authority, and
politiques relatives aux ressources humaines. human resource policies.
C. La compétence du personnel, les installations de sauvegarde, le C. Competence of personnel, backup facilities, laws, and
droit et la réglementation. regulations.
D. L'évaluation des risques, l'attribution des responsabilités et les D. Risk assessment, assignment of responsibility, and human
pratiques relatives aux ressources humaines. resource practices.
La réponse (A) est fausse. La planification ne fait pas partie de Answer (A) is incorrect because planning is not an element of
l'environnement de contrôle. the control environment.
La réponse (B) est juste. D'après le Glossaire annexé aux Normes, Answer (B) is correct. According to the Glossary
l'environnement de contrôle inclut « l'attitude et les actions du appended to the Standards, the control environment
Conseil et du management au regard de l'importance du contrôle includes the attitude and actions of the board and
dans l'organisation. L'environnement de contrôle constitue le management regarding the significance of control within
cadre et la structure nécessaires à la réalisation des objectifs the organization. The control environment provides the
primordiaux du système de contrôle interne. L'environnement de discipline and structure for the achievement of the
contrôle englobe les éléments suivants : intégrité et valeurs primary objectives of the system of internal control. The
éthiques, philosophie et style de direction, structure control environment includes the following elements:
organisationnelle, attribution des pouvoirs et responsabilités, Integrity and ethical values Management's philosophy
politiques et pratiques relatives aux ressources humaines, and operating style Organizational structure Assignment
compétence du personnel ». of authority and responsibility Human resource policies
La réponse (C) est fausse. Les installations de sauvegarde, le droit et and practices Competence of personnel
la réglementation ne font pas partie de l'environnement de contrôle. Answer (C) is incorrect because backup facilities, laws, and
La réponse (D) est fausse. L'évaluation des risques fait partie de la regulations are not elements of the control environment.
planification de l'audit interne et des missions spécifiques. Answer (D) is incorrect because risk assessment is part of
planning the internal audit activity and specific engagements.

Restitution

© IFACI – CIA1 V.5.1 page 24/73


Acquérir une certification
> Préparation au CIA, partie 1
P1- 42 – Le contrôle interne a pour seul objet de fournir une assurance P1- 42 -- Internal control can provide only reasonable
raisonnable que les objectifs et buts de l’organisation seront atteints de assurance that the organization's objectives and goals will be
manière efficace et efficiente. Lequel des facteurs ci-dessous met efficiently and effectively. One factor limiting the
pourraient limiter la probabilité d’atteindre ces objectifs likelihood of achieving those objectives is that
A. La responsabilité première d’un auditeur interne est la détection A. The internal auditor's primary responsibility is the detection
des fraudes. of fraud.
B. Le comité d’audit est actif et indépendant. B. The audit committee is active and independent.
C. Le coût du contrôle interne ne doit pas excéder ses bénéfices. C. The cost of internal control should not exceed its benefits.
D. Le management surveille la performance. D. Management monitors performance.
La réponse (A) est fausse car la responsabilité d’un auditeur interne Answer (A) is incorrect because the internal auditor's
est avant tout d’examiner et d’évaluer la cohérence et l’efficacité du responsibility regarding internal control is to examine and
dispositif de contrôle interne. evaluate the adequacy and effectiveness of the system of
La réponse (B) est fausse car un comité d’audit actif renforce internal control.
l’environnement de contrôle. Answer (B) is incorrect because an active audit committee
La réponse (C) est correcte. Un facteur limitant est que le coût du strengthens the control environment.
contrôle interne ne devrait pas excéder les bénéfices attendus Answer (C) is correct. A limiting factor is that the cost of
suite à la mise en œuvre du dispositif. Ainsi, la perte potentielle internal control should not exceed the benefits that are
associée à toute exposition au risque est mise en balance avec le expected to be derived. Thus, the potential loss
coût de son contrôle. Bien que la relation coût / bénéfice associated with any exposure or risk is weighed against
constitue un critère primordial devant être pris en copte lors de la the cost to control it. Although the cost-benefit
mise en œuvre d’un dispositif de contrôle, une mesure précise relationship is a primary criterion that should be
des coûts et de bénéfice reste cependant difficile à réaliser. considered in designing and implementing internal
La réponse (D) est fausse car la fonction managériale donne control, the precise measurement of costs and benefits
l’autorisation de mener des activités et des transactions, de surveiller usually is not possible.
la performance et de vérifier que les processus fonctionnent comme Answer (D) is incorrect because management's directing
prévus. function includes authorizing activities and transactions,
monitoring resulting performance, and verifying that
processes are operating as designed.

P1- 43 -- Le contrôle peut être décrit comme un système fermé P1- 43 -- Control may be described as a closed system
comportant six éléments. L'un de ces éléments est : consisting of six elements. One of the six elements is
A. L'instauration d'objectifs de performance. A. Setting performance standards.
B. La sécurisation adéquate de fichiers de données. B. Adequately securing data files.
C. L'approbation de la charte d'audit interne. C. Approval of the internal audit activity's charter.
D. L'instauration d'un service d'audit interne. D. Establishment of an independent internal audit activity.
La réponse (A) est juste. D'après Internal Auditing de Sawyer (IIA Answer (A) is correct. Sawyer's Internal Auditing (IIA
1988, p. 979 de l'original), ouvrage sur lequel se fonde cette 1988, p. 979), on which this question is based, describes
question, le système de contrôle comporte six éléments : (1) la a six-element control system: (1) setting standards, (2)
définition d'objectifs de performance, (2) la mesure de la measuring performance, (3) analyzing performance and
performance, (3) l'analyse de la performance et la comparaison comparing it with the standards, (4) evaluating deviations
avec les objectifs, (4) l'évaluation des écarts, qui doivent être and bringing them to the attention of appropriate
portés à l'attention des personnes concernées, (5) la correction persons, (5) correcting deviations, and (6) following up
de ces écarts, et (6) le suivi des actions correctives. on corrective action.
La réponse (B) est fausse. La sécurisation adéquate de fichiers de Answer (B) is incorrect because adequately securing data
données ne constitue pas l'un des éléments d'un système de contrôle files is not an element of a closed control system.
fermé. Answer (C) is incorrect because approval of the IAA's charter
La réponse (C) est fausse, l'approbation de la charte d'audit interne ne is not an element of a closed control system.
constitue pas l'un des éléments d'un système de contrôle fermé. Answer (D) is incorrect because establishment of an
La réponse (D) est fausse. L'établissement d'un service d'audit interne independent IAA is not an element of a closed control
ne constitue pas l'un des éléments d'un système de contrôle fermé. system.

Restitution

© IFACI – CIA1 V.5.1 page 25/73


Acquérir une certification
> Préparation au CIA, partie 1
P1- 44 -- Lequel des éléments suivants décrit une défaillance du P1- 44 -- Which of the following describes a control
contrôle ? weakness?

A. Les procédures d'achat sont bien conçues et respectées, sauf A. Purchasing procedures are well designed and are followed
lorsque le superviseur des achats exige qu'il en soit autrement. unless otherwise directed by the purchasing supervisor.
B. Des bons de commande vierges prénumérotés sont rangés en lieu B. Prenumbered blank purchase orders are secured within
sûr dans le service achats. the purchasing department.
C. Les achats opérationnels courants varient de 500 à 1 000 dollars, C. Normal operational purchases fall in the range from $500
deux signatures étant nécessaires pour les achats supérieurs à 1 000 to $1,000 with two signatures required for purchases over
dollars. $1,000.
D. L'acheteur investit dans une SICAV négociée sur le marché qui D. The purchasing agent invests in a publicly traded mutual
intègre le titre de l'un des fournisseurs de l'organisation dans son fund that lists the stock of one of the organization's suppliers
portefeuille. in its portfolio.
La réponse (A) est juste. Des procédures bien conçues qui sont Answer (A) is correct. Well-designed procedures that are
laissées de côté au gré du personnel dirigeant ne caractérisent set aside at management's discretion are not adequate
pas des contrôles adéquats. Pour être efficaces, les procédures controls. Control procedures must be followed
de contrôle doivent être respectées de manière systématique. consistently to be effective. However, the possibility of
Cependant, la possibilité pour la direction de passer outre management override is an inherent limitation of internal
constitue une limitation inhérente au contrôle interne. control.
La réponse (B) est fausse. L'utilisation de bons de commande vierges Answer (B) is incorrect because use of prenumbered blank
prénumérotés rangés en lieu sûr dans le service achats constitue un purchase orders secured within the purchasing department is
contrôle ordinaire. a common control.
La réponse (C) est fausse. Le fait d'exiger une procédure d'autorisation Answer (C) is incorrect because requiring a more stringent
plus stricte pour des achats plus importants constitue un contrôle authorization procedure for larger purchases is an
approprié tant que des pièces justificatives sont fournies à l'appui de appropriate control as long as documentation supports the
ces achats. purchases.
La réponse (D) est fausse. La SICAV retenue par l'acheteur ne doit Answer (D) is incorrect because the purchasing agent's
pas constituer un conflit d'intérêts. Or, la relation entre le retour sur mutual fund investment should not be a conflict of interest.
investissement et d'éventuelles mesures qui pourraient être prises par The relationship between the return on the investment and
l'acheteur pour favoriser le fournisseur est très ténue. any possible action by the agent to favor the supplier is very
weak.

P1- 45 -- L’auditeur interne sait que tout dispositif de contrôle interne a P1- 45 -- The internal auditor recognizes that certain
ses limites. Lequel des scénarios suivants est le résultat d’une limite limitations are inherent in any system of internal controls.
inhérente au dispositif de contrôle interne ? Which one of the following scenarios is the result of an
inherent limitation of internal control?
A. Le contrôleur financier réalise à la fois les dépôts d’argent liquide et A. The comptroller both makes and records cash deposits.
leur enregistrement. B. A security guard allows one of the warehouse employees
B. Un agent de sécurité autorise un des employés de l’entrepôt à sortir to remove assets from the premises without authorization.
des biens des locaux sans autorisation. C. The organization sells to customers on account, without
C. L’organisation vend à crédit à ses clients sans approbation du credit approval.
crédit. D. An employee, who is unable to read, is assigned custody
D. Un employé, qui ne sait pas lire, est affecté à la surveillance de la of the organization's computer tape library and run manuals
bandothèque de l’organisation et déroule des procédures qui sont that are used during the third shift.
utilisées au cours de la vacation de nuit.
La réponse (A) est fausse. La séparation de fonctions entre Answer (A) is incorrect because segregating the functions of
l’enregistrement et la conservation des biens est habituelle. Le fait que recording and asset custody is customary. That the
le contrôleur financier réalise à la fois les dépôts d’argent liquide et leur comptroller both makes and records cash deposits is an
enregistrement est une faiblesse de contrôle qui aurait pu être évitée. avoidable control weakness.
La réponse (B) est juste. Les limites inhérentes au contrôle Answer (B) is correct. Inherent limitations in internal
interne surviennent en cas d’erreurs de jugement, control arise from mistakes in judgment,
d’incompréhension des instructions, d’un manque d’attention du misunderstandings of instructions, personnel
personnel, de distraction, de fatigue, de collusion, d’actes en carelessness, distraction, fatigue, collusion,
provenance de la direction, de modifications du contexte, et de perpetrations by management, changing conditions, and
détérioration des niveaux de conformité. En conséquence, un deterioration of degrees of compliance. Thus, a control
contrôle (le recours à des agents de sécurité) basé sur la (use of security guards) based on segregation of
séparation de fonctions peut être contourné par la collusion entre functions may be overcome by collusion among two or
deux employés ou plus. more employees.
La réponse (C) est fausse. Les transactions peuvent et devraient être Answer (C) is incorrect because transactions can and should
autorisées avant leur exécution. be authorized before execution.
La réponse (D) est fausse. L’affectation de personnel non qualifié est Answer (D) is incorrect because assignment of an unqualified
une faiblesse de contrôle qui aurait pu être évitée. employee is an avoidable control weakness.

Restitution

© IFACI – CIA1 V.5.1 page 26/73


Acquérir une certification
> Préparation au CIA, partie 1
P1- 46 -- Lequel des exemples suivants représente un contrôle a P1- 46 -- Which of the following is an example of a feedback
posteriori ? control?
A. La maintenance préventive. A. Preventive maintenance.
B. L'inspection des produits finis. B. Inspection of completed goods.
C. Une supervision étroite des ouvriers de production. C. Close supervision of production-line workers.
D. La mesure des performances par rapport à une norme. D. Measuring performance against a standard.
La réponse (A) est fausse. La maintenance préventive fait partie des Answer (A) is incorrect because preventive maintenance is a
contrôles a priori. Elle cherche à anticiper et prévenir les problèmes. feedforward control. It attempts to anticipate and prevent
La réponse (B) est juste. Les contrôles a posteriori donnent des problems.
informations sur les activités achevées. Ils permettent Answer (B) is correct. Feedback controls obtain
l'amélioration des performances à venir grâce aux leçons tirées information about completed activities. They permit
des erreurs passées. Ainsi, l'action correctrice intervient après- improvement in future performance by learning from past
coup. L'inspection des produits finis constitue un exemple de mistakes. Thus, corrective action occurs after the fact.
contrôle a posteriori. Inspection of completed goods is an example of a
La réponse (C) est fausse. La supervision étroite des ouvriers de feedback control.
production est un contrôle concurrent. Il ajuste un processus en cours. Answer (C) is incorrect because the close supervision of
La réponse (D) est fausse. La mesure des performances par rapport à production-line workers is a concurrent control. It adjusts an
une norme constitue un aspect général du contrôle. ongoing process.
Answer (D) is incorrect because measuring performance
against a standard is a general aspect of control.

P1- 47 -- Un processus de contrôle type comporte les étapes P1- 47 -- The steps in a typical control process include:
suivantes:
1. Selecting strategic control points at which to gather
1. Sélection des points de contrôle stratégiques au niveau desquels information about activities being performed
seront recueillies des informations sur les activités en cours. 2. Accumulating, classifying, and recording data samples
2. Accumulation, classification et enregistrement d'échantillons de 3. Observing the work or collecting samples of data
données. 4. Determining whether performance is satisfactory
3. Observation du travail ou collecte d'échantillons de données. 5. Reviewing and revising standards
4. Vérification du degré satisfaisant des performances. 6. Reporting significant deviations to managers concerned
5. Examen et révision des normes.
6. Notification des écarts significatifs aux cadres concernés.
Dans quel ordre ces étapes doivent-elles se succéder ? What is the proper order of these steps?
A. 1, 3, 2, 4, 6, 5. A. 1, 3, 2, 4, 6, 5.
B. 1, 2, 3, 4, 5, 6. B. 1, 2, 3, 4, 5, 6.
C. 1, 3, 4, 2, 6, 5. C. 1, 3, 4, 2, 6, 5.
D. 1, 3, 4, 2, 5, 6. D. 1, 3, 4, 2, 5, 6.
La réponse (A) est juste. La succession appropriée des étapes Answer (A) is correct. The proper sequence of steps in a
d'un processus de contrôle type est la suivante : typical control process is as follows:
1. Sélection des points de contrôle stratégiques au niveau 1. Selecting strategic control points at which to gather
desquels seront recueillies des informations sur les activités en information about activities being performed
cours de réalisation. 2. Observing the work or collecting samples of data
2. Observation du travail ou collecte d'échantillons de données. 3. Accumulating, classifying, and recording data samples
3. Accumulation, classification et enregistrement d'échantillons 4. Comparing samples with predetermined quality,
de données. schedule, and cost standards
4. Comparaison des échantillons avec les normes prédéfinies 5. Determining whether performance is satisfactory
relatives à la qualité, au calendrier et aux coûts. 6. Reporting significant deviations to managers
5. Vérification du degré satisfaisant des performances. concerned
6. Notification des écarts significatifs aux cadres concernés. 7. Determining, by repeating the above steps, whether
7. Vérification, par la répétition des étapes ci-dessus, de action taken is effective in correcting reported deviations
l'efficacité des dispositions prises pour corriger les écarts (follow-up)
signalés (suivi). 8. Reviewing and revising standards
8. Examen et révision des normes. Answer (B) is incorrect because observation must occur
La réponse (B) est fausse. L'observation doit précéder la classification before classifying and recording data samples.
et l'enregistrement des échantillons de données. Answer (C) is incorrect because data must be recorded
La réponse (C) est fausse. Les données doivent être enregistrées before comparisons can occur.
avant de pouvoir être comparées. Answer (D) is incorrect because data must be recorded
La réponse (D) est fausse. Les données doivent être enregistrées before comparisons can occur.
avant de pouvoir être comparées.

Restitution

© IFACI – CIA1 V.5.1 page 27/73


Acquérir une certification
> Préparation au CIA, partie 1
P1- 48 -- Lequel des éléments suivants est un facteur de risque ? P1- 48 -- Which of the following is a factor affecting risk?
A. Du personnel nouveau. A. New personnel.
B. Des systèmes informatiques nouveaux ou modernisés. B. New or revamped information systems.
C. Une croissance rapide. C. Rapid Growth.
D. Toutes les réponses sont justes. D. All of the answers are correct.
La réponse (A) est fausse. Des systèmes informatiques nouveaux ou Answer (A) is incorrect because new or revamped information
modernisés et une croissance rapide font également partie des systems and rapid growth are also factors affecting risk.
facteurs qui exercent une incidence sur le risque. Answer (B) is incorrect because new personnel and rapid
La réponse (B) est fausse. Du personnel nouveau et une croissance growth are also factors affecting risk.
rapide font également partie des facteurs qui exercent une incidence Answer (C) is incorrect because new personnel and new or
sur le risque. revamped information systems are also factors affecting risk.
La réponse (C) est fausse. Du personnel nouveau et des systèmes Answer (D) is correct. New personnel, new or revamped
informatiques nouveaux ou modernisés font également partie des information systems, and rapid growth are all factors that
facteurs qui exercent une incidence sur le risque. affect risk.
La réponse (D) est juste. Du personnel nouveau, des systèmes
informatiques nouveaux ou modernisés et une croissance rapide
sont tous des facteurs qui exercent une incidence sur le risque.

P1- 49 -- Laquelle des affirmations suivantes sur le gouvernement P1- 49 -- Which of the following statements regarding
d'entreprise n'est pas correcte ? corporate governance is not correct?
A. Les mécanismes de contrôle des entreprises peuvent être internes A. Corporate control mechanisms include internal and
et externes. external mechanisms.
B. La grille de rémunération de la direction fait partie des mécanismes B. The compensation scheme for management is part of the
de contrôle des entreprises. corporate control mechanisms.
C. La dilution de la richesse des actionnaires dues aux stock-options C. The dilution of shareholders' wealth resulting from
ou aux primes en actions accordées aux salariés relève davantage de employee stock options or employee stock bonuses is an
la comptabilité que du gouvernement d'entreprise. accounting issue rather than a corporate governance issue.
D. L'auditeur interne d'une entreprise a davantage de responsabilités D. The internal auditor of a company has more responsibility
que le conseil d'administration dans le domaine du gouvernement than the board for the company's corporate governance.
d'entreprise.
La réponse (A) est fausse. Les mécanismes de contrôle des Answer (A) is incorrect. Corporate control mechanisms do
entreprises sont bien internes et externes. include internal and external mechanisms.
La réponse (B) est fausse. La grille de rémunération de la direction fait Answer (B) is incorrect. Management's compensation
bien partie des mécanismes de contrôle des entreprises. scheme is part of corporate control mechanisms.
La réponse (C) est fausse. La dilution de la richesse des actionnaires Answer (C) is incorrect. The dilution of shareholder's wealth
dues aux stock-options ou aux primes en actions accordées aux resulting from employee stock options or employee stock
salariés relève effectivement davantage de la comptabilité que du bonuses is an accounting issue rather than a corporate
gouvernement d'entreprise. governance issue.
La réponse (D) est juste. C'est le conseil d'administration, et non Answer (D) is correct. The board is ultimately
les auditeurs internes, qui est responsable en dernier ressort du responsible for the company's corporate governance, not
gouvernement d'entreprise. the internal auditors.

Restitution

© IFACI – CIA1 V.5.1 page 28/73


Acquérir une certification
> Préparation au CIA, partie 1
[Enoncé #14] Le processus d'audit interne fait appel à une pensée [Fact Pattern #14] The internal auditing process is one of
critique, à une analyse et à une évaluation précise. Toutes les critical thinking, analysis, and careful evaluation. All
procédures mécaniques sont intégrées dans un contexte plus vaste mechanical procedures are integrated into a larger context of
d'enquête minutieuse. Toutes les missions comportent une description thoughtful inquiry. All engagements include a description and
et une analyse des contrôles internes. Les clients de la mission sont analysis of internal controls. Engagement clients are selected
sélectionnés de diverses manières, le risque étant le premier critère de in a number of ways, with risk being the primary basis for
sélection. Les services qu'il est envisagé d'examiner dans l'année à selection. The departments being considered for possible
venir et leurs caractéristiques sont les suivants : review in the coming year and attributes of those departments
are as follows:
Service Actifs Coûts annuels Probabilité de perte Department Assets Annual Costs Probability of Loss
Production A 50 000 $ 700 000 $ 10 % Production A $50,000 $700,000 10%
Production B 5 000 000 $ 10 000 000 $ 1% Production B $5,000,000 $10,000,000 1%
Production C 1 000 000 $ 1 000 000 $ 1% Production C $1,000,000 $1,000,000 1%
Achats 50 000 $ 150 000 $ 10 % Purchasing $50,000 $150,000 10%
Marketing 50 000 $ 500 000 $ 10 % Marketing $50,000 $500,000 10%
Expéditions 60 000 $ 100 000 $ 50 % Shipping $60,000 $100,000 50%
Sécurité 10 000 $ 100 000 $ 90 % Security $10,000 $100,000 90%
Déplacements 6 000 $ 30 000 $ 50 % Travel $6,000 $30,000 50%

Tous ces départements, sauf deux, figurent sur la liste des clients de All of these departments, except two, are on the potential list
mission potentiels suite à une analyse des risques par le responsable of engagement clients because of a risk analysis performed
de l’audit interne. Le service de production A est sur la liste parce que by the chief audit executive. Production department A is on
le président pense qu’il existe trop de goulets d’étranglement dans ce the list because the president thinks too many bottlenecks
département. Le service marketing est sur la liste parce que le chef de occur in that department. The marketing department is on the
la sécurité a reçu un coup de téléphone anonyme qui affirmait que l’un list because the chief of security received an anonymous
des responsables du marketing acceptait d’importants pots de vin de la phone call accusing a marketing manager of accepting
part d’un organe de presse. Les contrôles internes semblent adéquats substantial financial kickbacks from a media outlet. Internal
dans tous les services, à l’exception éventuellement du marketing. controls seem adequate in all departments, with the possible
exception of marketing.

P1- 50 -- (Voir énoncé 14) Quelle est la définition la plus logique du P1- 50 -- (Refers to Fact Pattern #14) What is the chief audit
risque de perte que le responsable de l'audit interne utilise lorsqu'il executive's most logical definition of risk of loss to be used in
sélectionne les clients de la mission ? selecting engagement clients?

A. Le degré d'exposition au risque multiplié par la probabilité de perte. A. Amount of risk exposure times the probability of loss.
B. Le montant des coûts annuels dans un service. B. Amount of annual costs in a department.
C. La probabilité de perte. C. Probability of loss.
D. La quantité d'actifs au sein d'un service. D. Amount of assets in a department.

La réponse (A) est juste. Pour faciliter l'établissement des ordres Answer (A) is correct. To facilitate development of an
de priorité, le responsable de l'audit interne doit s'appuyer sur « engagement work schedule, the CAE should consider
une évaluation à jour des risques et de l'efficacité des dispositifs updated assessments of risks and the effectiveness of
de management des risques et de contrôle » (CRIPP). Le risque risk management and control processes (IPPF). Risk is
est la « possibilité que se produise un événement qui aura un the uncertainty of the occurrence of an event that could
impact sur la réalisation des objectifs. Le risque se mesure en affect the achievement of the organization's objectives. It
termes de conséquences et de probabilité » (glossaire annexé is measured in terms of consequences and likelihood
aux Normes). Ainsi, le risque de perte se définit logiquement (Glossary appended to the Standards). Thus, risk of loss
comme une valeur attendue. Il est égal au montant à risque is most logically defined as an expected value. It equals
multiplié par la probabilité de perte. the amount at risk times the probability of loss.
La réponse (B) est fausse. Le montant des coûts dans un département Answer (B) is incorrect because the amount of costs in a
n'est pas nécessairement équivalent au montant exposé à un risque department is not necessarily the amount exposed to a risk of
de perte. loss.
La réponse (C) est fausse. La probabilité de perte doit être multipliée Answer (C) is incorrect because the probability of a loss must
par le montant exposé à un risque de perte. be multiplied by the amount exposed to possible loss.
La réponse (D) est fausse. La quantité d'actifs dans un service n'est Answer (D) is incorrect because the amount of assets in a
pas nécessairement équivalante au montant exposé à un risque de department is not necessarily the amount exposed to a risk of
perte. loss.

Restitution

© IFACI – CIA1 V.5.1 page 29/73


Acquérir une certification
> Préparation au CIA, partie 1
P1 - 51 -- Quel outil de planification de mission est de nature générale et P1 - 51 -- Which engagement-planning tool is general in nature and
veille à ce que l'étendue de la mission de l'audit soit adéquate sur la durée ? is used to ensure adequate audit coverage over time?
A. Le programme de travail à long terme. A. The long-range schedule.
B. Le programme de mission. B. The engagment program.
C. Le budget de l'audit interne. C. The audit activity's budget.
D. La charte d'audit interne. D. The audit activity's charter.
La réponse (A) est juste. Le programme de travail à long terme prouve Answer (A) is correct. The long-range schedule provides
que les fonctions clés ont été vérifiées selon le calendrier prévu. evidence of coverage of key functions at planned intervals.
La réponse (B) est fausse. le champ du programme de mission est limité à Answer (B) is incorrect. The engagement program is limited in
un projet donné. scope to a particular project.
La réponse (C) est fausse. Le budget de l'audit interne peut servir à justifier Answer (C) is incorrect. The audit activity's budget may be used to
les effectifs du service d'audit, mais il ne permet pas de garantir que l'étendue justify the number of audit personnel, but it is not used to ensure
de la mission soit adéquate sur la durée. adequate audit coverage over time.
La réponse (D) est fausse. La charte d'audit interne n'est pas un outil de Answer (D) is incorrect. The audit activity's charter is not an
planification de mission. engagement-planning tool.

P1 - 52 -- La vision qu'a le management du rôle de l'audit interne sera P1 - 52 -- Management's view of the internal audit activity's role is
vraisemblablement déterminée par tous les facteurs suivants sauf : likely to be determined by all of the following factors except:
A. La culture de l'organisation. A. Organizational culture.
B. Les préférences de l'auditeur indépendant. B. Preferences of the independent auditor.
C. La capacité de l'équipe d'audit interne. C. Ability of the internal auditing staff.
D. La situation et les usages du pays. D. Local conditions and customs of the country.
La réponse (A) est fausse. C'est un facteur qui influence la vision du Answer (A) is incorrect because this is a factor that influences
management quant au rôle de l'audit interne. management's view on the role of internal audit.
La réponse (B) est juste. Il revient aux cadres dirigeants et au comité Answer (B) is correct. It is the role of executive management
d'audit de déterminer le rôle de l'audit interne dans le processus de and the audit committee to determine the role of internal audit
management des risques. La vision du management dans ce domaine in the risk management process. Management's view on
sera vraisemblablement fonction de facteurs tels que la culture de internal audit's role is likely to be determined by factors such
l'organisation, la compétence des auditeurs internes, la situation et les as the culture of the organization, ability of the internal
usages du pays (CRIPP). auditing staff, and local conditions and customs of the
La réponse (C) est fausse. C'est un facteur qui influence la vision du country (IPPF).
management quant au rôle de l'audit interne. Answer (C) is incorrect because this is a factor that influences
La réponse (D) est fausse. C'est un facteur qui influence la vision du management's view on the role of internal audit.
management quant au rôle de l'audit interne. Answer (D) is incorrect because this is a factor that influences
management's view on the role of internal audit.

P1 - 53 -- Les détails du programme d'audit servant à auditer les activités de P1 - 53 -- The details of the audit program used to audit e-
commerce électronique dans des organisations spécifiques varient en commerce activities in specific organizations will vary depending on
fonction de tous les facteurs suivants, à l'exception d'un, lequel ? all but which of the following factors?
A. Le secteur d'activité. A. Industry.
B. La culture de l'organisation. B. Organizational culture.
C. Le pays. C. Country.
D. Les modèles juridiques et d'entreprise. D. Legal and business models.
La réponse (A) est fausse. Il s'agit d'un facteur dont dépendent les détails du Answer (A) is incorrect because this is a factor upon which the
programme d'audit servant à vérifier les activités de commerce électronique. details of the audit program used to audit e-commerce activities are
La réponse (B) est juste. Les détails du programme d'audit servant à dependent.
vérifier les activités de commerce électronique dans des organisations Answer (B) is correct. The details of the audit program used to
spécifiques varient en fonction du secteur d'activité, du pays et des audit e-commerce activities in specific organizations will vary
modèles juridiques et d'entreprise. depending on industry, country, and legal and business
La réponse (C) est fausse. Il s'agit d'un facteur dont dépendent les détails du models.
programme d'audit servant à vérifier les activités de commerce électronique. Answer (C) is incorrect because this is a factor upon which the
La réponse (D) est fausse. Il s'agit d'un facteur dont dépendent les détails du details of the audit program used to audit e-commerce activities are
programme d'audit servant à vérifier les activités de commerce électronique. dependent.
Answer (D) is incorrect because this is a factor upon which the
details of the audit program used to audit e-commerce activities are
dependent.

Restitution

© IFACI – CIA1 V.5.1 page 30/73


Acquérir une certification
> Préparation au CIA, partie 1
P1 - 54 -- Le service d'audit interne d'une grande entreprise a établi son P1 - 54 -- The internal audit activity of a large corporation has
calendrier d'activités et son budget pour l'exercice à venir. Le calendrier se established its operating plan and budget for the coming year. The
limite aux aspects suivants : liste de toutes les missions par ordre de priorité, operating plan is restricted to the following categories: a prioritized
dotation en personnel, dépenses détaillées et date de début de chaque listing of all engagements, staffing, a detailed expense budget, and
mission. Laquelle des situations suivantes décrit le mieux la principale lacune the commencement date of each engagement. Which of the
de ce calendrier ? following best describes the major deficiency of this operating plan?
A. Les demandes de la direction portant sur des projets spéciaux ne sont pas A. Requests by management for special projects are not
envisagées. considered.
B. Les possibilités d'améliorer le fonctionnement ne sont pas prises en compte. B. Opportunities to achieve operating benefits are ignored.
C. Il n'existe aucun critère de mesure ni aucune date d'achèvement. C. Measurability criteria and targeted dates of completion are not
D. Le savoir, les compétences et les règles nécessaires pour exécuter les provided.
tâches ne sont pas pris en compte. D. Knowledge, skills, and disciplines required to perform work are
ignored.

La réponse (A) est fausse. Ce point serait pris en compte lors de la définition Answer (A) is incorrect. This factor would be considered in
de l'ordre de priorité des missions. prioritizing the engagements.
La réponse (B) est fausse. L'examen de la dotation en personnel, de l'ordre Answer (B) is incorrect. By reviewing staffing, prioritization of
de priorité des missions et des dépenses permet d'améliorer le engagements, and expenses, operating benefits can be achieved.
fonctionnement. Answer (C) is correct. The IPPF states that the goals of the
La réponse (C) est juste. Le CRIPP indique que les objectifs de l'audit internal audit activity, as stated in specific operating plans
interne doivent pouvoir être atteints dans le cadre des plans and budgets, should include measurement criteria and
opérationnels et des budgets fixés, et complétés par des critères de targeted dates of accomplishment.
mesure et un calendrier de réalisation. Answer (D) is incorrect. Staffing for each engagement would
La réponse (D) est fausse. Pour chaque mission, la dotation en personnel include this consideration.
prendrait ce facteur en compte.

P1 - 55 -- Deux auditeurs internes sont récemment partis et n'ont pas pu être P1 - 55 -- The internal audit activity has recently experienced the
remplacés tout de suite, pour des raisons budgétaires. Laquelle des departure of two internal auditors who cannot be immediately
méthodes suivantes constitue l'option la moins souhaitable pour la réalisation replaced due to budget constraints. Which of the following is the
efficiente des missions à venir, étant donné cette réduction des ressources ? least desirable option for efficiently completing future engagements,
given this reduction in resources?

A. Répondre aux objectifs des audits au moyen de questionnaires d'auto- A. Using self-assessment questionnaires to address audit
évaluation. objectives.
B. Recourir aux technologies de l'information pour la planification, B. Employing information technology in audit planning,
l'échantillonnage et la documentation des audits. sampling, and documentation.
C. Supprimer les missions de conseil du programme d'audit. C. Eliminating consulting engagements from the engagement work
D. Pourvoir les postes vacants avec du personnel venant de services schedule.
d'exploitation qui ne sont pas en cours d'audit. D. Filling vacancies with personnel from operating departments that
are not being audited.

La réponse (A) est fausse. Un questionnaire d'auto-évaluation constitue un Answer (A) is incorrect. Self-assessment questionnaires are a
moyen d'atteindre avec efficience les objectifs de certains audits internes. means of efficiently addressing the objectives of certain internal
La réponse (B) est fausse. Le recours à la technologie constitue un moyen audits.
approprié de réaliser un audit avec efficience. Answer (B) is incorrect. Use of technology is an appropriate means
La réponse (C) est juste. Le programme d'audit ne doit être allégé qu'en of achieving efficiencies in audit execution.
dernier recours, lorsque toutes les autres solutions viables ont été Answer (C) is correct. The audit schedule should only be
explorées, y compris la demande de ressources supplémentaires. reduced as a last resort once all other viable alternatives have
La réponse (D) est fausse. Le recours à du personnel d'exploitation manifestant been explored, including the request for additional resources.
de l'intérêt pour l'audit interne et disposant d'une expérience de l'entreprise Answer (D) is incorrect. Using operating personnel with internal
constitue un moyen approprié de renforcer les ressources de l'audit interne. audit interest and corporate experience is an appropriate way to
enhance internal audit resources.

Restitution

© IFACI – CIA1 V.5.1 page 31/73


Acquérir une certification
> Préparation au CIA, partie 1
P1 - 56 -- Lorsque deux projets doivent être audités mais que les P1 - 56 -- Which of the following comments is correct regarding the
ressources d'audit sont limitées, laquelle (ou lesquelles) des assessment of risk associated with two projects that are competing
considérations suivantes concernant l'évaluation des risques associés for limited audit resources?
à ces projets faut-il prendre en compte ?
I. Activities that are requested by the audit committee should
I. Il faut toujours considérer que les activités demandées par le comité d'audit always be considered higher risk than those requested by
portent sur des projets plus risqués que celles demandées par la direction. management.
II. Il faut toujours considérer que les projets dont les budgets sont les plus II. Activities with higher dollar budgets should always be
élevés induisent un risque supérieur aux autres. considered higher risk than those with lower dollar budgets.
III. Il faut toujours mesurer le risque par l'exposition financière potentielle ou III. Risk should always be measured by the potential dollar or
par ses effets délètères sur l'organisation. adverse exposure to the organization.
A. I uniquement. A. I only.
B. II uniquement. B. II only.
C. III uniquement. C. III only.
D. I et III uniquement. D. I and III only.
La réponse (A) est fausse. L'audit interne doit prendre en compte à la fois les Answer (A) is incorrect. Requests from management and the audit
demandes de la direction et celles du comité d'audit. Même si une demande committee should both be considered by the internal audit activity.
de ce dernier est importante, elle n'est pas toujours plus importante et ne Although an audit committee request is important, it is not always
porte pas dans tous les cas sur des projets plus risqués. more important, nor does it always imply higher risk.
La réponse (B) est fausse. Le risque est mesuré par l'exposition potentielle Answer (B) is incorrect. Risk is measured by the potential exposure
de l'organisation. La taille du budget des différents projets est un paramètre to the organization. The size of the departmental budget is an
important, mais pas suffisant. important determinant, but is not a sufficient determinant.
La réponse (C) est juste. D'après le CRIPP, le degré ou la matérialité de Answer (C) is correct. IPPF advises that the degree or
l'exposition constitue un important paramètre du risque. materiality of exposure is an important component of risk.
La réponse (D) est fausse. L'audit interne doit prendre en compte à la fois les Answer (D) is incorrect. Requests from management and the audit
demandes de la direction et celles du comité d'audit. Même si une demande committee should both be considered by the internal audit activity.
de ce dernier est importante, elle n'est pas toujours plus importante et ne Although an audit committee request is important, it is not always
porte pas dans tous les cas sur des projets plus risqués. more important, nor does it always imply higher risk.

P1 - 57 -- Lequel des facteurs suivants n'est pas à prendre en compte pour P1 - 57 -- Which of the following factors is not included in
fixer l'ordre des priorités du plan d'audit ? determining the engagement work schedule?
A. Les programmes de travail des missions. A. Engagement work programs.
B. L'efficacité des dispositifs de management des risques et de contrôle. B. The effectiveness of risk management and control processes.
C. Les exigences relatives à la charge de travail. C. Workload requirements.
D. Les aspects relatifs au gouvernement d'entreprise. D. Issues relating to organizational governance.
La réponse (A) est juste. “Le programme de missions doit préciser : a) Answer (A) is correct. Engagement work schedules should
les activités à auditer, b) la date des missions, c) une estimation du include (a) what activities are to be performed; (b) when they
temps nécessaire tenant compte de l'étendue de la mission prévue will be performed; and (c) the estimated time required, taking
ainsi que de la nature et de l'étendue des travaux d'audit réalisés par into account the scope of the engagement work planned and
d'autres intervenants. Les points à prendre en considération pour the nature and extent of related work performed by others.
l'établissement des ordres de priorité sont : a) la date et les résultats de Matters to be considered in establishing engagement work
la précédente mission ; b) une évaluation à jour des risques et de schedule priorities should include (a) the dates and results of
l'efficacité des dispositifs de management des risques et de contrôle ; the last engagement; (b) updated assessments of risks and
c) les demandes émanant de la direction générale, du comité d'audit et effectiveness of risk management and control processes; (c)
d'autres organes de direction ; d) les problématiques actuelles relatives requests by senior management, the audit committee, and the
au gouvernement d'entreprise ; e) les changements importants governing body; (d) current issues relating to organizational
survenus dans l'activité, les tâches opérationnelles, les programmes, governance; (e) major changes in the enterprise's business,
les systèmes et les contrôles ; f) les opportunités de réaliser des operations, programs, systems, and controls; (f)
bénéfices d'exploitation ; g) les modifications intervenues dans l'équipe opportunities to achieve operating benefits; and (g) changes
d'audit interne et ses aptitudes. Le planning des travaux doit être in and capabilities of the staff. The work schedules should be
suffisamment flexible pour faire face à des demandes de missions non sufficiently flexible to cover unanticipated demands on the
prévues” (CRIPP). L'élaboration des programmes de travail a lieu IAA (IPPF). Development of work programs occurs during the
pendant la phase de planification de la mission d'audit. planning phase of an individual engagement.
La réponse (B) est fausse. Les points à prendre en considération pour fixer Answer (B) is incorrect because determining an engagement work
l'ordre des priorités incluent l'évaluation de l'efficacité des dispositifs de schedule includes considering the effectiveness of risk
management des risques et de contrôle. management and control processes.
La réponse (C) est fausse. Les points à prendre en considération pour fixer Answer (C) is incorrect because determining an engagement work
l'ordre des priorités incluent les exigences relatives à la charge de travail. schedule includes considering workload requirements.
La réponse (D) est fausse. Les points à prendre en considération pour fixer Answer (D) is incorrect because determining an engagement work
l'ordre des priorités incluent les aspects relatifs au gouvernement d'entreprise. schedule includes considering issues relating to organizational
governance.

Restitution

© IFACI – CIA1 V.5.1 page 32/73


Acquérir une certification
> Préparation au CIA, partie 1
P1 - 58 -- En discutant avec la direction générale, le responsable de l'audit P1 - 58 -- During discussions with senior management, the chief
interne a identifié plusieurs problèmes stratégiques à prendre en audit executive identified several strategic business issues to
considération pour l'élaboration du calendrier des missions de l'année. consider in preparing the annual engagement work schedule.
Laquelle des propositions suivantes ne constitue pas un problème Which of the following dœs not represent a strategic issue for this
stratégique à cet égard ? purpose?
A. Une budgétisation mensuelle. A. A monthly budgeting process will be implemented.
B. Le lancement d'une campagne marketing internationale afin d'accroître la B. An international marketing campaign will be started to develop
reconnaissance des produits mais aussi de tirer parti du service publicité product recognition and also to leverage the new organization-
nouvellement créé au sein de l'organisation. based advertising department.
C. La recherche de candidats en vue d'une co-entreprise visant à renforcer C. Joint venture candidates will be sought to provide manufacturing
les capacités de production et les débouchés sur les marchés européens et and sourcing capabilities in European and Asian markets.
asiatiques. D. A human resources database will be established to ensure
D. La constitution d'une base de données sur les ressources humaines afin consistent administration of policies and to improve data retention.
de garantir une administration cohérente des mesures et d'améliorer la
conservation des données.
La réponse (A) est juste. “La démarche d'audit peut intégrer certaines Answer (A) is correct. The audit universe may include
composantes du plan stratégique de l'organisation, de façon à tenir components from the organization's strategic plan. By
compte de ses objectifs globaux” (CRIPP). Cependant, la mise en including components of this plan, the audit universe will
œuvre d'une budgétisation mensuelle constitue une décision consider and reflect the overall business plan objectives
opérationnelle destinée à faciliter le processus de budgétisation et (IPPF). However, implementing a monthly budgeting process
d'améliorer l'information. Elle ne constitue pas un problème is an operating decision to facilitate the budgeting process
stratégique, mais suppose notamment un changement majeur au and improve information. It dœs not constitute a strategic
niveau des opérations. issue, but it dœs entail a major change in operations, etc.
La réponse (B) est fausse. Le responsable de l'audit interne devra s'assurer que Answer (B) is incorrect because the CAE will need to ensure that
le nouveau dispositif de marketing et le service publicité centralisé sont traités et the new marketing process and the centralized advertising
suivis dans le cadre des activités de planification et d'évaluation des risques. department are recognized and monitored in risk assessment and
La réponse (C) est fausse. L'ajout des partenaires à la co-entreprise ne fera planning activities.
qu'accroître ou renforcer les problèmes d'évaluation des risques et de Answer (C) is incorrect because the addition of joint-venture
planification de l'audit interne. partners will add new or additional concerns for risk assessment
La réponse (D) est fausse, car tant les hypothèses que les activités en cours and planning in the IAA.
liées à une base de données sur les ressources humaines devront être Answer (D) is incorrect because both the assumptions and ongoing
prises en compte lors de la planification de l'audit interne. activities related to a human resources database will require
consideration in the planning of the IAA.

P1 - 59 -- Lequel des éléments suivants ne sera probablement pas inclus P1 - 59 -- Which of the following is least likely to be included in the
dans la planification des missions de l’audit interne ? engagement work schedule of the internal audit activity?
A. Être cohérent avec sa charte. A. To be consistent with its charter.
B. Être réalisable. B. To be capable of being accomplished.
C. Inclure une liste d’activités à accomplir. C. To include a list of activities to be performed.
D. Inclure les bases du programme de travail des missions. D. To include the basics of the engagement work program.
La réponse (A) est fausse. La cohérence avec la charte est nécessaire. Answer (A) is incorrect because consistency with the charter is
La réponse (B) est fausse. Les objectifs doivent être réalisables. necessary.
La réponse (C) est fausse. Le programme d’une mission inclut les Answer (B) is incorrect because goals should be capable of being
tâches à accomplir. accomplished.
La réponse (D) est juste. Le responsable de l’audit interne doit établir Answer (C) is incorrect because an engagement work schedule
une planification fondée sur les risques afin de définir les priorités includes the activities to be performed.
cohérentes avec les priorités de l’audit interne (CRIPP). Cette Answer (D) is correct. The chief audit executive should
planification doit être cohérente avec la charte de l’AI et avec les establish risk-based plans to determine the priorities of the
objectifs de l’organisation. De plus, les objectifs de l’AI doivent être internal audit activity (IPPF). These plans should be
réalisables à travers des plans opérationnels et des budgets qui ont été consistent with the IAA's charter and with the goals of the
définis et, dans la mesure du possible, doivent être quantifiables. Ils organization. Moreover, the goals of the IAA should be
doivent être accompagnés de critères de mesure et des dates de capable of being accomplished within specified operating
réalisation. De plus, la planification des missions doit inclure (a) les plans and budgets and, to the extent possible, should be
activités qui doivent être réalisées; (b) quand elles seront réalisées; et measurable. They should be accompanied by measurement
(c) une estimation du temps nécessaire, prenant en considération le criteria and targeted dates of accomplishment. Furthermore,
champ de la mission ainsi que la nature et l’étendue de tous les travaux engagement work schedules should include (a) what
effectués par d’autres. (CRIPP). Les éléments fondamentaux d’un activities are to be performed; (b) when they will be
programme pour une mission spécifique ne sont pas inclus dans les performed; and (c) the estimated time required, taking into
plans à long terme de l’AI. Un programme de travail documente les account the scope of the engagement work planned and the
procédures relatives à la mission et choisies avant de conduire la nature and extent of related work performed by others (IPPF).
mission. Cependant, il est normalement préparé après que les The basics of an engagement program for a specific
informations de base et de contexte ont été recueillies et une étude engagement are not included in the long-range plans of the
préliminaire réalisée. IAA. A work program documents the engagement procedures
selected in advance of performing the engagement. However,
it is normally prepared after background information has been
gathered and a survey has been conducted.

Restitution

© IFACI – CIA1 V.5.1 page 33/73


Acquérir une certification
> Préparation au CIA, partie 1
P1- 60 -- Lors de la planification d'une mission, lequel des P1- 60 -- In planning an engagement, internal auditors should
éléments suivants, doit être pris en compte par les auditeurs consider which of the following items?
internes ?
I. Les objectifs de l'activité auditée et les moyens par lesquels I. The objectives of the activity being reviewed and the means by
celle-ci contrôle ses performances. which the activity controls performance.
II. L'adéquation et l'efficacité des systèmes de gestion des risques II. The adequacy and effectiveness of the activity's risk
et de contrôle en référence à un cadre ou modèle pertinent. management and control systems compared to a relevant control
III. Les opportunités d'améliorations significatives des systèmes de framework or model.
contrôle et des systèmes informatiques de l'activité concernée. III. The opportunities for making significant improvements to the
activity's information technology systems and control systems.
A. I et II.
B. II et III. A. I and II.
C. I et III. B. II and III.
D. I, II et III. C. I and III.
D. I, II, and III.
La réponse (A) est juste. En planifiant leur mission, « les Answer (A) is correct. In planning the engagement, internal
auditeurs internes doivent prendre en compte : les objectifs auditors should consider: The objectives of the activity being
de l'activité soumise à l'audit et la manière dont elle est reviewed and the means by which the activity controls
maîtrisée ; les risques significatifs liés à l'activité, ses performance. The significant risks to the activity, its
objectifs, les ressources mises en œuvre et ses tâches objectives, resources, and operations and the means by
opérationnelles, ainsi que les moyens par lesquels l'impact which the potential impact of risk is kept to an acceptable
potentiel du risque est maintenu à un niveau acceptable ; la level. The adequacy and effectiveness of the activity's risk
pertinence et l'efficacité des systèmes de management des management and control systems compared to a relevant
risques et de contrôle de l'activité, en référence à un cadre ou control framework or model. The opportunities for making
modèle de contrôle appropriés ; les opportunités d'améliorer significant improvements to the activity's risk management
de manière significative les systèmes de management des and control systems. (IPPF)
risques et de contrôle de l'activité ». (CRIPP) Answer (B) is incorrect because the internal auditor does not have
La réponse (B) est fausse. L'auditeur interne ne doit pas prendre to consider opportunities to improve the activity's information
en compte les opportunités d'amélioration du système technology system.
informatique mis en place par l'activité concernée. Answer (C) is incorrect because the internal auditor does not have
La réponse (C) est fausse. L'auditeur interne ne doit pas prendre to consider opportunities to improve the activity's information
en compte les opportunités d'amélioration du système technology system.
informatique mis en place par l'activité concernée. Answer (D) is incorrect because the internal auditor does not have
La réponse (D) est fausse. L'auditeur interne ne doit pas prendre to consider opportunities to improve the activity's information
en compte les opportunités d'amélioration du système technology system.
informatique mis en place par l'activité concernée.

Restitution

© IFACI – CIA1 V.5.1 page 34/73


Acquérir une certification
> Préparation au CIA, partie 1
P1- 61 -- Comment le responsable de l'audit interne doit-il P1- 61 -- For control and monitoring functions that the CAE has a
envisager sa relation avec les autres services investis d'une relationship with (risk management, compliance, security, legal,
fonction de contrôle et de surveillance (management des risques, etc.) the CAE should do which of the following?
déontologie, sécurité, juridique, etc.) ? Il doit :
A. Faciliter la remontée d'informations vers le directeur des risques A. Facilitate the reporting of material risk and control issues to the
significatifs et des problèmes liés aux contrôles. Chief Executive Officer.
B. Faciliter la remontée d'informations vers l'auditeur externe des B. Facilitate the reporting of material risk and control issues to the
risques significatifs et des problèmes liés aux contrôles. external auditor.
C. Faciliter la remontée d'informations vers le comité d'audit des C. Facilitate the reporting of material risk and control issues to the
risques significatifs et des problèmes liés aux contrôles. audit committee.
D. Ne faciliter la remontée d'informations des risques significatifs D. Facilitate the reporting of material risk and control issues to no
et des problèmes liés aux contrôles vers personne, car cela ne one, since this is not a responsibility of the CAE.
relève pas du responsable de l'audit interne.
La réponse (A) est fausse. Les responsables de l'audit interne Answer (A) is incorrect because CAEs should consider their
doivent aussi envisager leurs relations avec les autres services relationships with other control and monitoring functions and
investis d'une fonction de contrôle et de surveillance et faciliter la facilitate the reporting of material risk and control issues to the
communication au comité d'audit des risques significatifs et des audit committee.
problèmes liés aux contrôles. Answer (B) is incorrect because CAEs should consider their
La réponse (B) est fausse. Les responsables de l'audit interne relationships with other control and monitoring functions and
doivent aussi envisager leurs relations avec les autres services facilitate the reporting of material risk and control issues to the
investis d'une fonction de contrôle et de surveillance et faciliter la audit committee.
communication au comité d'audit des risques significatifs et des Answer (C) is correct. CAEs should consider their
problèmes liés aux contrôles. relationships with other control and monitoring functions
La réponse (C) est juste. « Les responsables de l'audit interne (risk management, compliance, security, legal, ethics,
doivent aussi envisager leurs relations avec les autres environmental, external audit) and facilitate the reporting of
services investis d'une fonction de contrôle et de surveillance material risk and control issues to the audit committee
(management des risques, déontologie, sécurité, juridique, (IPPF).
éthique, environnement, audit externe), et faciliter la Answer (D) is incorrect because CAEs should consider their
communication au comité d'audit des risques significatifs et relationships with other control and monitoring functions and
des problèmes liés aux contrôles » (CRIPP). facilitate the reporting of material risk and control issues to the
La réponse (D) est fausse. Les responsables de l'audit interne audit committee.
doivent aussi envisager leurs relations avec les autres services
investis d'une fonction de contrôle et de surveillance et faciliter la
communication au comité d'audit des risques significatifs et des
problèmes liés aux contrôles.

Restitution

© IFACI – CIA1 V.5.1 page 35/73


Acquérir une certification
> Préparation au CIA, partie 1
P1- 62 -- Un auditeur interne a jugé qu'un élément n'était pas P1- 62 -- An internal auditor judged an item to be immaterial when
important au moment de la planification de sa mission planning an assurance engagement. However, the assurance
d'assurance. Cette mission pourra néanmoins inclure cet élément engagement may still include the item if it is subsequently
s'il est par la suite déterminé que : determined that
A. Des effectifs suffisants sont disponibles. A. Sufficient staff is available.
B. Des effets négatifs liés à cet élément sont susceptibles de B. Adverse effects related to the item are likely to occur.
survenir. C. Related information is reliable.
C. Les informations y afférentes sont fiables. D. Miscellaneous income is affected.
D. Des sources de revenus diverses sont affectées.
La réponse (A) est fausse. En l'absence de toute autre Answer (A) is incorrect because, in the absence of other
considération, déployer un effort de mission complémentaire pour considerations, devoting additional engagement effort to an
un élément non important serait inefficient. immaterial item is inefficient.
La réponse (B) est juste. L'auditeur interne doit apporter tout Answer (B) is correct. Internal auditors should exercise due
le soin nécessaire à sa pratique professionnelle en prenant en professional care by considering the relative complexity,
considération la complexité relative, la matérialité ou le materiality, or significance of matters to which assurance
caractère significatif des domaines auxquels sont appliquées procedures are applied (IPPF). Materiality judgments are
les procédures propres aux missions d'assurance (CRIPP). made in the light of all the circumstances and involve
L'évaluation de l'importance est effectuée à la lumière du qualitative as well as quantitative considerations. Moreover,
contexte dans son ensemble et suppose des considérations internal auditors must also consider the interplay of risk with
tant qualitatives que quantitatives. Les auditeurs internes materiality. Consequently, engagement effort may be
doivent en outre prendre en compte l'interaction du risque et required for a quantitatively immaterial item if adverse effects
de la matérialité. Par conséquent, un effort de mission are likely to occur, for example, a material contingent liability
particulier peut être nécessaire pour un élément arising from an illegal payment that is otherwise immaterial.
quantitativement négligeable si des répercussions négatives Answer (C) is incorrect because additional engagement
risquent d'en découler, par exemple un élément de passif procedures might not be needed if related information is reliable.
éventuel important découlant d'un paiement illégal, qui, par Answer (D) is incorrect because the item is more likely to be
ailleurs, serait négligeable. included if it affects recurring income items rather than
La réponse (C) est fausse. Des procédures complémentaires de miscellaneous income.
mission ne seront peut-être pas nécessaires si les informations y
afférentes sont fiables.
La réponse (D) est fausse. L'élément est plus susceptible d'être
inclus s'il affecte des postes de revenu récurrents plutôt que des
recettes diverses.

Restitution

© IFACI – CIA1 V.5.1 page 36/73


Acquérir une certification
> Préparation au CIA, partie 1
P1- 63 -- Après affectation de la mission, la direction a demandé à P1- 63 -- When the engagement was assigned, management
l’auditeur interne d’évaluer si le recours à l’auto-assurance pour asked the internal auditor to evaluate the appropriateness of
minimiser les risques de l’organisation est approprié. Du fait du using self-insurance to minimize risk to the organization. Given
périmètre de la mission demandé par la direction, l’auditeur the scope of the engagement requested by management, should
devrait-il faire appel à un consultant en actuariat si cette the internal auditor engage an actuarial consultant to assist in the
compétence n’existe pas au sein de l’équipe ? engagement if these skills do not exist on staff?
A. Non. L’audit interne est compétent en évaluation des contrôles, A. No. The internal audit activity is skilled in assessing controls,
et les concepts relatifs aux contrôles en assurance ne sont pas and the insurance control concepts are not distinctly different from
vraiment différents des autres concepts en matière de contrôle. other control concepts.
B. Non. L’évaluation des risques est une activité normale pour un B. No. It is a normal internal auditor function to assess risk; this
auditeur ; de ce fait cette mission n’est pas spécifique. engagement is therefore not unique.
C. Oui. Un actuaire est indispensable pour déterminer si les coûts C. Yes. An actuary is essential to determine whether the health
de prévoyance santé sont raisonnables. care costs are reasonable.
D. Oui. L’actuaire dispose de compétences qui ne sont pas D. Yes. The actuary has skills, not usually found among internal
habituelles chez les auditeurs internes, pour identifier et quantifier auditors, to identify and quantify self-insurance risks.
les risques relatifs à l’auto-assurance.
La réponse (A) est fausse. L’évaluation des risques relatifs à Answer (A) is incorrect because assessing self-insurance risks is
l’auto-assurance est en dehors du périmètre normal de l’audit outside the normal scope of the IAA. The internal auditor may
interne. L’auditeur interne peut avoir besoin de faire appel à un need to engage an actuary.
actuaire. Answer (B) is incorrect because assessing self-insurance risks is
La réponse (B) est fausse. L’évaluation des risques relatifs à outside the normal scope of the IAA. The internal auditor may
l’auto-assurance est en dehors du périmètre normal de l’audit need to engage an actuary.
interne. L’auditeur interne peut avoir besoin de faire appel à un Answer (C) is incorrect because an internal auditor might be able
actuaire. to determine whether the health care costs are reasonable.
La réponse (C) est fausse. Un auditeur interne pourrait être Answer (D) is correct. The CAE should obtain competent
capable de déterminer si les coûts de prévoyance santé sont advice and assistance if the internal audit staff lacks the
raisonnables. knowledge, skills, or other competencies needed to perform
La réponse (D) est juste. Le RAI doit chercher conseil et all or part of the engagement (IPPF). Furthermore, the IAA
assistance si le personnel de l’audit interne n’a pas la should have employees or use outside service providers who
connaissance, les qualités ou autres compétences are qualified in such disciplines as accounting, auditing,
nécessaires à la réalisation de tout ou partie d’une mission economics, finance, statistics, information technology,
(CRIPP). Par ailleurs, l’audit interne doit avoir du personnel engineering, taxation, law, and environmental affairs as
compétent en comptabilité, audit, économie, finance, needed to meet the IAA's responsibilities (IPPF). Thus,
statistiques, technologies de l’information, production, unless the IAA has an employee with actuarial skills, an
fiscalité, législation, et environnement ou faire appel à des actuarial consultant should be hired to assess self-insurance
prestataires externes qui ont ces compétences afin d’être à risks.
même d’assumer les responsabilités de l’audit interne
(CRIPP). De ce fait et à moins que l’audit interne n’ait un
employé qui a des compétences en actuariat, un actuaire
devrait être recruté afin d’évaluer les risques relatifs à l’auto-
assurance.

Restitution

© IFACI – CIA1 V.5.1 page 37/73


Acquérir une certification
> Préparation au CIA, partie 1
P1- 64 -- Les auditeurs internes peuvent jouer un rôle plus proactif P1- 64 -- Internal auditors can play a more proactive role in
en participant à la mise en place initiale d'un processus de assisting with the initial establishment of a risk management
management des risques au sein de l'organisation. Si cette process for the organization. However, if such assistance exceeds
assistance dépasse le cadre des missions d'assurance et de normal assurance and consulting activities conducted by internal
conseil généralement confiées aux auditeurs internes, auditors, independence may be impaired. Which of the following
l'indépendance des auditeurs peut être altérée. Laquelle des would impair the independence of an internal auditor who had
propositions suivantes pourrait altérer l'indépendance d'un participated in the initial establishment of a risk management
auditeur interne qui a participé à la mise en place initiale d'un process?
processus de management des risques ?
A. Élaborer des évaluations et rédiger des rapports sur le A. Developing assessments and reports on the risk management
processus de management des risques. process.
B. Gérer les risques identifiés. B. Managing the identified risks.
C. Évaluer le caractère adéquat et l'efficacité des processus de C. Evaluating the adequacy and effectiveness of management's
management des risques. risk processes.
D. Mettre en place des contrôles pour traiter les risques identifiés. D. Implementing controls to address the risks identified.

La réponse (A) est fausse, car élaborer des évaluations et rédiger Answer (A) is incorrect because developing assessments and
des rapports sur le processus de gestion des risques de reports on the organization's risk management processes is not
l'organisation fait non seulement partie du rôle de l'audit interne, only an internal auditing role, but it is normally also a high audit
mais figure normalement en haut de ses priorités. priority.
La réponse (B) est juste. Jouer un rôle plus proactif en Answer (B) is correct. A more proactive role in the initial
participant à la mise en place initiale d'un processus de establishment of a risk management process supplements
management des risques complète la mission classique traditional assurance activities with a consultative approach
d'assurance. Les auditeurs internes assument alors un rôle to improving fundamental processes. However, a proactive
de conseil en vue de l'amélioration des processus role in developing a risk management process is not the
fondamentaux. Il convient de différencier le rôle proactif de same as an "ownership of risks" role, which is a role of
l'auditeur dans la mise en place d'un dispositif de management (IPPF). The internal auditor cannot assume this
management des risques, du rôle de « responsable des role, or any other management, board, or audit committee
risques » qui incombe à la direction (CRIPP). L'auditeur role in the risk management process, without impairing
interne ne peut assumer ce rôle, ni n'importe quel autre rôle independence. Boards and audit committees have an
de la direction, du conseil ou du comité d'audit dans le oversight role to determine that adequate and effective
processus de management des risques, sans porter atteinte à processes are in place, and managers have the responsibility
son indépendance. Il incombe au Conseil et au comité d'audit for the management of the risks identified, and for ensuring
de veiller à ce que des processus de management des that sound risk management processes are in place and
risques appropriés, suffisants et efficaces soient en place et functioning (IPPF). By managing the identified risks, the
la gestion des risques est une responsabilité majeure du internal auditor impairs his/her independence by assuming
management qui doit s'assurer, pour atteindre ses objectifs, management's role.
de la mise en place et du bon fonctionnement des processus Answer (C) is incorrect because internal auditors should assist
de management des risques (CRIPP). En gérant les risques both management and the audit committee by examining,
identifiés, l'auditeur interne porte atteinte à son indépendance evaluating, reporting, and recommending improvements on the
car il assume un rôle qui incombe à la direction. adequacy and effectiveness of management's risk processes.
La réponse (C) est fausse, car « le rôle des auditeurs internes Answer (D) is incorrect because internal auditors acting in a
consiste à assister et le management et le comité d'audit, en consulting role may assist the organization in identifying,
examinant et en évaluant les processus de management des evaluating, and implementing risk management methodologies
risques mis en oeuvre par le management, en vérifiant qu'ils sont and controls to address those risks (IPPF).
suffisants et efficaces, puis en émettant des rapports et des
recommandations en vue de leur amélioration ».
La réponse (D) est fausse, car « les auditeurs internes peuvent,
dans le cadre d'une mission de conseil, aider l'organisation à
identifier, évaluer et mettre en place un dispositif de management
des risques et des contrôles permettant de maîtriser ces risques »
(CRIPP).

Restitution

© IFACI – CIA1 V.5.1 page 38/73


Acquérir une certification
> Préparation au CIA, partie 1
P1- 65 -- Les auditeurs internes doivent élaborer et enregistrer un P1- 65 -- Internal auditors should develop and record a plan for
plan pour chaque mission. La planification doit inclure tous les each engagement. The planning process should include all the
éléments suivants sauf following except

A. La définition des objectifs et du champ d'intervention. A. Establishing engagement objectives and scope of work.
B. L'obtention d'informations sur le contexte des activités B. Obtaining background information about the activities to be
examinées. reviewed.
C. La collecte d'informations suffisantes pour permettre d'atteindre C. Identifying sufficient information to achieve engagement
les objectifs de la mission. objectives.
D. La détermination des moyens, du moment et des personnes à D. Determining how, when, and to whom the engagement results
qui les résultats de la mission seront communiqués. will be communicated.
La réponse (A) est fausse. La planification inclut la définition des Answer (A) is incorrect because the planning process includes
objectifs et du champ d'application. establishing engagement objectives and scope of work.
La réponse (B) est fausse. La planification inclut l'obtention Answer (B) is incorrect because the planning process includes
d'informations sur le contexte. obtaining background information.
La réponse (C) est juste. La planification doit notamment Answer (C) is correct. Planning should include establishing
définir les objectifs et le champ d'application de la mission, engagement objectives and scope of work, obtaining
obtenir des informations sur le contexte des activités background information about the activities to be reviewed,
examinées, déterminer les ressources appropriées pour determining the resources necessary to perform the
atteindre les objectifs de la mission et informer les niveaux engagement, and informing those in management who need
hiérarchiques qui doivent être au courant de la mission. Elle to know about the engagement. It also includes performing,
inclut également la réalisation, si nécessaire, d'un examen as appropriate, a survey to become familiar with activities,
préliminaire sur place destiné à se familiariser avec les risks, and controls; to identify areas for engagement
activités, les risques et les contrôles, à identifier les emphasis; and to invite comments and suggestions from
domaines où la mission devra être approfondie et à inviter les engagement clients. Furthermore, planning extends to
clients de la mission à formuler leurs commentaires et developing work programs; determining how, when, and to
suggestions. De plus, la planification recouvre l'élaboration whom engagement results will be communicated; and
de programmes de travail, la détermination de comment, obtaining approval of the engagement work program.
quand et à qui les résultats de l'audit seront communiqués. Identifying sufficient information to achieve engagement
La planification inclut également l'approbation du programme objectives is done during field work, not planning.
de travail de la mission. La collecte d'informations suffisantes Answer (D) is incorrect because the planning process includes
pour permettre d'atteindre les objectifs de la mission determining how, when, and to whom the engagement results will
s'effectue durant le travail sur place et non pendant la be communicated.
planification.
La réponse (D) est fausse. La planification inclut la détermination
des moyens, du moment et des personnes à qui les résultats de la
mission seront communiqués.

Restitution

© IFACI – CIA1 V.5.1 page 39/73


Acquérir une certification
> Préparation au CIA, partie 1
P1- 66 -- Une mission spécifique dans un domaine à haut risque P1- 66 -- As a particular engagement is being planned in a high-
est en cours de planification, le responsable d’audit interne risk area, the chief audit executive determines that the available
identifie que le personnel disponible n’a pas les compétences staff does not have the requisite skills to perform the assignment.
requises pour conduire la mission. La meilleure chose à faire afin The best course of action consistent with engagement planning
d’être cohérent avec les principes relatifs à la planification des principles is to
missions est de
A. Ne pas réaliser la mission parce que les compétences requises A. Not perform the engagement because the requisite skills are
ne sont pas disponibles. not available.
B. Considérer la mission comme une opportunité de formation et B. Use the engagement as a training opportunity and let the
laisser les auditeurs internes apprendre au fil de la mission. internal auditors learn as the engagement is performed.
C. Envisager de faire appel à des ressources externes afin de C. Consider using external resources to supplement the needed
compléter les savoirs, connaissances, et autres compétences knowledge, skills, and other competencies and complete the
nécessaires et constituer l’équipe. assignment.
D. Réaliser la mission mais limiter le périmètre au regard du D. Perform the engagement but limit the scope in light of the skill
manque de connaissances. deficiency.

La réponse (A) est fausse. Ne pas réaliser la mission est Answer (A) is incorrect because not performing the engagement
inacceptable, particulièrement dans un domaine à haut risque. is unacceptable, especially for a high-risk area.
La réponse (B) est fausse. Les missions doivent être correctement Answer (B) is incorrect because engagements should be properly
supervisées. Le service d’audit interne n’a personne pour prendre supervised. The IAA has no one to provide this supervision.
en charge cette supervision. Answer (C) is correct. In determining the resources needed
La réponse (C) est juste. En identifiant les ressources to perform the engagement, the CAE should consider the
nécessaires à la réalisation de la mission, le RAI doit tenir knowledge, skills, and other competencies of the internal
compte des savoirs, connaissances, et autres compétences auditing staff in selecting internal auditors for the
du personnel de l’audit interne en choisissant les auditeurs engagement. The CAE should consider the use of external
internes. Le RAI doit envisager le recours à des ressources resources when additional knowledge, skills, and other
externes lorsque des savoirs, connaissances, et autres competencies are needed (IPPF).
compétences du personnel sont nécessaires (CRIPP). Answer (D) is incorrect because limiting the scope of the
La réponse (D) est fausse. Limiter le périmètre de la mission ne engagement should be done only when the requisite skills are not
devrait être fait que lorsque les connaissances requises ne sont available even from external resources. If the scope is limited,
pas non plus disponibles en externe. Si le périmètre de la mission management should be informed of the constraint in an interim
est limité, la direction devrait en être informée dans un rapport report.
intermédiaire.

P1- 67 -- Lors de la planification d'une mission d'assurance, un P1- 67 -- In planning an assurance engagement, a survey could
examen préliminaire peut permettre la réalisation des actions assist with all of the following, except
suivantes, sauf
A. Obtenir les commentaires et suggestions des clients de la A. Obtaining engagement client comments and suggestions on
mission sur les problèmes de contrôle. control problems.
B. Obtenir des informations préliminaires sur les contrôles. B. Obtaining preliminary information on controls.
C. Identifier les secteurs importants nécessitant une analyse plus C. Identifying areas for engagement emphasis.
approfondie. D. Evaluating the adequacy and effectiveness of controls.
D. Évaluer le caractère adéquat et l'efficacité des contrôles.
La réponse (A) est fausse. Un examen préliminaire peut permettre Answer (A) is incorrect because a survey could assist with
d'obtenir les commentaires et suggestions des clients de la obtaining client comments and suggestions on control problems.
mission sur les problèmes de contrôle. Answer (B) is incorrect because a survey could assist with
La réponse (B) est fausse. Un examen préliminaire peut permettre obtaining preliminary information on controls.
d'obtenir des informations préliminaires sur les contrôles. Answer (C) is incorrect because a survey could assist with
La réponse (C) est fausse. Un examen préliminaire peut permettre identifying areas for engagement emphasis.
d'identifier les domaines importants nécessitant une analyse plus Answer (D) is correct. A survey is a process for gathering
approfondie. information, without detailed verification, on the activity
La réponse (D) est juste. « Un examen préliminaire est un being examined. A survey may involve discussions with the
procédé permettant de rassembler des informations sur client, documenting key control activities, and identifying
l'activité étudiée, sans vérification détaillée. Un examen significant engagement issues (IPPF). A survey does not help
préliminaire peut entraîner l'utilisation des procédures in evaluating the adequacy and effectiveness of controls
suivantes : les discussions avec le client de la mission, la except to the extent the internal auditor gains familiarity with
documentation des activités de contrôle clé » et the controls. Evaluation requires testing.
l'identification des problèmes importants (CRIPP). Un examen
préliminaire ne permet pas d'évaluer le caractère adéquat et
l'efficacité des contrôles, sauf dans la mesure où l'auditeur
interne se familiarise avec les contrôles. L'évaluation
nécessite la réalisation de tests.

Restitution

© IFACI – CIA1 V.5.1 page 40/73


Acquérir une certification
> Préparation au CIA, partie 1
P1- 68 -- En planifiant une mission, un auditeur interne établit les P1- 68 -- While planning an engagement, an internal auditor
objectifs de la mission pour décrire le travail à effectuer. Laquelle establishes engagement objectives to describe what is to be
des options suivantes est un point clé à prendre en considération accomplished. Which of the following is a key issue to consider in
dans la définition des objectifs de la mission ? developing engagement objectives?
A. Les qualifications des membres de l’équipe d’audit interne qui A. The qualifications of the internal auditing staff selected for the
vont être choisis pour participer à la mission. engagement.
B. Les risques associés aux activités à auditer. B. Risks associated with the activities to be reviewed.
C. Les recommandations faites par les salariés du client de la C. Recommendations of the engagement client's employees.
mission. D. The recipients of the final engagement communication.
D. Les destinataires de la communication finale de la mission
d’audit.
La réponse (A) est fausse. Les objectifs de la mission doivent Answer (A) is incorrect because the engagement objectives
déterminer le choix des membres de l’équipe, et pas le contraire. should regulate the selection of staff members, not vice versa.
La réponse (B) est juste. Les objectifs d’une mission doivent Answer (B) is correct. The engagement's objectives should
porter sur les processus de management des risques, de address the risks, controls, and governance procedures
contrôle et de gouvernement d’entreprise dans la limite associated with the activities under review (IPPF). The
convenue avec le client (CRIPP). L’objectif du processus purpose of the risk assessment during the planning phase is
d’évaluation des risques pendant la phase de planification est to identify significant areas of activity that should be
d’identifier les domaines importants à auditer en tant examined as potential engagement objectives (IPPF).
qu’objectifs potentiels de la mission (CRIPP). Answer (C) is incorrect because internal auditors must set their
La réponse (C) est fausse. Les auditeurs internes doivent établir own objectives. Client input is more useful for defining the
leurs propres objectifs. Les recommandations du client sont plus operating objectives to which the engagement objectives must
utiles dans la définition des objectifs opérationnels auxquels les relate.
objectifs de la mission doivent correspondre. Answer (D) is incorrect because the needs of recipients
La réponse (D) est fausse. Les besoins des destinataires du addressed by the final engagement communication are
rapport de la mission sont déterminés par les objectifs du client qui determined by the engagement client's objectives.
a été audité.

Restitution

© IFACI – CIA1 V.5.1 page 41/73


Acquérir une certification
> Préparation au CIA, partie 1
P1- 69 -- Un auditeur interne doit évaluer le programme P1- 69 -- An internal auditor is assigned to perform an
d'assurance de l'organisation, notamment le caractère approprié engagement to evaluate the organization's insurance program,
de l'approche d'atténuation des risques. L'organisation s'assure including the appropriateness of the approach to minimizing risks.
elle-même contre les sinistres importants et les prestations The organization self-insures against large casualty losses and
maladie destinées à l'ensemble de ses salariés. Il s'agit d'une health benefits provided for all its employees. It is a large national
importante entité nationale employant plus de 15 000 personnes entity with over 15,000 employees located in various parts of the
dans diverses régions du pays. Elle a recours à un organisme country. It uses an outside claims processor to administer its
extérieur de traitement des demandes, qui gère son programme health care program. The organization's medical costs have been
d'assurance maladie. Les coûts médicaux de l'organisation ont rising by approximately 8% per year for the past five years, and
progressé d'environ 8 % par an sur les cinq dernières années, et management is concerned with controlling these costs. The
la direction souhaite maîtriser ces coûts. L'auditeur interne doit internal auditor needs to determine the scope of the proposed
déterminer le champ de l'évaluation proposée de la couverture evaluation of insurance coverage. Which of the following
d'assurance. Laquelle (ou lesquelles) des propositions suivantes statements are true regarding the potential scope of the
est (sont) sont juste(s) ? engagement?
I. Le service d'audit interne doit se concentrer sur le traitement I. The internal audit activity should concentrate on processing that
effectué en interne et non vérifier le bien-fondé du traitement occurs within the organization and not on evaluating the
effectué par le prestataire extérieur. correctness of transactions processing by the health care
II. L'auditeur interne doit interroger la direction avant de processor.
commencer sa mission afin de comprendre (1) ses préoccupations II. The internal auditor should interview management prior to
et (2) les hypothèses implicites et le raisonnement qui sous-tend la beginning the engagement to understand (1) its concerns and (2)
décision de s'auto-assurer. the underlying assumptions made and rationale used when
III. Pour déterminer le champ de la mission, l'auditeur interne doit making the self-insurance decision.
envisager de faire appel à un actuaire afin de mieux comprendre III. The internal auditor should consider engaging an actuarial
les risques en jeu. consultant to better understand the risks involved in order to help
A. I uniquement. determine the scope of the engagement.
B. II uniquement. A. I only.
C. I et II. B. II only.
D. II et III. C. I and II.
D. II and III.

La réponse (A) est fausse. L'auditeur interne doit enquêter sur la Answer (A) is incorrect because the internal auditor needs to
cause de l'augmentation des coûts et sur la qualité des prestations investigate the cause for the increase in costs and the accuracy
de l'organisme de traitement des demandes. L'auditeur interne with which the claims processor is handling claims. The internal
doit, dans le cadre de son activité ordinaire, interroger la direction auditor should routinely interview engagement client management
du client de la mission mais aussi envisager de faire appel à un and should also consider engaging an actuarial consultant.
actuaire. Answer (B) is incorrect because the internal auditor needs to
La réponse (B) est fausse. L'auditeur interne doit enquêter sur la investigate the cause for the increase in costs and the accuracy
cause de l'augmentation des coûts et sur la qualité des prestations with which the claims processor is handling claims. The internal
de l'organisme de traitement des demandes. L'auditeur interne auditor should routinely interview engagement client management
doit, dans le cadre de son activité ordinaire, interroger la direction and should also consider engaging an actuarial consultant.
du client de la mission mais aussi envisager de faire appel à un Answer (C) is incorrect because the internal auditor needs to
actuaire. investigate the cause for the increase in costs and the accuracy
La réponse (C) est fausse. L'auditeur interne doit enquêter sur la with which the claims processor is handling claims. The internal
cause de l'augmentation des coûts et sur la qualité des prestations auditor should routinely interview engagement client management
de l'organisme de traitement des demandes. L'auditeur interne and should also consider engaging an actuarial consultant.
doit, dans le cadre de son activité ordinaire, interroger la direction Answer (D) is correct. One step in planning the engagement
du client de la mission mais aussi envisager de faire appel à un is to define engagement objectives (intended engagement
actuaire. accomplishments) and procedures (means of achieving the
La réponse (D) est juste. L'une des étapes de la panification objectives). Taken together, the objectives and procedures
de la mission consiste à définir les objectifs (réalisations define the scope of the internal auditor's work (PA 2210-1).
attendues de la mission) et les procédures (moyens pour Objectives and procedures are documented in the
atteindre les objectifs). « L'ensemble des objectifs et des engagement work program. Before the work program can be
procédures de la mission définissent le champ d'intervention drafted, however, a preliminary survey is conducted. This
de l'auditeur interne » (CRIPP). Les objectifs et les procédures survey includes, among other steps, discussions with the
sont consignés dans le programme de travail de la mission. engagement client to increase the internal auditor's
Toutefois, avant que le programme de travail ne soit élaboré, familiarity with the activities to be reviewed (IPPF).
un examen préliminaire est mené. Celui-ci comprend, entre Furthermore, if the internal auditing staff does not have the
autres, les discussions avec le client de la mission destinées requisite knowledge, skills, and other competencies to
à permettre à l'auditeur interne de se familiariser avec les perform all or part of the engagement, the CAE should obtain
activités à examiner (CRIPP). En outre, « le responsable de competent advice and assistance (IPPF). However, the
l'audit interne doit obtenir l'avis et l'assistance de personnes outside claims processing function is an integral part of the
qualifiées si les auditeurs internes ne possèdent pas les organization's internal control. Thus, the scope of the
connaissances, le savoir-faire et les autres compétences internal auditor's work should extend to the adequacy and
nécessaires pour s'acquitter de tout ou partie de leur mission effectiveness of internal control over claims processing.
» (CRIPP). Cependant, le prestataire extérieur chargé du
traitement des demandes fait aussi partie du dispositif de
contrôle interne de l'organisation. Le travail de l'auditeur
interne doit donc également porter sur le caractère adéquat et
l'efficacité du contrôle interne en matière de traitement des
demandes.

Restitution

© IFACI – CIA1 V.5.1 page 42/73


Acquérir une certification
> Préparation au CIA, partie 1
P1- 70 -- Dans le cadre du processus de planification, le P1- 70 -- The chief audit executive should establish goals as part
responsable de l'audit interne doit fixer des objectifs au service of the planning process for the internal audit activity. What are the
d'audit interne. Comment ces objectifs se définissent-ils ? traits of internal auditing goals?
A. Mesurables et atteignables. A. Measurable and attainable.
B. Budgétés et approuvés. B. Budgeted and approved.
C. Planifiés et atteignables. C. Planned and attainable.
D. Demandés et approuvés. D. Requested and approved.
La réponse (A) est juste. « Les objectifs de l'audit interne Answer (A) is correct. Goals should be capable of
doivent pouvoir être atteints dans le cadre des plans accomplishment within given operating plans and budgets
opérationnels et des budgets fixés et, dans la mesure du and should be measurable to the extent possible. They
possible, doivent être mesurables. Ils doivent être complétés should be accompanied by measurement criteria and
par des critères de mesure et un calendrier de réalisation » targeted dates of accomplishment (IPPF).
(CRIPP). Answer (B) is incorrect because goals should be attainable within
La réponse (B) est fausse. Les objectifs doivent pouvoir être budget constraints. However, approval is not a trait of goals
atteints dans le cadre des budgets fixés. Cependant, l'approbation themselves.
ne constitue pas une caractéristique des objectifs en tant que tels. Answer (C) is incorrect because goals should be measurable.
La réponse (C) est fausse. Les objectifs doivent être mesurables. Answer (D) is incorrect because goals are not usually requested.
La réponse (D) est fausse. Les objectifs ne sont généralement pas Instead, they are established by the CAE.
demandés. Ils sont en revanche établis par le responsable de
l'audit interne.

P1- 71 -- Lors d’une réunion avec les chefs de missions, le P1- 71 -- At a meeting with engagement managers, the chief audit
directeur d’audit attribue le calendrier des missions d’audit pour executive is allocating the engagement work schedule for next
l’année à venir. Laquelle des méthodes suivantes garantira que year's plan. Which of the following methods will ensure that each
chaque responsable recevra une part équitable aussi bien du manager receives an appropriate share of both the work schedule
calendrier de travail que des ressources allouées ? and internal audit activity resources?
A. Le travail est attribué à chaque responsable à partir d’une A. Work is assigned to each manager based on risk and skill
analyse de risque et de compétence. analysis.
B. Chacun des responsables sélectionne les missions qu’il B. Each of the managers selects the individual assignments
souhaite réaliser, en fonction des départements qu’il affectionne et desired, based on preferences for the area and the management
des managers qui les dirigent. personnel involved.
C. Chaque responsable choisit ses missions en fonction du C. Each manager chooses assignment preferences based on the
nombre total d’heures / homme à sa disposition. total staff hours that are currently available to each manager.
D. La liste complète des missions planifiées est publiée à D. The full list of scheduled engagements is published for the
l’attention de l’ensemble du personnel et leur attribution est faite staff, and work assignments are made based on career interests
en fonction des souhaits de carrière et des voyages que les and travel requirements.
auditeurs souhaitent effectuer.
La réponse (A) est correcte. Les missions doivent être Answer (A) is correct. Engagements should be performed
conduites avec compétence et conscience professionnelle. with proficiency and due professional care. Thus,
Ainsi, la conscience professionnelle doit être à la hauteur de professional care should be commensurate with the
la complexité de la mission, et l’audit interne doit s’assurer complexities of the engagement, and the IAA should assure
que l’équipe d’auditeurs désignée possède collectivement les that the technical proficiency and educational background of
connaissances théoriques et les compétences techniques the personnel assigned are appropriate. A skill analysis of
pour mener à bien la mission. Une analyse de compétences tasks to be performed is therefore necessary. Furthermore,
pour accomplir les tâches à réaliser est ainsi nécessaire. Par matters to be considered in establishing engagement work
ailleurs, l’élaboration du plan d’audit doit tenir compte (1) schedule priorities include (1) updated assessments of risks
d’une évaluation actualisée des risques et de l’efficacité des and effectiveness of risk management and control processes
processus de gestion des risques et de contrôle et (2) des and (2) changes to and capabilities of the staff.
changements au niveau des compétences des salariés. Answer (B) is incorrect because choice based on personal
La réponse (C) est fausse. La sélection basée sur le choix preference does not ensure the exercise of due professional care.
personnel ne garantit pas l’application de la conscience Answer (C) is incorrect because available staff hours do not
professionnelle. correlate with risk or the composite skills necessary for individual
La réponse (C) est fausse. Le nombre d’heures disponible au assignments.
niveau de l’équipe d’audit ne peut être rapproché du risque ou des Answer (D) is incorrect because although career interests and
diverses compétences nécessaires aux missions individuelles. travel requirements are considerations for staffing engagements,
La réponse (D) est incorrecte car bien que les souhaits de carrière these factors do not constitute an objective basis for making
et de voyage sont des considérations à prendre en compte, ces assignments.
facteurs ne constituent pas une base objective d’attribution de
mission.

Restitution

© IFACI – CIA1 V.5.1 page 43/73


Acquérir une certification
> Préparation au CIA, partie 1
P1- 72 – On fait souvent appel à la modélisation des risques ou à P1- 72 -- Risk modeling or risk analysis is often used in
l’analyse des risques au moment de la planification à long terme conjunction with development of long-range engagement work
des missions. L’élément clef qui vient alimenter l’évaluation des schedules. The key input in the evaluation of risk is
risques est constitué
A. Previous engagement results.
A. Des résultats d’une mission précédente. B. Management concerns and preferences.
B. Des préoccupations et des préférences de la direction. C. Specific requirements of professional standards.
C. Des exigences spécifiques aux normes professionnelles. D. Judgment of the internal auditors.
D. De l’opinion des auditeurs internes.
Answer (A) is incorrect because the informed judgment of the
La réponse (A) est fausse. L’opinion avisée de l’auditeur interne internal auditor is still required to assess the magnitude of risk
est toujours nécessaire pour évaluer l’amplitude des risques indicated by previous engagement results.
relevés à l’occasion d’une mission précédente. Answer (B) is incorrect because, to assess the risk posed by
La réponse (B) est fausse. L’évaluation des risques relatifs aux management concerns, informed judgment of the internal auditor
préoccupations de la direction nécessite l’opinion avisée de is required.
l’auditeur interne. Answer (C) is incorrect because professional standards do not
La réponse (C) est fausse. Les normes professionnelles ne specify the basic inputs for a risk analysis.
définissent pas les éléments de base qui viennent alimenter une Answer (D) is correct. Matters to be considered in
analyse des risques. establishing engagement work schedule priorities should
La réponse (D) est juste. Les dates et les résultats des include the dates and results of the last engagement;
missions précédentes, les évaluations actualisées des updated assessments of risks and effectiveness of risk
risques et l’efficacité des processus de gestion des risques et management and control processes; requests by senior
de contrôle, les demandes de la direction générale, du comité management, the audit committee, and the governing body;
d’audit et de l’organe exécutif, les questions en cours current issues relating to organizational governance; major
relatives à la gouvernance de l’organisation, les principaux changes in the enterprise's business, operations, programs,
changements qui se sont produits dans l’entreprise (activité, systems, and controls; opportunities to achieve operating
production, plans, systèmes et contrôles), les opportunités benefits; and changes to and capabilities of the internal
d’atteindre les objectifs opérationnels et les changements au auditing staff (IPPF). Risk is concerned with the probability
niveau des compétences des salariés doivent être pris en rather than the certainty of loss. Assessing the risk of an
compte lorsqu’on définit les priorités du plan d’audit (CRIPP). activity entails analysis of numerous factors, estimation of
Le risque est lié à la probabilité plus qu’à la certitude d’une probabilities and amounts of potential losses, and an
perte. L’évaluation des risques d’une activité conduit à appraisal of the costs and benefits of risk reduction.
l’analyse de nombreux facteurs, à l’estimation de probabilités Consequently, in assessing the magnitude of risk associated
et du montant des pertes éventuelles, et à une appréciation with any factor in a risk model, informed judgment by the
des coûts et des bénéfices de la réduction des risques. De ce internal auditor is required.
fait, en évaluant l’amplitude du risque lié à un facteur dans un
modèle de risque, l’opinion avisée de l’auditeur interne est
nécessaire.

Restitution

© IFACI – CIA1 V.5.1 page 44/73


Acquérir une certification
> Préparation au CIA, partie 1
P1- 73 -- Le responsable de l'audit interne d'un industriel met à P1- 73 -- The chief audit executive of a manufacturer is updating
jour le plan à long terme des missions. Sur une période donnée, il the long-range engagement work schedule. There are several
y a plusieurs missions possibles. Des informations ont été possible assignments that can fill a given time spot. Information
recueillies sur le poids financier des différents domaines possibles on potential dollar exposure and key internal controls has been
et sur les contrôles internes clés. En fonction du risque perçu, il gathered. Based on perceived risk, select the assignment of
convient de sélectionner la mission offrant le plus d'intérêt. greatest merit.
A. Le stock de métaux précieux - valeur comptable : 1 000 000 A. Precious metals inventory -- carrying amount, $1,000,000;
dollars ; entreposage séparé, mais accès non restreint. separately stored, but access not restricted.
B. La caisse des succursales - montant inscrit dans le grand livre : B. Branch office petty cash -- ledger amount, $50,000; ten branch
50 000 dollars ; dix succursales, montants égaux ; le offices, equal amounts; replenishment of accounts requires three
réapprovisionnement des comptes nécessite trois autorisations separate approvals.
distinctes. C. Sales force travel expenses -- budget, $1,000,000; 50 sales
C. Les frais de déplacement des commerciaux - budget : 1 000 people; all expenditures over $25 must be receipted.
000 dollars ; 50 commerciaux ; toutes les dépenses supérieures à D. Expendable tools inventory -- carrying amount, $500,000;
25 dollars nécessitent un reçu. issued by tool crib attendant upon receipt of authorization form.
D. Le stock d'outils non réutilisables - valeur comptable : 500 000
dollars ; mise à disposition par le magasinier à réception du
formulaire d'autorisation.
La réponse (A) est juste. « L'audit interne doit aider Answer (A) is correct. The IAA should assist the organization
l'organisation en identifiant et en évaluant les risques by identifying and evaluating significant exposures to risk
significatifs et contribuer à l'amélioration des systèmes de and contributing to the improvement of risk management and
management des risques et de contrôle » (CRIPP). Parmi les control processes (IPPF). Among the many considerations in
nombreux paramètres à considérer pour évaluer le risque lié judging an item's risk are the ease with which it can be
à un poste figurent la facilité de transformation en liquidités, converted to cash, its accessibility, and its monetary value.
son accessibilité et sa valeur monétaire. Le stock de métaux The precious metals inventory should receive special
précieux doit faire d'objet d'une attention particulière en emphasis because of its high inherent risk. The inventory
raison de son risque intrinsèque élevé. Le stock peut être can be easily converted to cash, access is not restricted, and
facilement transformé en liquidités, son accès n'est pas its monetary value is relatively high.
restreint et sa valeur monétaire est relativement élevée. Answer (B) is incorrect because the monetary exposure of petty
La réponse (B) est fausse. Le risque monétaire lié à la caisse est cash is much smaller than for the other proposed engagements,
nettement inférieur à celui des autres projets de mission, et les and the related controls are very stringent.
contrôles y afférents sont très rigoureux. Answer (C) is incorrect because, although the monetary value of
La réponse (C) est fausse, car même si la valeur monétaire des the sales force travel expense is identical to that of the precious
frais de déplacement des commerciaux est identique à celle du metal inventory, the exposure is divided among 50 people, and
stock des métaux précieux, le risque est réparti entre 50 the receipting requirement provides substantial safety against
personnes et l'obligation de présenter un reçu constitue une bonne false claims.
protection contre les fausses créances. Answer (D) is incorrect because the expendable tools inventory is
La réponse (D) est fausse. Le stock d'outils non réutilisables est subject to adequate control.
soumis à un contrôle adéquat.

P1 - 74 -- Lors d'une mission d'assurance portant sur des opérations de P1 - 74 -- In an assurance engagement of treasury operations, an
trésorerie, un auditeur interne doit examiner tous les points suivants à internal auditor is required to consider all of the following issues
l'exception duquel ? except:
A. Le comité d'audit demande qu'on lui apporte l'assurance que le service de A. The audit committee has requested assurance on the treasury
la trésorerie respecte les nouvelles règles d'utilisation des instruments department's compliance with a new policy on use of financial
financiers. instruments.
B. La direction de la trésorerie n'a mis en place aucune mesure de B. Treasury management has not instituted any risk management
gestion du risque. policies.
C. En raison de la récente cession d'une division, le montant des liquidités et C. Due to the recent sale of a division, the amount of cash and
des titres négociables gérés par le service de la trésorerie s'est accru de 350 %. marketable securities managed by the treasury department has
D. Les auditeurs externes ont indiqué qu'ils avaient eu des difficultés à obtenir increased by 350 percent.
des confirmations de la part de tiers. D. The external auditors have indicated some difficulties in
obtaining account confirmations.

La réponse (A) est fausse. Selon le CRIPP, l'auditeur doit prendre en Answer (A) is incorrect. The IPPF states that the auditor should
considération l'étendue du travail nécessaire pour atteindre les objectifs de la consider the extent of work needed to achieve the engagement's
mission. Il s'agit d'un objectif de mission spécifique. objectives. This is a specific engagement objective.
La réponse (B) est fausse. Selon le CRIPP, l'auditeur doit prendre en Answer (B) is incorrect. The IPPF states that the auditor should
considération la pertinence et l'efficacité des processus de management des consider the adequacy and effectiveness of risk management
risques. processes.
La réponse (C) est fausse. Selon le CRIPP, l'auditeur doit prendre en Answer (C) is incorrect. The IPPF states that the auditor should
considération la complexité relative, la matérialité ou le caractère significatif consider significance and materiality of matters to which assurance
des domaines auxquels sont appliquées les procédures propres aux procedures are applied. This is a significant increase.
missions d'assurance. L'augmentation est ici significative. Answer (D) is correct. This is the responsibility of the external
La réponse (D) est juste. Ce point relève de la responsabilité des auditors and should not change what should be considered
auditeurs externes et ne doit pas changer ce dont doit tenir compte by the internal auditor.
l'auditeur interne.

Restitution

© IFACI – CIA1 V.5.1 page 45/73


Acquérir une certification
> Préparation au CIA, partie 1
P1 - 75 -- Lequel des objectifs de mission possibles suivants permettrait de tester P1 - 75 -- Which of the following possible engagement objectives
l'efficience de l'utilisation des ressources humaines dans une organisation ? would lead to a test of the efficiency of an organization's use of
labor resources?
A. Vérifier que tous les salariés sont payés conformément à l'échelle
syndicale des salaires. A. To determine that all employees are paid in accordance with union
B. Vérifier que les salariés sont affectés à des tâches correspondant à leur wages.
niveau de formation et de compétence. B. To determine that employees are assigned to work situations
C. Vérifier que les performances de la main-d'oeuvre correspondent aux equivalent to their training and skill level.
standards de l'organisation. C. To determine that the quality of performance by labor meets
D. Vérifier que seuls les salariés habilités sont rémunérés. organizational standards.
D. To determine that only authorized employees are paid.

La réponse (A) est fausse. Cet objectif relève d'une mission de conformité et Answer (A) is incorrect because it states an objective of a
ne concerne pas l'efficience. compliance engagement, not one of efficiency.
La réponse (B) est juste. Les auditeurs internes doivent déterminer si Answer (B) is correct. Internal auditors should appraise the
les ressources sont employées de manière rentable et efficiente. economy and efficiency with which resources are employed.
Confier à des salariés des tâches qui ne correspondent pas à leurs Assignment of employees to tasks not commensurate with
compétences peut entraîner des coûts de main-d'oeuvre excessifs their skills may result in excess labor costs (when more
(lorsque des travailleurs plus qualifiés et mieux rémunérés effectuent skilled and more highly paid workers perform jobs for which
des tâches pour lesquelles ils sont trop qualifiés) ou des performances they are overqualified) or in poor performance (when
médiocres (lorsqu'une main-d'oeuvre sous-qualifiée est employée). underqualified labor is used).
La réponse (C) est fausse. Ce facteur concerne l'efficacité et non l'efficience. Answer (C) is incorrect because it concerns effectiveness, not
La réponse (D) est fausse. Ce facteur ne concerne pas l'efficience. efficiency.
Answer (D) is incorrect because it is irrelevant to efficiency.

P1 - 76 -- Laquelle des caractéristiques suivantes fera le plus P1 - 76 -- Which of the following is most likely to be an element of
vraisemblablement partie d'un programme efficace de conformité à la an effective regulatory compliance program?
réglementation ?
A. The internal audit activity should be assigned overall
A. L'audit interne doit être chargé de superviser le programme. responsibility for overseeing the program.
B. Le programme est communiqué aux salariés une seule fois sous un B. The program is communicated to employees in a video format
format vidéo. on a one-time basis.
C. L'organisation doit exploiter des systèmes de suivi et d'audit conçus pour C. The organization should use monitoring and auditing systems
détecter les pratiques délictueuses. reasonably designed to detect criminal conduct.
D. L'organisation doit réunir autant d'informations que possible lorsqu'elle D. The organization should obtain as much information as possible
procède à la vérification des antécédents des salariés. when performing background checks on employees.
La réponse (A) est fausse. Il convient de désigner au sein de l'organisation un Answer (A) is incorrect because specific individual(s) within high-
ou plusieurs collaborateurs de haut niveau chargés de veiller au respect des level personnel of the organization should be assigned overall
normes et des procédures. L'expression “collaborateurs de haut niveau” vise responsibility to oversee regulatory compliance with standards and
les personnes qui jouent un rôle important dans l'organisation ou dans son procedures. High-level personnel of the organization means
processus de décision. individuals who have substantial control of the organization or who
La réponse (B) est fausse. L'efficacité d'un programme de conformité have a substantial role in the making of policy within the
dépendra du mode de présentation adopté. En règle générale, la organization.
communication de type interactif est plus efficace qu'un exposé formel. De Answer (B) is incorrect because the effectiveness of a compliance
même, les programmes présentés par un animateur fonctionnent program will depend upon the ways in which it is communicated to
généralement mieux que les programmes entièrement présentés sous forme employees. Generally, an interactive format works better than a
de vidéo ou de jeu. L'organisation de sessions périodiques est plus efficace lecture. Programs communicated in person tend to work better
qu'une présentation unique. than programs communicated entirely through video or game
La réponse (C) est juste. L'organisation doit prendre des mesures formats. Programs that are periodically repeated work better than
appropriées afin d'assurer le respect de ses normes. À titre d'exemple, one-time presentations.
ces mesures incluent l'utilisation de systèmes de suivi et d'audit Answer (C) is correct. The organization should take
permettant de détecter les infractions commises par les salariés et reasonable steps to achieve compliance with its standards,
agents, ainsi que la mise en place et la présentation aux salariés et e.g., by using monitoring and auditing systems reasonably
agents de l'organisation, d'un système grâce auquel ils peuvent designed to detect criminal conduct by its employees and
indiquer, sans crainte de représailles, les infractions commises par other agents and by having in place and publicizing a
autrui au sein de l'organisation. reporting system whereby employees and other agents could
La réponse (D) est fausse. L'organisation doit veiller à ne pas déléguer de report criminal conduct by others within the organization
pouvoirs discrétionnaires importants à des personnes dont elle sait, de par sa without fear of retribution.
surveillance, ou devrait savoir, qu'elles sont susceptibles de commettre des Answer (D) is incorrect because the organization should use due
actes illicites. En revanche, toutes les précautions doivent être prises pour care not to delegate substantial discretionary authority to
s'assurer que l'organisation ne viole pas les lois protégeant la vie privée des individuals the organization knows, or should know through the
salariés et des candidats. exercise of due diligence, have a propensity to engage in illegal
activities. However, care should be taken to ensure that the
organization does not infringe upon employees' and applicants'
privacy rights under applicable laws.

Restitution

© IFACI – CIA1 V.5.1 page 46/73


Acquérir une certification
> Préparation au CIA, partie 1
[Enoncé #5] Les auditeurs internes d’une institution financière évaluent les [Fact Pattern #5] The internal auditors of a financial institution are
activités d’investissement et de crédit de l’institution. L’année précédente, evaluating the institution's investing and lending activities. During
l’institution a adopté de nouvelles politiques et procédures pour le suivi des the last year, the institution has adopted new policies and
investissements et du portefeuille de crédits. Les auditeurs internes savent procedures for monitoring investments and the loan portfolio. The
que l’institution a investi dans de nouveaux types de produits financiers internal auditors know that the organization has invested in new
pendant l’année et qu’elle utilise beaucoup les produits dérivés pour couvrir types of financial instruments during the year and is heavily
convenablement les risques. involved in the use of financial derivatives to hedge risks
appropriately.

P1 - 77 -- (Voir énoncé 5) Le comité d’audit a exprimé son inquiétude sur le P1 - 77 -- (Refers to Fact Pattern #5) The audit committee has
fait que l’institution financière s’est engagée sur des crédits plus risqués afin expressed concern that the financial institution has been taking on
de réaliser des objectifs de rentabilité à court terme. Laquelle des procédures higher-risk loans in pursuit of short-term profit goals. Which of the
de mission suivantes fournit le moins d’informations pour répondre à cette following engagement procedures provides the least amount of
inquiétude ? information to address this concern?

A. Effectuer une revue analytique des intérêts créditeurs en tant que A. Perform an analytical review of interest income as a percentage
pourcentage de la valeur du portefeuille d’investissements comparé à un of the investment portfolio in comparison with a group of peer
groupe d’institutions financières comparables. financial institutions.
B. Prendre, au hasard, un échantillon de prêts accordés pendant la période B. Take a random sample of loans made during the period and
et comparer le niveau de risque des prêts avec celui d’un échantillon de compare the riskiness of the loans with that of a random sample of
prêts, pris au hasard, accordés il y a deux ans. loans made 2 years ago.
C. Effectuer une revue analytique qui implique la création d’un tableau C. Perform an analytical review that involves developing a chart to
comparatif des intérêts créditeurs tracé sur les dix dernières années. compare interest income plotted over the past 10 years.
D. Effectuer une analyse de régression multiple par série chronologique des D. Develop a multiple-regression time-series analysis of income
intérêts créditeurs sur les dernières cinq années en incluant des facteurs tels over the past 5 years including such factors as interest rate in the
que les taux d’intérêts dans l’économie, l’importance du portefeuille des economy, size of loan portfolio, and dollar amount of new loans
crédits, et la valeur en dollars des nouveaux crédits chaque année. each year.

La réponse (A) est fausse. Les crédits plus risqués doivent générer des intérêts Answer (A) is incorrect because higher-risk loans should generate
plus élevés à court terme comparés à ceux d’autres institutions similaires. Les higher short-term interest income compared with that earned by
crédits qui sont plus risqués fournissent une rémunération plus élevée. comparable institutions. Higher-risk loans have higher yields.
La réponse (B) est fausse. Une comparaison historique du risque des prêts Answer (B) is incorrect because a historical comparison of loan risk
encourus par l’institution répond à l’objectif de la mission. for the institution addresses the engagement objective.
La réponse (C) est juste. Le fait de tracer les fluctuations des intérêts Answer (C) is correct. Plotting the changes in interest income
créditeurs sur les dix dernières années est l’approche la moins utile. over the past 10 years is the least useful procedure. It does
Cela ne prend pas en compte d’autres facteurs importants, tels que not consider other important factors, such as size of the
l’importance du portefeuille, les changements des taux d’intérêts, le portfolio, changes in interest rates, the development of new
développement de nouveaux instruments financiers, le taux d’inflation, financial instruments, the level of inflation, and government
et la réglementation gouvernementale. regulation.
La réponse (D) est fausse. La régression multiple explique la fluctuation d’une Answer (D) is incorrect because multiple regression explains the
variable dépendante (les intérêts créditeurs) imputable à deux variables change in a dependent variable (interest income) attributable to two
indépendantes ou plus. Il permet donc à l’auditeur d’estimer quelle proportion de la or more independent variables. Thus, it allows the internal auditor
fluctuation pourrait être attribuable au niveau de risque des crédits accordés. to estimate how much of the change might be due to a change in
the riskiness of the loans.

P1 - 78 -- Lequel des concepts suivants ne fait pas partie des concepts P1 - 78 -- Which of the following is not an interrelated concept that
interdépendants fondamentaux pour le champ de l'audit interne et le travail is fundamental to the field of internal auditing and the work of
des auditeurs internes ? internal auditors?
A. La gouvernance. A. Governance.
B. L'importance relative. B. Materiality.
C. Le risque. C. Risk.
D. Le contrôle. D. Control.
La réponse (A) est fausse. Il fait partie des concepts interdépendants Answer (A) is incorrect because it is an interrelated concept that is
fondamentaux pour le champ de l'audit interne et le travail des auditeurs internes. fundamental to the field of internal auditing and the work on internal
La réponse (B) est juste. La gouvernance, le risque et le contrôle sont auditors.
des concepts interdépendants fondamentaux pour le champ de l'audit Answer (B) is correct. Governance, risk, and control are
interne et le travail des auditeurs internes. L'importance relative est un interrelated concepts that are fundamental to the field of internal
concept lié à l'audit externe. auditing and the work of internal auditors. Materiality is a concept
La réponse (C) est fausse. Il fait partie des concepts interdépendants related to external auditing.
fondamentaux pour le champ de l'audit interne et le travail des auditeurs Answer (C) is incorrect because it is an interrelated concept that is
internes. fundamental to the field of internal auditing and the work on internal
La réponse (D) est fausse. Il fait partie des concepts interdépendants auditors.
fondamentaux pour le champ de l'audit interne et le travail des auditeurs internes. Answer (D) is incorrect because it is an interrelated concept that is
fundamental to the field of internal auditing and the work on internal
auditors.

Restitution

© IFACI – CIA1 V.5.1 page 47/73


Acquérir une certification
> Préparation au CIA, partie 1
P1 - 79 -- Lorsqu'il évalue l'efficacité et l'efficience de l'utilisation des P1 - 79 -- In evaluating the effectiveness and efficiency with which
ressources, un auditeur interne doit : resources are employed, an internal auditor is responsible for
A. Déterminer dans quelle mesure des critères opérationnels adéquats ont A. Determining the extent to which adequate operating criteria have
été fixés. been established.
B. Vérifier l'existence des actifs. B. Verifying the existence of assets.
C. Examiner la fiabilité des informations opérationnelles. C. Reviewing the reliability of operating information.
D. Vérifier la précision de l'évaluation des actifs. D. Verifying the accuracy of asset valuation.
La réponse (A) est juste. L'audit interne évalue les contrôles “portant Answer (A) is correct. The internal audit activity evaluates the
sur le gouvernement d'entreprise, les opérations et les systèmes controls encompassing governance, operations, and
d'information de l'organisation. Cette évaluation doit porter sur information systems. This evaluation includes the
l'efficacité et l'efficience des opérations” (CRIPP). En outre, “les effectiveness and efficiency of operations (IPPF). Moreover,
auditeurs internes doivent déterminer dans quelle mesure le the internal auditors should "ascertain the extent to which
management a défini des critères adéquats pour apprécier si les management has established adequate criteria to determine
objectifs et les buts ont été atteints” (CRIPP). Ils doivent également whether objectives and goals have been accomplished"
“évaluer les cibles et attentes opérationnelles établies et déterminer si (IPPF). They should also "evaluate the established operating
ces normes opérationnelles sont acceptables et respectées” (CRIPP). targets and expectations and should determine whether those
La réponse (B) est fausse. La vérification de l'existence concerne la operating standards are acceptable and are being met" (IPPF).
protection des actifs. Answer (B) is incorrect because verifying existence relates to the
La réponse (C) est fausse. La fiabilité de l'information opérationnelle et la précision safeguarding of assets.
de l'évaluation des actifs concernent la fiabilité et l'intégrité de l'information. Answer (C) is incorrect because the reliability of operating
La réponse (D) est fausse. La fiabilité de l'information opérationnelle et information and the accuracy of asset valuation concern the
la précision de l'évaluation des actifs concernent la fiabilité et l'intégrité reliability and integrity of information.
de l'information. Answer (D) is incorrect because the reliability of operating
information and the accuracy of asset valuation concern the
reliability and integrity of information.

P1 - 80 -- Des informations sur les billets d'avion, notamment le tarif, la P1 - 80 -- Specific airline ticket information, including fare, class,
classe, la date d'achat et les options tarifaires les plus économiques purchase date, and lowest available fare options, as prescribed in
disponibles, exigées par la politique de l'organisation relative aux the organization's travel policy, is obtained and reported to
déplacements, sont collectées et communiquées à la direction du department management when employees purchase airline tickets
département lorsque les salariés achètent des billets d'avion auprès de from the organization's authorized travel agency. Such a report
l'agence de voyage agréée par l'organisation. De tels éléments donnent des provides information for
informations :
A. Sur la qualité de ce qui est fait par rapport à la politique de l'organisation A. Quality of performance in relation to the organization's travel
sur les déplacements. policy.
B. Pour identifier les coûts nécessaires au traitement des données relatives B. Identifying costs necessary to process employee business
aux frais de déplacement des salariés. expense report data.
C. Pour comparer les dépenses budgétées et réelles au niveau du service C. Departmental budget-to-actual comparisons.
concerné. D. Supporting employer's business expense deductions.
D. Pour justifier les frais de déplacement professionnels de l'employeur.
La réponse (A) est juste. Le retour d'information fait partie du cycle de Answer (A) is correct. Feedback is a part of the internal
contrôle interne. Il forme une base de comparaison entre les control cycle. It provides a basis for comparing actual
performances réelles (achats de billets en fonction des options performance (purchases of tickets given the available
disponibles) et les normes (la politique de l'organisation). options) with standards (organizational policy).
La réponse (B) est fausse. Ces informations sont préliminaires. Les salariés Answer (B) is incorrect because this ticket information is
peuvent modifier leurs billets et leurs itinéraires avant leur déplacement. preliminary; employees may change tickets and routings prior to
La réponse (C) est fausse. La comparaison des dépenses budgétées et their trip.
réelles au niveau du service ne reflète pas forcément les coûts réels. Answer (C) is incorrect because departmental budget-to-actual
La réponse (D) est fausse. Les justificatifs pour les frais de déplacement ne comparisons do not necessarily reflect the actual costs ultimately
reflètent pas forcément les coûts réels. incurred.
Answer (D) is incorrect because supporting expense deductions
may not necessarily reflect actual costs.

Restitution

© IFACI – CIA1 V.5.1 page 48/73


Acquérir une certification
> Préparation au CIA, partie 1
P1 - 81 -- Lorsqu'il réceptionne des marchandises achetées, le personnel du P1 - 81 -- Upon receipt of purchased goods, receiving department
service de la réception compare la quantité reçue à celle figurant sur le personnel match the quantity received with the packing slip
bordereau d'emballage et note le prix de vente au détail sur la base d'un quantity and mark the retail price on the goods based on a master
barème de prix. Le bordereau ainsi annoté est transmis au service de price list. The annotated packing slip is then forwarded to inventory
contrôle des stocks et les marchandises sont automatiquement envoyées au control and goods are automatically moved to the retail sales area.
service des ventes au détail. Le point fort le plus significatif du contrôle de The most significant control strength of this activity is
cette activité consiste à :
A. Indiquer immédiatement le prix sur les marchandises en vue de leur vente A. Immediately pricing goods for retail sale.
au détail. B. Matching quantity received with the packing slip.
B. Comparer la quantité reçue au chiffre figurant sur le bordereau. C. Using a master price list for marking the sale price.
C. Utiliser un barème de prix pour indiquer le prix de vente. D. Automatically moving goods to the retail sales area.
D. Envoyer automatiquement les marchandises au service des ventes au
détail.
La réponse (A) est fausse. La rapidité de traitement n'est pas aussi Answer (A) is incorrect because timing is not as important as the
importante que l'exactitude du prix. accuracy of prices.
La réponse (B) est fausse. Comparer la quantité reçue au bordereau ne Answer (B) is incorrect because matching quantity received with
garantit pas que la quantité commandée a été reçue. the packing slip does not ensure receipt of the quantity ordered.
La réponse (C) est juste. L'utilisation d'un barème de prix garantit que le Answer (C) is correct. Use of the master price list assures that
bon prix de détail est indiqué. the correct retail price is marked.
La réponse (D) est fausse. Les marchandises peuvent ou non être Answer (D) is incorrect because goods may or may not be needed
nécessaires au service des ventes au détail. in retail sales.

P1 - 82 -- Lequel des énoncés suivants décrit le mieux la finalité de l'examen, P1 - 82 -- Which of the following best describes an internal auditor's
par un auditeur interne, des processus de gestion du risque, de contrôle et de purpose in reviewing the organization's existing risk management,
gouvernance de l'organisation ? control, and governance processes?
A. Aider à déterminer la nature, la durée et l'étendue des contrôles A. To help determine the nature, timing, and extent of tests
nécessaires à la réalisation des objectifs de la mission. necessary to achieve engagement objectives.
B. Vérifier que les déficiences du système de contrôle interne ont été corrigées. B. To ensure that weaknesses in the internal control system are
C. Apporter une assurance raisonnable que ces processus permettront la corrected.
réalisation efficiente et économique des objectifs et buts de l'organisation. C. To provide reasonable assurance that the processes will enable
D. Déterminer si ces processus permettent des écritures comptables the organization's objectives and goals to be met efficiently and
correctes et l'élaboration d'états financiers justes. economically.
D. To determine whether the processes ensure that the accounting
records are correct and that financial statements are fairly stated.

La réponse (A) est fausse. C'est la finalité de la planification de l'audit. Answer (A) is incorrect. This is a purpose of audit planning.
La réponse (B) est fausse. C'est à la direction, et non à l'auditeur interne, de Answer (B) is incorrect. Correcting control weaknesses is a
corriger les déficiences du contrôle. function of management, not of the internal auditor.
La réponse (C) est juste. C'est la finalité énoncée dans le CRIPP. Answer (C) is correct. This is the purpose stated in IPPF.
La réponse (D) est fausse. C'est un objectif fondamental du point de vue de Answer (D) is incorrect. This is a basic objective from a financial
l'audit et de la certification des comptes, mais il ne décrit pas la finalité globale accounting and auditing perspective, but is not broad enough to
de l'examen mené par l'auditeur interne. cover the internal auditor's entire purpose for review.

Restitution

© IFACI – CIA1 V.5.1 page 49/73


Acquérir une certification
> Préparation au CIA, partie 1
P1 - 83 -- Pendant un examen des marchés passés par sa société, un P1 - 83 -- During a review of contracts, a chief audit executive (CAE)
responsable de l'audit interne soupçonne qu'un fournisseur a bénéficié d'un suspects that a supplier was given an unfair advantage in bidding on
avantage déloyal lors d'un processus d'appel d'offres. Après avoir appris que le a contract. After learning that the chief executive officer (CEO) of the
directeur général (DG) de sa société est aussi membre du conseil company is a member of the supplier's board of directors, how
d'administration du fournisseur, que doit faire le responsable de l'audit interne ? should the CAE proceed?
A. Présenter un projet de rapport à la direction, mais pas au directeur général. A. Submit a draft report to senior management, excluding the CEO.
B. Contacter les auditeurs externes de l'organisation pour leur demander de l'aide. B. Contact the organization's external auditors for assistance.
C. Se procurer les pièces justificatives et présenter les constats au président C. Obtain supporting documentation and present the finding to the
du comité d'audit. chairperson of the audit committee.
D. En informer immédiatement le conseil d'administration. D. Immediately notify the board of directors.

La réponse (A) est fausse. Le directeur général est un membre de la Answer (A) is incorrect. The chief executive officer (CEO) is a
direction. D'autres membres de la direction peuvent recevoir la version member of senior management. Other members of senior
définitive du rapport qui a été révisé et approuvé par le conseiller juridique. management may receive a final report that has been reviewed
La réponse (B) est fausse. Les auditeurs externes ne doivent pas être and approved by legal counsel.
contactés. Ils peuvent recevoir la version définitive du rapport qui a été révisé Answer (B) is incorrect. External auditors should not be contacted.
et approuvé par le conseiller juridique. External auditors may be given a final report that has been
La réponse (C) est juste. Il convient de présenter un projet du rapport reviewed and approved by legal counsel.
faisant état de la fraude ou du conflit d'intérêts au président du comité Answer (C) is correct. A draft of the proposed report on fraud
d'audit, étant donné la position du DG au sein de l'entreprise. or conflict-of-interest situations should be submitted to the
La réponse (D) est fausse. Il convient de disposer des pièces justificatives chairman of the audit committee as a next step in light of the
avant d'informer le comité d'audit ou le conseil. CEO's position in the company.
Answer (D) is incorrect. Supporting documentation would be
necessary before informing the audit committee or the board.

P1 - 84 -- La direction et le conseil d'administration sont chargés de mettre en P1 - 84 -- Management and the board of directors are responsible for
place des mesures correctives formulées à la suite des observations et des following up on observations and recommendations made by the
recommandations des auditeurs externes. Quel est le rôle de l'audit interne external auditors. What role, if any, should the internal audit activity
dans ce processus, le cas échéant ? have in this process?
A. L'audit interne ne joue aucun rôle dans ce processus pour des impératifs A. The internal audit activity should have no role in this process in
d'indépendance. order to ensure independence.
B. L'audit interne ne doit y prendre part que si son responsable dispose de B. The internal audit activity should only become involved if the
preuves suffisantes d'une absence de mesures correctives. chief audit executive has sufficient evidence that the follow-up is
C. L'audit interne doit instaurer un processus de suivi afin de vérifier le bien- not occurring.
fondé et l'efficacité des mesures correctives mises en place par la direction. C. The internal audit activity should establish a monitoring process
D. L'audit interne ne doit y prendre part uniquement si elle y est to review the adequacy and effectiveness of management's follow-
expressément conviée par la direction ou le conseil d'administration. up actions.
D. The internal audit activity should become involved only if
specifically requested by management or the board of directors.

Answer (A) is incorrect. See answer "c".


La réponse (A) est fausse. Voir réponse C. Answer (B) is incorrect. See answer "c".
La réponse (B) est fausse. Voir réponse C. Answer (C) is correct. A chief audit executive should establish
La réponse (C) est juste. Un responsable de l'audit interne doit établir a follow-up process to monitor the adequacy, effectiveness,
un processus de suivi afin de contrôler le bien-fondé, l'efficacité et la and timeliness of actions taken by management on reported
promptitude des mesures prises par la direction en réaction à des engagement observations and recommendations, including
observations et des recommandations formulées dans le rapport de those made by the external auditors and others.
mission, y compris par les auditeurs externes, notamment. Answer (D) is incorrect. See answer "c".
La réponse (D) est fausse. Voir réponse C.

Restitution

© IFACI – CIA1 V.5.1 page 50/73


Acquérir une certification
> Préparation au CIA, partie 1
P1 - 85 -- L’audit interne (AI) d’une organisation est partie intégrante des P1 - 85 -- The internal audit activity of an organization is an integral
processus de gestion des risques, de contrôle et de gouvernement part of the organization's risk management, control, and
d’entreprise de l’organisation parce qu’il évalue et contribue à leur governance processes because it evaluates and contributes to the
amélioration. Choisissez le type de contrôle exercé quand l’AI conduit une improvement of those processes. Select the type of control
revue d’un développement de systèmes. provided when the IAA conducts a systems development review.
A. Le contrôle rétroactif (feedback control). A. Feedback control.
B. Les plans stratégiques. B. Strategic plans.
C. Les politiques et procédures. C. Policies and procedures.
D. Le contrôle par anticipation (feedforward control). D. Feedforward control.
La réponse (A) est fausse. Un contrôle rétroactif fournit une information sur Answer (A) is incorrect because a feedback control provides
les résultats d’une activité terminée. information on the results of a completed activity.
La réponse (B) est fausse. Les plans stratégiques sont développés par la Answer (B) is incorrect because strategic plans are developed by
direction générale afin de fournir une ligne de conduite à long terme pour senior management to provide long-range guidance for the
l’organisation. organization.
La réponse (C) est fausse. Les politiques et les procédures sont développées Answer (C) is incorrect because policies and procedures are
par la direction. Elles représentent le sous-système le plus fondamental d’une developed by management. They are the most basic control
organisation. subsystem of an organization.
La réponse (D) est juste. La régulation par l’amont fournit une information Answer (D) is correct. A feedforward control provides
sur des problèmes potentiels pour qu’une action corrective puisse être information on potential problems so that corrective action
déclenchée par anticipation, plutôt qu'à la suite, d’un problème. can be taken in anticipation, rather than as a result, of a
problem.

P1 - 86 -- Lequel des processus suivants constitue un contrôle opérationnel P1 - 86 -- Which of the following is an operating control relating to
de la fonction de gestion exercée par la direction ? management's directing function?
A. Informer suffisamment à l'avance le service des achats des futurs besoins A. Informing purchasing personnel of the future need for long-lead-
en produits nécessitant un long délai d'approvisionnement. time products in ample time.
B. Présenter au service des achats des comptes rendus élaborés B. Supplying buyers with timely, accurate, and useful reports on
rapidement, exacts et utiles sur les produits reçus, acceptés ou rejetés. products received, accepted, or rejected.
C. Définir des procédures formelles pour sélectionner les fournisseurs potentiels. C. Prescribing formal procedures for selecting potential suppliers.
D. Fixer des objectifs mesurables pour ce service. D. Establishing measurable goals for the department.
La réponse (A) est juste. La gestion est la fonction consistant à motiver Answer (A) is correct. Directing is the function of motivating
les ressources humaines d'une organisation afin qu'elles contribuent people in an organization to contribute effectively and
avec efficacité et efficience à la réalisation des objectifs et buts de efficiently to the achievement of enterprise objectives and
l'entreprise (le CRIPP donne une définition plus large de la gestion). goals. (IPPF provides a broader definition of directing.)
Avertir rapidement le personnel des changements au niveau de leurs Providing adequate notice to employees of changes in their
activités constitue une facette d'un bon leadership. duties is a facet of proper leadership.
La réponse (B) est fausse. Le retour d'information rapide est une Answer (B) is incorrect because providing timely feedback is a control
fonction de contrôle. function.
La réponse (C) est fausse. La définition de procédures formelles pour Answer (C) is incorrect because prescribing formal procedures for
sélectionner des fournisseurs potentiels est une fonction du processus de selecting potential suppliers is a function of the planning process.
planification. Answer (D) is incorrect because establishing measurable goals for
La réponse (D) est fausse. La fixation d'objectifs mesurables pour le service the department is a function of the planning process.
concerné est une fonction du processus de planification.

Restitution

© IFACI – CIA1 V.5.1 page 51/73


Acquérir une certification
> Préparation au CIA, partie 1
P1 - 87 -- Le stock d'un fabricant comporte un investissement significatif dans P1 - 87 -- A manufacturer's inventory includes a significant
des métaux précieux. L'examen, par les auditeurs internes, des contrôles investment in precious metals. The internal auditors' review of
internes mis en place par la direction pour ces éléments comporte le plus management's system of internal controls over these items most
probablement : likely includes
I. Des procédures d'examen, pour faire en sorte que la valeur des I. Reviewing procedures to ensure that the value of the materials
matériaux soit correctement indiquée dans le bilan. is properly stated on the balance sheet.
II. L'examen des formulaires d'acquisition des matières afin de vérifier qu'ils II. Reviewing materials acquisition forms for approvals and tracing
sont dûment validés et la comparaison des formulaires de sortie aux release forms to perpetual inventory records to verify that
fichiers d'inventaire permanent, afin de vérifier que les éléments qui inventory is issued upon proper authorization.
sortent du stock font l'objet d'une autorisation en bonne et due forme. III. Observing inventory transactions to ascertain if materials thefts
III. L'observation des opérations sur le stock, pour s'assurer que des are occurring.
matières n'ont pas été dérobées. IV. Reviewing the manufacturing department's system for
IV. L'examen du système mis en place par le service de production, pour comparing the usage of these metals with standards.
comparer l'utilisation de ces métaux aux normes établies par l'entreprise.
A. III and IV.
A. III et IV. B. I and IV.
B. I et IV. C. I only.
C. I uniquement. D. II and III.
D. II et III.
La réponse (A) est fausse. L'observation des opérations sur le stock n'est pas Answer (A) is incorrect because observing inventory transactions is
susceptible de permettre d'identifier des vols de matières. not likely to identify materials thefts.
La réponse (B) est juste. Un auditeur interne doit procéder à des tests Answer (B) is correct. An internal auditor must conduct
de corroboration afin de vérifier l'évaluation du stock telle que substantive tests to verify the reported inventory valuation,
rapportée, par exemple en observant les prélèvements effectués sur le for example, by observing inventory taking, considering the
stock, en analysant la qualité du stock disponible et en vérifiant la quality of inventory on hand, and testing inventory pricing. In
valorisation du stock. Dans le cas de métaux précieux, les prix du the case of precious metals, market prices may be volatile.
marché peuvent être volatiles. En outre, la comparaison des résultats Furthermore, a fundamental component of the control loop is
réels aux normes (montants budgétés, par exemple) forme un comparison of actual results with standards (e.g., budgeted
composant fondamental de la boucle de contrôle. amounts).
La réponse (C) est fausse. L'auditeur interne doit étudier les procédures de Answer (C) is incorrect because the internal auditor should review
comparaison de l'utilisation du stock réelle et budgétée. procedures for comparing actual and budgeted usage.
La réponse (D) est fausse. La comparaison des formulaires de sortie aux Answer (D) is incorrect because tracing inventory issuance forms
comptes de stock ne garantit pas une autorisation en bonne et due forme. En to inventory records does not ensure proper authorizations.
revanche, les formulaires de sortie doivent être comparés aux bons de Instead, the inventory issuance forms should be compared with
réquisition des matières validés. authorized materials requisitions.

P1 - 88 -- Le champ d'une mission d'assurance doit tenir compte : P1 - 88 -- The scope of an internal auditing assurance engagement
should include consideration of
A. Uniquement des systèmes et documents sous le contrôle du client de la A. Only those systems and records under the control of the
mission. engagement client.
B. Des biens pertinents sous le contrôle de tiers. B. Relevant physical properties under third-party control.
C. Des observations, conclusions et recommandations de la mission. C. Engagement observations, conclusions, and recommendations.
D. Des communications finales de la mission. D. Final engagement communications.

La réponse (A) est fausse. Les systèmes, les documents, le personnel et les Answer (A) is incorrect because systems, records, personnel, and
biens physiques sous le contrôle de tiers doivent être pris en compte s'ils sont physical properties under third-party control should be considered if
pertinents. relevant.
La réponse (B) est juste. Le champ de la mission doit couvrir les Answer (B) is correct. The scope of the engagement should
systèmes, les documents, le personnel et les biens concernés, y include consideration of relevant systems, records,
compris ceux qui se trouvent sous le contrôle de tiers (CRIPP). Les personnel, and physical properties, including those under the
résultats de la mission rapportés dans les communications finales de control of third parties (IPPF). Engagement results reported in
la mission émanent de l'exécution des procédures et de la réalisation final engagement communications follow from performing the
des objectifs qui définissent le champ de la mission. procedures and achieving the objectives that define the
La réponse (C) est fausse. Les résultats de la mission rapportés dans les engagement scope.
communications finales de la mission émanent de l'exécution des procédures Answer (C) is incorrect because engagement results reported in
et de la réalisation des objectifs qui définissent le champ de la mission. final engagement communications follow from performing the
La réponse (D) est fausse. Les résultats de la mission rapportés dans les procedures and achieving the objectives that define the
communications finales de la mission émanent de l'exécution des procédures engagement scope.
et de la réalisation des objectifs qui définissent le champ de la mission. Answer (D) is incorrect because engagement results reported in
final engagement communications follow from performing the
procedures and achieving the objectives that define the
engagement scope.

Restitution

© IFACI – CIA1 V.5.1 page 52/73


Acquérir une certification
> Préparation au CIA, partie 1
P1 - 89 -- Les vérifications effectuées au cours du programme de travail de la P1 - 89 -- Engagement work programs testing controls should
mission doivent :
A. Etre adaptées à chaque opération évaluée. A. Be tailored for each operation evaluated.
B. Etre standardisées pour correspondre à toutes les situations sans tenir B. Be generalized to fit all situations without regard to departmental
compte de la structure départementale. lines.
C. Etre standardisées de manière à pouvoir être utilisées sur tous les sites C. Be generalized so as to be usable at all locations of a particular
d'un département donné. department.
D. Limiter une coûteuse duplication des efforts en veillant à ce que tous les D. Reduce costly duplication of effort by ensuring that every aspect
aspects d'une opération soient examinés. of an operation is examined.

La réponse (A) est juste. Le programme de la mission doit documenter Answer (A) is correct. Work programs document procedures
les procédures permettant à l'auditeur interne de collecter, analyser, for collecting, analyzing, interpreting, and documenting
interpréter et documenter les informations pendant la mission ; définir information; state engagement objectives; set forth the scope
les objectifs de la mission ; établir le champ d'intervention et le degré and degree of testing needed to achieve objectives in each
d'approfondissement nécessaire des tests pour atteindre les objectifs phase of the engagement; identify technical aspects, risks,
de chaque phase de la mission ; identifier les aspects techniques, les processes, and transactions to be examined; state the nature
risques, les procédés et les transactions qui doivent être audités ; and extent of testing required, and are prepared before work
établir la nature et l'étendue nécessaires des tests ; et être préparé begins, with appropriate modification during the engagement
avant le démarrage de la mission, et modifié, si nécessaire, au cours de (IPPF). However, a work program must be adapted to the
la mission (CRIPP). Cependant, le programme de la mission doit être specific needs of the engagement after the internal auditor
adapté aux besoins spécifiques de cette dernière une fois que establishes the engagement objectives and scope and
l'auditeur interne a fixé les objectifs et le champ de la mission et determines the resources required.
déterminé les ressources requises. Answer (B) is incorrect because a generalized program not allows
La réponse (B) est fausse. Un programme standard ne tient pas compte des for variations resulting from changing circumstances and varied
variations résultant de l'évolution des circonstances et de l'environnement. conditions.
La réponse (C) est fausse. Un programme standard ne peut tenir compte des Answer (C) is incorrect because a generalized program cannot
variations des circonstances et des environnements différents. consider variations in circumstances and conditions.
La réponse (D) est fausse, car tous les aspects d'une opération n'ont pas Answer (D) is incorrect because every aspect of an operation need
besoin d'être examinés, mais uniquement ceux susceptibles de dissimuler not be examined, only those aspects likely to conceal problems
des problèmes et des difficultés. and difficulties.

P1 - 90 -- Inclure un budget temps dans le programme de travail d'une P1 - 90 -- The purpose of including a time budget in an
mission vise à : engagement work program is to
A. Procurer un moyen objectif d'évaluation de la compétence de A. Provide an objective means of evaluating the internal auditor's
l'auditeur interne. competence.
B. Garantir la réalisation de la mission dans les délais. B. Assure timely completion of the engagement.
C. Se doter d'un moyen de contrôle et d'évaluation de l'avancement de la mission. C. Provide a means of controlling and evaluating the progress of
D. Limiter le champ de la mission. the engagement.
D. Restrict the scope of the engagement.

La réponse (A) est fausse. De nombreux facteurs autres que la compétence Answer (A) is incorrect because whether an internal auditor
professionnelle influent sur la capacité d'un auditeur interne à ne pas remains within the time budget is affected by many factors other
dépasser le budget temps. than professional competence.
La réponse (B) est fausse. L'établissement d'un budget ne garantit pas la Answer (B) is incorrect because the establishment of a budget
réalisation du travail dans les délais. cannot assure that work will be completed on a timely basis.
La réponse (C) est juste. Les auditeurs internes doivent élaborer un Answer (C) is correct. Internal auditors should develop and
programme de travail permettant d'atteindre les objectifs de la mission record work programs to achieve the engagement objectives
(CRIPP). Ce programme doit répertorier les procédures à suivre (IPPF). The work program lists the procedures to be followed
pendant la mission. Il constitue donc un outil utile pour la during the engagement. Accordingly, a work program is a
programmation et le contrôle (supervision) de la mission. La useful tool for scheduling and controlling (supervising) the
supervision consiste notamment à vérifier que le programme approuvé engagement. Supervision includes, among other things,
est correctement exécuté sous réserve des modifications justifiées et determining that the approved work program is carried out
autorisées. De plus, la supervision inclut le suivi des dépenses et du unless changes are justified and authorized. Moreover,
temps passé (CRIPP). À cette fin, un budget temps est nécessaire pour supervision extends to time and expense control (IPPF). For
l'évaluation et le contrôle de l'avancement de la mission. Il permet de this purpose, a time budget is necessary to evaluate and
comparer le temps réel passé sur une procédure au temps alloué. control the progress of the engagement. It permits
La réponse (D) est fausse. Un budget temps n'a pas pour finalité de limiter le comparison of the actual time spent on a procedure with its
champ de la mission. allotted time.
Answer (D) is incorrect because a time budget is not intended to
limit the scope of the engagement.

Restitution

© IFACI – CIA1 V.5.1 page 53/73


Acquérir une certification
> Préparation au CIA, partie 1
P1 - 91 -- Afin de maîtriser les projets et d'éviter de dépasser le budget P1 - 91 -- As a means of controlling projects and avoiding time-
temps, les décisions portant sur la révision du budget temps alloué à une budget overruns, decisions to revise time budgets for an
mission donnée doivent normalement être prises : engagement should normally be made
A. Immédiatement après l'examen préliminaire. A. Immediately after the survey.
B. Lorsqu’un risque significatif a été démontré. B. When a significant risk exposure has been substantiated.
C. Lorsqu'un personnel inexpérimenté est affecté à une tâche. C. When inexperienced staff are assigned to an engagement.
D. Immédiatement après l'approfondissement des tests visant à établir la D. Immediately after expanding tests to establish reliability of
fiabilité des observations. observations.
La réponse (A) est juste. Un examen est un procédé permettant de Answer (A) is correct. A survey is a process for gathering
rassembler des informations sur l'activité étudiée, sans vérification information, without detailed verification, on the activity being
détaillée. Si nécessaire, un examen préliminaire peut être réalisé pour examined. If appropriate, a survey should be conducted to
se familiariser avec les activités, les risques et les contrôles, pour become familiar with the activities, risks, and controls; to
identifier les domaines où la mission devra être approfondie et pour identify areas for engagement emphasis; and to invite
inviter les clients de la mission à fournir leurs commentaires et comments and suggestions from engagement clients (IPPF).
suggestions (CRIPP). À l'issue de cet examen, des activités autres ou This survey may lead to a determination that activities other
supplémentaires par rapport à celles prévues dans le plan d'audit à than or in addition to those contemplated by the long-range
long terme peuvent s'avérer nécessaires. Par conséquent, la révision engagement work schedule are necessary. Consequently,
du budget temps peut alors être indiquée. revision of the time budget may then be indicated.
La réponse (B) est fausse. Lorsqu'une exposition au risque a été démontrée, Answer (B) is incorrect because, when a risk exposure has been
aucun travail supplémentaire n'est requis. substantiated, no further engagement work is required.
La réponse (C) est fausse. L'affectation de personnel inexpérimenté ne doit Answer (C) is incorrect because the assignment of inexperienced
avoir aucune incidence sur la décision de revoir le budget temps. staff should have no effect on the decision to revise the time
budget.
La réponse (D) est fausse. Les tests approfondis ne doivent avoir
aucune incidence sur le budget temps, car suffisamment de Answer (D) is incorrect because expanded tests should have no
ressources ont déjà été allouées. effect on the time budget; the budget would have already been
expanded as necessary.

Restitution

© IFACI – CIA1 V.5.1 page 54/73


Acquérir une certification
> Préparation au CIA, partie 1
P1 - 92 -- L'audit interne vient d'achever une mission qui consistait à évaluer P1 - 92 -- The internal audit activity has just completed an
le traitement des prêts et les soldes des prêts commerciaux pour le compte engagement to evaluate loan processing and commercial loan
d'un établissement financier. Voici quelques extraits des papiers de travail account balances for a financial institution. Following are a few
indiquant des observations potentielles pour le rapport de mission : excerpts from their working papers indicating potential engagement
1. Nous avons pris un échantillon statistique de 100 demandes de prêts et observations:
observé que seuls 85 prêts avaient été accordés. 1. We took a statistical sample of 100 loan applications and
2. Sur les 85 prêts accordés, nous avons observé que quatre auraient dû determined that only 85 loans were granted.
être vérifiés et approuvés par la commission chargée des prêts, ce qui n’a 2. Of the 85 loans granted, we noted that four loans should have
pas été fait. La politique de l’organisation affirme que tous les prêts doivent been reviewed and approved by the loan committee, but were
être approuvés par la commission avant le déblocage des fonds. not. Organization policy states that all loans must be approved
Cependant, chacun des quatre prêts a été approuvé par le vice-président. by the committee prior to funding. Each of the four loans,
Nous en avons discuté avec le vice-président, qui a indiqué qu’il s’agissait however, was approved by the vice president. The matter was
d’un cas dans lequel un nouveau client avait mis la banque en discussed with the vice president, who indicated it was a
concurrence et qu’il était dans l’intérêt de l’établissement d’agir vite et de competitive loan situation to a new customer and in the best
nouer une relation solide avec un client prometteur. Tous les autres prêts interests of the financial institution to expedite the loan and
ont reçu l’approbation formelle de la commission des prêts. establish a firm relationship with a growing customer. All of the
3. Sur les 81 prêts approuvés par la commission des prêts, nous en avons trouvé other loans were formally approved by the loan committee.
sept pour lesquels le montant réellement prêté dépassait le montant approuvé. 3. Of the 81 loans approved by the loan committee, we found seven in
4. Nous avons noté trois cas où des prêts ont été accordés à des groupes which the actual amount lent exceeded the approved amount.
d’organisations liées sans analyse du montant total des prêts concédés à 4. We noted three instances in which loans were made to related
l’entité qui exerce le contrôle. La loi peut imposer des restrictions sur le groups of organizations without an analysis of the total amount
montant des prêts qui peuvent être accordés à une telle entité. of loans made to the controlling entity. There may be statutory
5. Sur les 81 prêts approuvés par la commission chargée des prêts, nous en limitations on the amount of loans that can be made to any
avons trouvé 14 qui étaient accompagnés d’une documentation insuffisante individual controlling entity.
ou qui n’ont pas été reçus par la commission dans les délais avant la réunion 5. Of the 81 loans approved by the loan committee, we found that
de cette dernière. Le sondage statistique a été effectué avec un niveau de 14 contained either insufficient documentation or were not
confiance de 95 % et un sondage d’attribut avec une limite d’erreur tolérable received by the committee in a timely fashion in advance of their
de 4 %. On peut supposer que le plan d’échantillonnage a été mis en oeuvre meeting. The statistical sample was taken with a 95%
correctement. confidence level using attribute sampling with a tolerable error
limit of 4%. You may assume that the sampling plan was
implemented correctly.
Concernant le point 3, laquelle des actions suivantes serait inappropriée de la Regarding item 3, which of the following actions would be
part de l’auditeur ? inappropriate on the part of the auditor?
A. Examiner les prêts afin de déterminer si un schéma se dégage des prêts A. Examine the loans to determine if there is a pattern of the loans
qui ont été consentis à d'autres organisations. Récapituler les montants et les to other organizations. Summarize amounts and include in the
inclure dans les communications de la mission. engagement communication.
B. Rapporter les montants à la commission chargée des prêts et lui laisser le B. Report the amounts to the loan committee and leave it up to
soin de procéder aux corrections. N'entreprendre aucune autre action de suivi à them to correct. Take no further follow-up action at this time and do
ce moment-là et ne pas mentionner ces éléments dans les communications. not include the items in the engagement communication.
C. Faire le suivi avec le vice-président et inclure l'opinion de ce dernier sur la C. Follow up with the vice president and include the vice president's
situation dans les communications. acknowledgment of the situation in the engagement communication.
D. Déterminer le montant des écarts et estimer si les différences chiffrées D. Determine the amount of differences and make an assessment
sont importantes. Si les montants ne sont pas significatifs, qu'ils n'enfreignent as to whether the monetary differences are material. If the amounts
pas la loi et qu'ils peuvent s'expliquer, ne pas mentionner cette observation are not material, not in violation of government regulations, and can
dans les rapports. be rationally explained, omit the observation from the engagement
communication.

La réponse (A) est fausse. L'auditeur interne doit essayer de Answer (A) is incorrect because the internal auditor should attempt
déterminer les causes des observations de la mission et, si to determine the causes of engagement observations and, if
nécessaire, les inclure dans les rapports. appropriate, include them in the engagement communication.
La réponse (B) est juste. Les auditeurs internes doivent communiquer Answer (B) is correct. Internal auditors should report the
les résultats de la missions rapidement (CRIPP). De plus, “le engagement results promptly (IPPF). Moreover, the CAE
responsable de l'audit interne doit mettre en place un processus de should establish a follow-up process to monitor and ensure
suivi permettant de surveiller et de garantir que des mesures ont été that management actions have been effectively implemented
effectivement mises en œuvre par le management ou que la Direction or that senior management has accepted the risk of not taking
Générale a accepté de prendre le risque de ne rien faire” (CRIPP). En action (IPPF). Furthermore, failure to disclose known material
outre, la non communication des faits matériels dont l'auditeur interne facts that, if not disclosed, may distort the reporting of
a connaissance et qui, s'ils n'étaient pas révélés, auraient pour activities under review is a violation of Conduct Rule 2.3.
conséquence de fausser le rapport sur les activités examinées Answer (C) is incorrect because the engagement client's view about
constitue une violation de la Règle de Conduite 2.3. engagement observations, conclusions, and recommendations
La réponse (C) est fausse. L'opinion du client de la mission sur les should be included in the engagement communication.
observations, conclusions et recommandations de la mission doit figurer dans Answer (D) is incorrect because failure to report the deviations may
les communications. be justified if the internal auditor has concluded that the amounts are
La réponse (D) est fausse. La non communication des écarts peut se justifier clearly not material, that they are not in violation of governmental
si l'auditeur interne juge que leurs montants ne sont pas significatifs, qu'ils regulations, and that a rationale for the deviations exists.
n'enfreignent pas la loi et qu'ils peuvent s'expliquer.

Restitution

© IFACI – CIA1 V.5.1 page 55/73


Acquérir une certification
> Préparation au CIA, partie 1
P1 - 93 -- Afin de déterminer l’étendue des contrôles à effectuer pendant le P1 - 93 -- To determine the extent of testing to be performed during
travail sur le terrain, la préparation du programme de travail doit être l'étape field work, preparing the engagement work program should be the
qui suivra : next step after completing the
A. L’étude préliminaire A. Preliminary survey.
B. Une revue des politiques de la société. B. Survey of company policies.
C. L’allocation des tâches aux auditeurs. C. Assignment of audit staff.
D. La définition des délais à respecter pour les différentes tâches de la mission. D. Time budgets for specific audit tasks.
La réponse (A) est juste. La planification inclut la réalisation, si cel(A) Answer (A) is correct. Planning includes performing, if
est opportun, d’une étude afin de se familiariser avec les activités, les appropriate, a survey to become familiar with the activities,
risques, et les contrôles qui doivent être analysés; l’identification des risks, and controls to be reviewed; to identify areas for
domaines qui doivent être étudiés en profondeur pendant la mission; et engagement emphasis; and to invite comments and
l’invitation aux clients de soumettre leurs commentaires (CRIPP). La suggestions from engagement clients (IPPF). Writing the work
prochaine étape est de rédiger le programme de travail. program is the next step.
La réponse (B) est fausse. Cette revue n’est pas une base suffisante pour un Answer (B) is incorrect because this survey is not a sufficient basis
programme de travail, qui est une liste détaillée des procédures pour la mission. for a work program, which is a detailed listing of engagement
procedures.
La réponse (C) est fausse. L’allocation des tâches aux auditeurs est faite
avant l’étude préliminaire. Answer (C) is incorrect because staff assignments are made prior
to the preliminary survey.
La réponse (D) est fausse. La définition des délais à respecter pour les
différentes tâches fait partie de la préparation du programme de travail. Answer (D) is incorrect because time budgets for specific tasks are
determined as part of the preparation of the work program.

P1 - 94 -- Un auditeur interne vient d'être informé de la prochaine mission, et P1 - 94 -- The internal auditor-in-charge has just been informed of
l'équipe a été constituée. Sélectionner la phase appropriée pour finaliser le the next engagement, and the engagement team has been
budget de la mission : assigned. Select the appropriate phase for finalizing the
engagement budget.
A. Pendant la formulation du plan à long terme.
B. Après l'examen préliminaire. A. During formulation of the long-range plan.
C. Durant la réunion de planification initiale. B. After the preliminary survey.
D. Après l'achèvement de tous les travaux sur place. C. During the initial planning meeting.
D. After the completion of all field work.

La réponse (A) est fausse. Un budget initial est fixé pendant la formulation du Answer (A) is incorrect because an initial budget is determined
plan à long terme, mais des révisions sur la base de l'examen préliminaire during the formulation of the long-range plan, but revisions based
peuvent se révéler nécessaires. on the preliminary survey may be required.
La réponse (B) est juste. “Un examen préliminaire permet une Answer (B) is correct. A survey permits an informed approach
approche documentée pour planifier et réaliser la mission. C'est un to planning and carrying out engagement work and is an
moyen efficace pour affecter les ressources de l'audit interne aux effective tool for applying the IAA's resources where they can
secteurs où elles peuvent être utilisées le plus efficacement”. Ses be used most effectively. Among other things, the results of
résultats doivent entre autres comprendre “les premières estimations the survey should include preliminary estimates of time and
en matière de délai et de besoin en ressources” (CRIPP). Ainsi, une fois resource requirements (IPPF). Thus, after the preliminary
l'examen préliminaire achevé, le budget final de la mission peut être survey has been completed, the final engagement budget can
élaboré. be prepared.
La réponse (C) est fausse. Lors de la réunion de planification initiale, le projet Answer (C) is incorrect because at the initial planning meeting
n'est pas suffisamment défini pour déterminer le budget final. stage, the project is not sufficiently defined to complete the final
La réponse (D) est fausse. Après l'achèvement des travaux sur place, le budget.
budget ne sert plus d'outil de contrôle et d'évaluation. Answer (D) is incorrect because, after the completion of the field
work, the budget is no longer useful as a control and evaluation
tool.

Restitution

© IFACI – CIA1 V.5.1 page 56/73


Acquérir une certification
> Préparation au CIA, partie 1
[Enoncé #9] La direction d'une entreprise de téléphonie et de télévision par [Fact Pattern #9] Management of a telephone and cable television
câble commande un audit de son service clients. Ce dernier est chargé de enterprise has requested an engagement to evaluate its service
traiter toutes les demandes d'information de la clientèle qui portent sur les department. The department is responsible for handling all
performances du téléphone ou du câble et d'affecter son personnel aux customer inquiries regarding telephone or cable performance and
interventions, à l'installation et au raccordement de nouvelles liaisons par câble, assigning staff personnel to fix the problem, installing new cable
à la vente de produits complémentaires tels que des téléphones de designers, lines and hooking them up to the customer's home, selling
à la vente de services complémentaires, par exemple des services de télévision complementary products such as designer telephones, selling
par câble sur abonnement ou des services téléphoniques additionnels, comme complementary services such as premium cable television
le signal d'appel et la conversation à trois. services, and additional telephone services such as call-waiting,
La direction est très orientée sur la satisfaction du client et s’est fixé comme three-line hookup, etc.
règle que 95 % des plaintes et des demandes du client devaient être traitées Management is very customer oriented and has a policy that 95%
de façon à satisfaire le client dans les 24 heures après la réception de l’appel. of customer complaints or inquiries must be fully addressed to the
Toutes les demandes qui nécessitent l’intervention d’un technicien doivent customer’s satisfaction within 24 hours of receiving the call. All
être enregistrées par l’agent qui prend l’appel. Même si ce service existe pour customer service inquiries that require a service technician must be
répondre aux besoins des abonnés, la direction le considère aussi comme un logged in by the responding service agent. Although the
centre de profit. Toutes les recettes sont générées par la vente de services department exists to handle customer needs, it is also viewed by
ou d’équipements supplémentaires et de services après-vente. Ce management as a profit center. All revenue is generated from the
département existe depuis trois ans seulement et la direction n’a pas élaboré sale of complementary services or equipment and from service
de mécanisme pour en évaluer les performances. Cependant, la direction revenue. The department has been in existence for only three
s’inquiète de ce qu’il ne remplisse pas les objectifs de la direction. Pendant la years and management has not developed a mechanism to
planification préliminaire, les informations suivantes sont collectées. evaluate the department’s performance. However, management is
concerned that the department might not be meeting
Informations Ex. en cours Ex. précédent Ex. antérieur management’s goals. During the preliminary planning, the following
préliminaires (20X3) (20X2) (20X1) information is gathered.

Nb. de plaintes Current Year Prev. Year Prior Year


37 500 35 000 34 000 Preliminary Information
enregistrées (20X3) (20X2) (20X1)
Nb. d’interventions
10 080 10 000 8 640 Number of complaints
chez les clients 37,500 35,000 34,000
logged
Nb. de clients 1 250 000 1 100 000 1 000 000 Number of service calls
10,080 10,000 8,640
at customer locations
Chiffre d’affaires des
510 000 $ 500 000 $ 432 000 $ Number of customers 1,250,000 1,100,000 1,000,000
interventions

Ventes de services ou Service call revenue $510,000 $500,000 $432,000


d’équipements 6 250 000 $ 4 400 000 $ 3 750 000 $
complémentaires Sales of complementary
$6,250,000 $4,400,000 $3,750,000
services or equipment
Personnel d'accueil 7 7 5
Techniciens 7 7 6 Staff 7 7 5
Technicians 7 7 6
S.A.V. 1 344 000 $ 1 275 000 $ 1 056 000 $
Total Service
$1,344,000 $1,275,000 $1,056,000
Department Costs

Restitution

© IFACI – CIA1 V.5.1 page 57/73


Acquérir une certification
> Préparation au CIA, partie 1
P1 - 95 -- (Voir énoncé 9) Pour déterminer si l'organisation atteint son objectif P1 - 95 -- (Refers to Fact Pattern #9) The best engagement
- répondre à 95 % des demandes d'information de ses clients sous 24 procedure to determine whether the organization is meeting its
heures-, la meilleure méthode consiste à : objective of satisfying 95% of customer inquiries within 24 hours is
to
A. Sélectionner par sondage pour estimation d'attributs un échantillon des A. Select an attribute sample from the population of logged-in
réclamations de clients enregistrées et le comparer aux documents customer complaints and trace to records indicating its disposition,
comptables indiquant la façon dont ces réclamations ont été traitées et dans noting time and nature of disposition.
quel délai. B. Develop a customer satisfaction survey and send it to all
B. Elaborer un questionnaire de satisfaction et l'envoyer à tous les clients, en customers, and include a question about the timeliness of service.
y incluant une question sur la rapidité du service. C. Develop a customer satisfaction survey and send it to a
C. Elaborer un questionnaire de satisfaction et l'envoyer à un échantillon de statistically selected sample of customers based on mean-per-unit
clients recueilli au moyen d'un sondage statistique pour estimation de sampling and ask them to identify the time it took for the service
variables par la moyenne, en demandant à ces clients de préciser dans quel department to satisfactorily address their problem. Calculate a
délai le service clients a réglé leur problème. mean and standard deviation.
D. Sélectionner au moyen d'un sondage statistique pour estimation de D. Select a mean-per-unit statistical sample from the total
variables par la moyenne un échantillon de la population totale et comparer population base and trace the sample to the customer complaint
ce dernier au journal des réclamations des clients, afin de déterminer si 95 % log to determine whether or not 95% of the sample had their
de l'échantillon avaient vu leur problème réglé en 24 heures. complaints addressed within 24 hours.

La réponse (A) est juste. La meilleure méthode consiste à sélectionner Answer (A) is correct. The best method is to take an attribute
par sondage pour estimation d'attributs un échantillon des sample from the log of customer complaints. Whether
réclamations des clients. Elle permet d'obtenir uniquement des inquiries have been satisfied promptly is a yes/no, either/or
réponses de type oui/non et ou/ou à la question portant sur le délai de proposition appropriate for this sampling method. The
traitement des réclamations. Les éléments échantillonnés doivent être sampled items should be traced to documentation that
comparés aux documents qui doivent faire apparaître l'intervention du should evidence the completion of customer service.
service après-vente. Answer (B) is incorrect because surveying all customers would be
La réponse (B) est fausse. Il serait onéreux de questionner tous les clients. costly. The procedure also has two additional faults: (1) the
Cette procédure présente deux autres inconvénients : (1) le client ne se customer might not recall the time necessary for the repair, and (2)
souvient pas toujours du délai d'intervention qui a été nécessaire et (2) il ne the customer might not respond.
répondra pas forcément. Answer (C) is incorrect because surveying a sample of customers
La réponse (C) est fausse. La méthode consistant à questionner un suffers from the same two faults noted above. Moreover, MPU is a
échantillon de clients présente les deux inconvénients décrits ci-dessus. De variables sampling technique that is appropriate for testing values,
plus, le sondage pour estimation de variables par la moyenne est une not binary propositions.
technique de sondage pour estimation de variables qui est appropriée pour Answer (D) is incorrect because sampling from the total customer
vérifier des valeurs, mais pas des propositions binaires. population is inefficient. Not all customers have complained during
La réponse (D) est fausse. La méthode consistant à prendre un échantillon the year. Also MPU is inappropriate.
de la population totale de clients est inefficiente. En effet, les clients ne se
sont pas tous plaints au cours de l'année et le sondage pour estimation de
variables par la moyenne est inapproprié.

Restitution

© IFACI – CIA1 V.5.1 page 58/73


Acquérir une certification
> Préparation au CIA, partie 1
P1 - 96 -- Lequel des facteurs suivants est considéré comme étant le moins P1 - 96 -- Which of the following factors is considered the least
important dans la décision de réaffecter les ressources d'audit interne important in deciding whether existing internal audit resources
existantes en train de réaliser une mission relative à la conformité à une should be moved from an ongoing compliance engagement to a
mission au niveau d'une division, demandée par la direction ? divisional-level engagement requested by management?
A. Un audit financier de la division en question réalisé par l'auditeur externe A. A financial audit of the division performed by the external auditor
un an auparavant. a year ago.
B. Le risque de fraude associé à la mission en cours. B. The potential for fraud associated with the ongoing engagement.
C. Un relèvement du niveau des dépenses enregistré par cette division pour C. An increase in the level of expenditures experienced by the
l'exercice précédent. division for the past year.
D. Le risque de se voir infliger des amendes significatives suite à la mission D. The potential for significant regulatory fines associated with the
en cours. ongoing engagement.
La réponse (A) est juste. “Il convient de définir des priorités de façon à Answer (A) is correct. Prioritizing is needed to make decisions
affecter les ressources en fonction du caractère significatif des about applying relative resources based on the significance
risques[...]. La plupart de ces modèles déterminent l'ordre de priorité of risk and exposure. Most risk models use risk factors to
des missions en fonction de facteurs de risque.” (CRIPP). L'un de ces establish engagement priorities (IPPF). One such factor is the
facteurs est le risque de fraude. Les auditeurs internes considèrent potential for fraud. Internal auditors traditionally regard fraud
traditionnellement la fraude comme significative, même si le risque as significant even if the immediate exposure is not
immédiat n'est pas significatif. Un relèvement des dépenses constitue significant. Increased expenditures also constitute a
également un facteur de risque significatif, car il représente une significant risk factor because they represent an increase in
augmentation du risque de pertes. Pour la même raison, le risque de se potential loss. For the same reason, potential regulatory fines
voir infliger des amendes peut également créer un risque suffisamment may also create an exposure sufficiently great to affect the
important pour avoir une incidence sur la définition des priorités. Ainsi, determination of priorities. Thus, the result of an external
le résultat d'un audit financier externe réalisé il y a un an est le moins financial audit performed a year ago is the least likely to affect
susceptible d'affecter l'allocation actuelle des ressources de l'audit the current allocation of internal audit resources. Any adverse
interne. Toutes les observations négatives ont certainement été suivies engagement observations most probably have been acted
de mesures correctrices et, dans tous les cas, ne seront pas en rapport upon, and, in any case, may not be germane to the ongoing
avec la mission relative à la conformité en cours ou avec la mission compliance engagement or the proposed divisional-level
proposée au niveau de la division. engagement.
La réponse (B) est fausse. Le risque de fraude est susceptible de peser Answer (B) is incorrect because potential fraud is likely to be a
davantage dans la décision d'utiliser les ressources limitées de l'audit interne more important factor in the use of limited internal audit resources
que les résultats d'un audit financier externe. than the results of an external financial audit.
La réponse (C) est fausse. Une hausse des dépenses est susceptible de Answer (C) is incorrect because increased expenditures is likely to
peser davantage dans la décision d'utiliser les ressources limitées de l'audit be a more important factor in the use of limited internal audit
interne que les résultats d'un audit financier externe. resources than the results of an external financial audit.
La réponse (D) est fausse. Le risque d'amendes importantes est susceptible Answer (D) is incorrect because potential significant fines are likely
de peser davantage dans la décision d'utiliser les ressources limitées de to be a more important factor in the use of limited internal audit
l'audit interne que les résultats d'un audit financier externe. resources than the results of an external financial audit.

Restitution

© IFACI – CIA1 V.5.1 page 59/73


Acquérir une certification
> Préparation au CIA, partie 1
P1 - 97 -- Lequel des énoncés suivants est faux en ce qui concerne P1 - 97 -- Which of the following statements is false regarding risk
l’évaluation de risques dans le sens où le terme est utilisé par rapport à l’audit assessment as the term is used in internal auditing?
interne ?
A. L’évaluation des risques est un processus raisonné d’allocation de A. Risk assessment is a judgmental process of assigning monetary
montants au niveau du risque perçu dans une activité évaluée. Ces montants amounts to the perceived level of risk found in an activity being
permettent à un responsable d’audit interne de choisir la mission qui evaluated. These amounts allow a chief audit executive to select
entraînera le plus d’économies pour le client. the engagement clients most likely to result in identifiable savings.
B. Le responsable d’audit interne doit incorporer dans le processus d’évaluation B. The chief audit executive should incorporate information from a
des risques des informations d’une variété de sources, y compris des variety of sources into the risk assessment process, including
discussions avec le conseil d’administration, les auditeurs externes, une revue discussions with the board, management, external auditors, review
des règlements, et une analyse des données financières et opérationnelles. of regulations, and analysis of financial/operating data.
C. L’évaluation des risques est un processus systématique d’évaluation et de C. Risk assessment is a systematic process of assessing and
prise en compte de jugements professionnels sur des événements qui integrating professional judgments about events that could affect
pourraient avoir un impact sur la réalisation des objectifs de la société. Elle the achievement of organizational objectives. It provides a means
fournit un moyen d’organiser un programme de travail pour une mission. of organizing an engagement work schedule.
D. Suite à une mission ou à une étude préliminaire, le responsable d’audit D. As a result of an engagement or preliminary survey, the chief
interne peut à tout moment réviser le niveau de risque évalué d’un client de la audit executive may revise the level of assessed risk of an
mission, apportant les modifications nécessaires au programme de travail. engagement client at any time, making appropriate adjustments to
the work schedule.
La réponse (A) est juste. L’audit interne doit évaluer les processus de Answer (A) is correct. The nature of work of the IAA is to
management des risques, de contrôle et de gouvernement d’entreprise evaluate and contribute to the improvement of risk
et contribuer à leur amélioration (CRIPP). Le responsable d’audit management, control, and governance systems (IPPF).
interne doit donc établir une planification fondée sur les risques, afin de Accordingly, the CAE should establish riskbased plans, to
définir les priorités cohérentes avec les objectifs de l’organisation determine the priorities of the IAA, consistent with the
(CRIPP). Ces plans sont reflétés dans les programmes des missions organization's goals (IPPF). These plans are reflected in
basées, parmi d’autres choses, sur une évaluation de la priorité et de engagement work schedules based on, among other things,
l’exposition au risque. Les priorités sont les décisions d’allocation de an assessment of risk priority and exposure. Priorities are
ressources en fonction de l’importance des risques et de l’exposition. decisions for applying resources based on the significance of
Plusieurs modèles de risque peuvent être utilisés pour définir les risk and exposure. A variety of risk models may be used to
priorités. La plupart des modèles de risque sont basés sur des facteurs set priorities. Most risk models are based on risk factors, for
de risque, par exemple la matérialité des montants financiers, la example, materiality of monetary amounts, asset liquidity,
liquidité des actifs, la compétence de la direction, la qualité du contrôle management competence, quality of internal control, degree
interne, le degré du changement, la date de la dernière mission, la of change, time of the last engagement, complexity, employee
complexité, les relations de travail, ou relations gouvernementales relations, or government relations (IPPF). Consequently, risk
(CRIPP). Par conséquent, l’évaluation des risques n’est pas toujours un assessment is not a rigid process that can always be made in
processus rigide qui peut être fait en termes spécifiquement financiers. specific financial terms.
La réponse (B) est fausse. Les auditeurs internes doivent identifier et évaluer Answer (B) is incorrect because internal auditors are expected to
toute exposition à un risque important pendant l’exécution normale de leurs identify and evaluate significant risk exposures in the normal
fonctions. Non seulement utilisent-ils donc l’analyse des risques dans la course of their duties. Thus, they not only use risk analysis to plan
planification de leurs missions mais également pour aider la direction et le engagements but also to assist management and the board by
conseil d’administration en examinant, évaluant, rendant compte, et examining, evaluating, reporting, and recommending
recommandant des améliorations quant au caractère adéquat et à l’efficacité improvements on the adequacy and effectiveness of the
des processus de risque de la direction (CRIPP). Pour cela, le RAI doit management's risk processes (IPPF). For these purposes, the
incorporer les informations de diverses sources dans le processus d’évaluation CAE should incorporate information from a variety of sources into
des risques. Les Normes n’imposent aucune limite sur les sources. the risk assessment process. The Standards place no limit on such
La réponse (C) est fausse. Le programme d’une mission est basé sur une sources.
évaluation des risques, conduite au moins une fois par an (CRIPP). Answer (C) is incorrect because an engagement work schedule is
La réponse (D) est fausse. Les évaluations des risques peuvent être révisées based on a risk assessment, undertaken at least annually (IPPF).
sur la base de nouvelles informations. Answer (D) is incorrect because risk assessments may be revised
on the basis of new information.

Restitution

© IFACI – CIA1 V.5.1 page 60/73


Acquérir une certification
> Préparation au CIA, partie 1
P1 - 98 -- Quelle stratégie peut-on recommander à un participant à une P1 - 98 -- An advisable strategy for a participant in a meeting of the
réunion du service d'audit interne ? internal auditing staff is to
A. Lire l'ordre du jour et les documents afférents à la réunion pendant la A. Read the agenda and supporting materials for the meeting
première partie de la réunion, pour se préparer à la discussion ultérieure. during the early part of the meeting to prepare for later discussion.
B. Présenter immédiatement des opinions tranchées sur un aspect d'une B. Present strong opinions on one side of a proposal right away.
proposition. C. Present views as trial balloons that can be researched later.
C. Lancer des idées en vue de les approfondir plus tard. D. Consider the opinions and information needs of other
D. Tenir compte des opinions et des besoins d'information des autres participants before speaking.
participants avant de prendre la parole.
La réponse (A) est fausse. Les participants les plus efficaces se sont Answer (A) is incorrect because the most effective meeting
préparés avant d'arriver à la réunion. L'ordre du jour et les documents participants come to meetings prepared. The agenda and other
concernant la réunion doivent être lus à l'avance. materials should be read in advance.
La réponse (B) est fausse, car à moins que la personne qui parle ne soit Answer (B) is incorrect because, unless the speaker is certain of
certaine des opinions des autres (ou qu'elle soit la personne la plus influente others' opinions (or is the most powerful person in the
de l'organisation), elle ne doit pas prendre une position ferme avant que le organization), (s)he should not commit to a position until the degree
degré d'adhésion pour cette opinion puisse être estimé. of support for that view can be estimated.
La réponse (C) est fausse. Les idées doivent être étudiées avant la réunion Answer (C) is incorrect because ideas should be researched in
de manière à ce que le participant paraisse préparé et productif. advance of the meeting so that the participant appears to be
La réponse (D) est juste. L'analyse de son auditoire permet à celui qui prepared and productive.
parle de collecter les bonnes informations pour la réunion. En outre, Answer (D) is correct. Analyzing the audience assists a
comprendre les opinions et les besoins des autres participants permet speaker to gather the right information for the meeting.
à celui qui parle d'exprimer ses idées de la manière qu'il juge la mieux Moreover, understanding the other participants' opinions and
appropriée pour être convaincant. needs enables the speaker to express his/her ideas in the way
best calculated to be persuasive.

P1 - 99 -- Vous êtes le responsable de l'audit interne d'une grande P1 - 99 -- You are the chief audit executive for a large decentralized
organisation décentralisée. Vous avez élaboré un manuel qui contient des organization. You have developed a manual containing
procédures écrites détaillées et complètes pour guider les groupes de travail comprehensive detailed written procedures as a guide for your
décentralisés. Chacun de ces groupes compte 20 à 30 auditeurs internes. decentralized engagement work groups, each of which has 20 to
L'organisation vient d'acquérir une petite entité qui possède un petit service 30 internal auditors. The organization recently acquired a small
d'audit interne composé d'un superviseur et de deux auditeurs. Laquelle des entity that has an internal audit activity consisting of a supervisor
actions suivantes sera la mieux à même de donner une orientation and two staff personnel. Which of the following actions is the most
administrative au nouveau service d'audit interne ? practical in providing administrative guidance for this new internal
audit activity?
A. Sélectionner des procédures clés dans le manuel et utiliser les indications A. Select key procedures from the manual and use informal
informelles des superviseurs pour les autres questions relatives au supervisory direction for other engagement management issues.
management de la mission. B. Use informal supervisory direction for engagement management
B. Utiliser les indications informelles des superviseurs pour les questions issues.
relatives au management de la mission. C. Use the already developed manual.
C. Utilisé le manuel déjà élaboré. D. Adopt the administrative procedures being followed by the
D. Adopter les procédures administratives suivies par les auditeurs internes internal auditors of the acquired entity.
de l'entité rachetée.
La réponse (A) est juste. Afin de familiariser le personnel de l'entité Answer (A) is correct. Orientation to acquaint the acquired
nouvellement acquise avec son nouvel environnement, il convient de entity's staff with the established environment should be
l'exposer à des procédures clés sélectionnées et extraites du manuel. through exposure to selected key procedures from the formal
La forme et le contenu des règles et procédures écrites doivent manual. The form and content of written policies and
correspondre à la taille et à la structure du service d'audit interne. Un procedures should be appropriate to the size and structure of
petit service d'audit interne peut être dirigé de manière moins formelle. the IAA. Thus, a small IAA may be managed less formally, for
Il peut par exemple être “contrôlé au jour le jour par une supervision example, through daily close supervision and written
étroite et des notes de service” (CRIPP). memoranda (IPPF).
La réponse (B) est fausse. L'utilisation de la seule supervision informelle pour Answer (B) is incorrect because the use of informal supervisory
du personnel nouvellement embauché est inadéquate. direction alone for new staff is inadequate.
La réponse (C) est fausse. L'utilisation du manuel existant dans son Answer (C) is incorrect because complete reliance on the existing
intégralité nécessiterait de diriger le service de manière plus formelle que manual would require more formal management than is necessary
nécessaire pour un petit service d'audit interne. for a small IAA.
La réponse (D) est fausse. Le management de la nouvelle entité d'audit Answer (D) is incorrect because management of the new internal
interne doit être en droite ligne de celui du reste de l'organisation. auditing entity should not be inconsistent with the rest of the
organization.

Restitution

© IFACI – CIA1 V.5.1 page 61/73


Acquérir une certification
> Préparation au CIA, partie 1
P1 - 100 -- La synthèse annuelle des travaux achevés, que le responsable de P1 - 100 -- An annual summary report of completed engagement
l'audit interne soumet à la direction générale et au Conseil doit : work submitted to senior management and the board by the chief
audit executive should
A. Traiter du statut administratif de l'audit interne.
B. Informer la direction du champ d'intervention des travaux proposés pour A. Discuss the administrative condition of the internal audit activity.
l'année suivante. B. Inform management of the scope of proposed work for the
C. Dire dans quelle mesure l'audit interne a rempli son programme de missions. following year.
D. Mettre en avant le nombre d'observations relatives à des anomalies C. Describe the extent to which the internal audit activity has
découvertes par les auditeurs internes. completed its engagement work schedule.
D. Emphasize the number of deficiency observations discovered
by the internal auditors.
La réponse (A) est fausse. Le statut administratif de l'audit interne est un Answer (A) is incorrect because the administrative condition of the
aspect qui relève de l'évaluation externe. IAA is a subject appropriate for an external assessment.
La réponse (B) est fausse. Cette information est contenue dans la synthèse Answer (B) is incorrect because this information is contained in the
du programme de travail, des prévisions d'effectifs et du budget financier pour summary of the engagement work schedule, staffing plan, and
l'année à venir qui est soumise à la direction générale et au Conseil (CRIPP). financial budget for the coming year submitted to senior
La réponse (C) est juste. “Les rapports d'activité doivent mentionner management and the board (IPPF).
les constatations les plus importantes obtenues lors des audits et les Answer (C) is correct. Activity reports should highlight
recommandations correspondantes. Ils doivent également permettre significant engagement observations and recommendations
d'informer la direction générale et le Conseil de toute variation and inform senior management and the board of any
importante par rapport au programme de travail, aux prévisions significant deviations from approved engagement work
d'effectifs, et aux budgets. Les raisons de ces variations doivent être schedules, staffing plans, and financial budgets, and the
expliquées” (CRIPP). reasons for them (IPPF).
La réponse (D) est fausse. C'est l'importance des observations, et non leur Answer (D) is incorrect because the materiality of observations, not
nombre, qui doit être mise en avant. their number, should be emphasized.

P1 - 101 -- Le développement organisationnel (DO) est l'une des grandes P1 - 101 -- Organizational development (OD) is one of the major
approches de la gestion dynamique du changement dans les organisations. approaches to proactive management of change in organizations.
L'un de ses principaux objectifs est : One of the major objectives of OD is to
A. De renforcer le pouvoir des leaders. A. Increase the power of leaders.
B. D'aligner les objectifs de l'organisation et des salariés. B. Align the organization's and the employees' goals.
C. D'attirer de meilleurs éléments au sein de l'organisation. C. Attract better employees to the organization.
D. De fournir à l'organisation et à ses dirigeants des moyens d'accroître D. Provide the organization and its managers with ways to increase
l'efficience. efficiency.
La réponse (A) est fausse. Le DO est axé sur la participation et le partage Answer (A) is incorrect because OD focuses on participation and
des pouvoirs. power sharing.
La réponse (B) est juste. Le DO a pour but d'approfondir le sens de Answer (B) is correct. The objectives of OD are to deepen the
l'objectif organisationnel et de faire en sorte que les individus y sense of organizational purpose and align individuals with it;
adhèrent, de favoriser la confiance, la communication, la coopération et to promote interpersonal trust, communication, cooperation,
le soutien entre les individus, de promouvoir une approche axée sur la and support; to encourage a problem-solving approach; to
résolution des problèmes, de rendre l'expérience professionnelle develop a satisfying work experience; to supplement formal
satisfaisante, de compléter l'autorité formelle par une autorité fondée authority with authority based on expertise; to increase
sur le savoir-faire, de conférer davantage de poids à la responsabilité personal responsibility; and to encourage willingness to
personnelle et d'encourager la volonté de changement. change.
La réponse (C) est fausse. Attirer de meilleurs éléments au sein d'une Answer (C) is incorrect because attracting better applicants to an
organisation ne fait pas partie des objectifs premiers du DO. organization is not a major goal of OD.
La réponse (D) est fausse. Le DO accroît l'efficacité pour l'organisation et sa Answer (D) is incorrect because OD provides an organization and
direction. its managers with higher effectiveness.

Restitution

© IFACI – CIA1 V.5.1 page 62/73


Acquérir une certification
> Préparation au CIA, partie 1
P1 - 102 -- Les situations de leadership varient en fonction du degré auquel le P1 - 102 -- Leadership situations vary with regard to the degree to
leader peut déterminer ce que ses subordonnés feront, comment ils le feront which the leader can determine what subordinates will do, how
et ce qu'il en résultera. Selon la théorie de la contingence de Fiedler, un they will do it, and what the results will be. According to Fiedler's
leader dont le style de management est orienté sur les relations humaines contingency theory, a leader with a relationshiporiented
sera efficace s'il exerce : management style will be most effective when (s)he exerts
A. Un contrôle important. A. Great control.
B. Un contrôle modéré. B. Moderate control.
C. Un contrôle limité. C. Little control.
D. Un contrôle important ou limité. D. Great or little control.
La réponse (A) est fausse. Le leader orienté sur la tâche [ne correspond pas Answer (A) is incorrect because the task-oriented leader is least
à la question] sera le moins efficace dans cette situation. effective in this situation.
La réponse (B) est juste. Un leader orienté sur les relations est axé sur Answer (B) is correct. A relationship-oriented manager is
les salariés. Son estime de soi dépend fortement de ses interactions employee centered. His/her self-esteem is strongly affected
avec ses subordonnés. Fiedler indique que ce cadre est le plus efficace by personal interactions with subordinates. Fiedler indicated
lorsqu'il ne se trouve pas dans des situations extrêmes, où le contrôle that such a manager is most effective when not faced with the
est maximal ou minimal. Un contrôle strict découle d'un pouvoir lié à extremes of high or low control situations. High control
une position forte, une tâche structurée et de bonnes relations entre follows from strong position power, a structured task, and
leader et membres. Une situation de faible contrôle présente les good leader-member relations. A low control situation has
caractéristiques inverses. Dans un environnement de contrôle just the opposite characteristics. In a high-control
important, les relations interpersonnelles peuvent entrer peu en ligne environment, a concern for personal relations may be
de compte. Dans une situation de contrôle faible, le leader orienté sur unimportant. In a low-control situation, the relationship-
les relations humaines peut se révéler incapable de procéder à la oriented leader may be unable to provide the needed task
structuration des tâches nécessaire. La situation de contrôle modérée structuring. Thus, the moderate control situation is best. An
est ainsi la meilleure. La chaîne de montage (tâche structurée) en example is an assembly-line situation (a structured task) in
constitue un bon exemple : les relations entre leader et membres sont which leader-member relations are poor.
faibles dans ce cas. Answer (C) is incorrect because the task-oriented leader is least
La réponse (C) est fausse. Le leader orienté sur la tâche sera le moins effective in this situation.
efficace dans cette situation. Answer (D) is incorrect because the task-oriented leader is least
La réponse (D) est fausse. Le leader orienté sur la tâche sera le moins effective in this situation.
efficace dans cette situation.

P1 - 103 -- La fonction de directeur du risque (chief risk officer - CRO) est P1 - 103 -- The function of the chief risk officer (CRO) is most
surtout efficace lorsque celui-ci : effective when the CRO:
A. Gère le risque en sa qualité de membre de la direction. A. Manages risk as a member of senior management.
B. Partage la gestion du risque avec l'encadrement intermédiaire. B. Shares the management of risk with line management.
C. Partage la gestion du risque avec le responsable de l'audit interne. C. Shares the management of risk with the chief audit executive.
D. Supervise le risque au sein de l'équipe de gestion du risque de l'entreprise. D. Monitors risk as part of the enterprise risk management team.
La réponse (A) est fausse. L'encadrement supérieur a une mission de Answer (A) is incorrect. Senior management has an oversight role
supervision dans le cadre de la gestion du risque. in risk management.
La réponse (B) est fausse. Le niveau de l'encadrement intermédiaire a une Answer (B) is incorrect. The risk knowledge at the line level would
connaissance du risque cantonnée à sa sphère d'activité. be specific only to that area of the organization.
La réponse (C) est fausse. Le responsable de l'audit interne n'est pas chargé Answer (C) is incorrect. The chief audit executive (CAE) does not
de la gestion du risque. have the responsibility for managing risk.
La réponse (D) est juste. Le directeur du risque est le plus efficace Answer (D) is correct. The chief risk officer is most effective
lorsqu'il est épaulé par une équipe disposant du savoir-faire et de when supported by a specific team with the necessary
l'expérience requis concernant les risques supportés par expertise and experience related to organizational risk.
l'organisation.

Restitution

© IFACI – CIA1 V.5.1 page 63/73


Acquérir une certification
> Préparation au CIA, partie 1
P1 - 104 -- Un responsable de la construction utilise une méthode de P1 - 104 -- A construction manager is using a distributive-
négociation distributive pour négocier le prix du bois de construction avec un bargaining approach in negotiating the price of lumber with a
fournisseur. Le responsable de la construction doit : supplier. The construction manager will
A. Accepter le prix demandé par le fournisseur afin de conserver une bonne A. Concede to the supplier's asking price in order to maintain a
relation de travail. positive working relationship.
B. Embaucher un médiateur pour négocier le contrat pour le compte du B. Hire a mediator to negotiate the deal on behalf of the manager.
responsable. C. Attempt to get agreement on a price within the settlement range
C. Essayer de s'entendre sur un prix compris dans la fourchette de règlement (that is, within both the manager's and supplier's aspiration ranges).
(c'est-à-dire dans la plage souhaitée par le responsable comme par son D. State the resistance point (that is, the highest price acceptable)
fournisseur). and ask the supplier to concede.
D. Déterminer le seuil de résistance (c'est-à-dire le prix acceptable le plus
élevé) et demander au fournisseur de l'accepter.
La réponse (A) est fausse. Lorsqu'il recourt à une méthode de négociation Answer (A) is incorrect because when using a distributive-
distributive, le responsable doit négocier un prix sur lequel lui-même et le bargaining approach, the manager should negotiate a price that
fournisseur peuvent s'entendre. both the manager and the supplier can agree on.
La réponse (B) est fausse, car si l'on utilise la méthode de négociation Answer (B) is incorrect because a mediator is not used when the
distributive pour régler un conflit, on ne fait pas appel à un médiateur. distributive-bargaining approach is used to resolve a conflict.
La réponse (C) est juste. Lorsqu'on utilise la méthode de négociation Answer (C) is correct. When using a distributive-bargaining
distributive, le négociateur a en tête un maximum souhaité (cible) et un approach, the negotiator operates with a maximum desired
minimum acceptable (seuil de résistance). Si la plage des résultats result (target point) and a minimum acceptable result
possibles (fourchette des aspirations) se chevauche, une entente est (resistance point) in mind. If the ranges of feasible outcomes
possible. (aspiration ranges) overlap, an agreement is possible.
La réponse (D) est fausse. Le responsable ne doit pas révéler le seuil de Answer (D) is incorrect because the manager should not reveal the
résistance (minimum acceptable). En revanche, il doit négocier pour pousser resistance point (the minimum acceptable result). Instead, (s)he
le fournisseur à accepter un montant plus proche de la cible. should negotiate to induce the supplier to agree to an amount
closer to the target point.

P1 - 105 -- Laquelle des affirmations suivantes sur les systèmes de P1 - 105 -- Which of the following statements is correct regarding
rémunération et de prime d'une entreprise est-elle correcte ? corporate compensation systems and related bonuses?
I. Un système de prime doit être considéré comme faisant partie de I. A bonus system should be considered part of the control
l'environnement de contrôle d'une organisation et doit être pris en compte environment of an organization and should be considered in
dans la formulation d'un rapport sur le contrôle interne. formulating a report on internal control.
II. Les systèmes de rémunération ne font pas partie de l'environnement de II. Compensation systems are not part of an organization's control
contrôle d'une organisation et les rapports ne doivent donc pas en tenir system and should not be reported as such.
compte. III. An audit of an organization's compensation system should be
III. Il convient d'effectuer l'audit du système de rémunération d'une entreprise performed independently of an audit of the control system over
indépendamment de celui du système de contrôle sur les autres fonctions other functions that impact corporate bonuses.
qui exercent une influence sur les primes.
A. I only.
A. I uniquement. B. II only.
B. II uniquement. C. III only.
C. III uniquement. D. II and III only.
D. II et III uniquement.
La réponse (A) est juste. Les systèmes de rémunération influencent le Answer (A) is correct. Compensation systems influence
comportement et doivent donc être considérés comme faisant partie behavior and should be considered an integral part of an
intégrante de la structure de contrôle d'une organisation. organization's control structure.
La réponse (B) est fausse. Les systèmes de rémunération font partie des Answer (B) is incorrect. Compensation systems are part of the
systèmes de contrôle de l'organisation. organization's control systems.
La réponse (C) est fausse. Les audits des systèmes de rémunération Answer (C) is incorrect. Audits of the compensation systems can
peuvent être associés à un audit d'autres fonctions qui influent sur les primes. be combined with an audit over other functions that impact
La réponse (D) est fausse. Les audits des systèmes de rémunération corporate bonuses.
peuvent être associés à un audit d'autres fonctions qui influent sur les primes. Answer (D) is incorrect. Compensation systems are part of the
organization's control systems. Audits of the compensation
systems can be combined with an audit over other functions that
impact corporate bonuses.

Restitution

© IFACI – CIA1 V.5.1 page 64/73


Acquérir une certification
> Préparation au CIA, partie 1
P1 - 106 -- L'activité qui consiste à faire des transactions sur les marchés à P1 - 106 -- The activity of trading futures with the objective of
terme dans le but de réduire le risque ou de le maîtriser s'appelle : reducing or controlling risk is called:
A. L'assurance. A. Insuring.
B. La couverture. B. Hedging.
C. La vente à découvert. C. Short-selling.
D. L'affacturage. D. Factoring.
La réponse (A) est fausse. L'assurance est une activité de gestion du risque. Answer (A) is incorrect. Insuring is a risk management activity.
La réponse (B) est juste. Une opération de couverture consiste à Answer (B) is correct. Hedging is the use of future contracts
recourir aux marchés à terme pour limiter son exposition au risque de to limit risk exposure on exchange rates.
change. Answer (C) is incorrect. Short-selling refers to the sales of
La réponse (C) est fausse. La vente à découvert consiste à vendre des commodities or shares of stocks.
matières premières ou des actions. Answer (D) is incorrect. Factoring applies to discounting of
La réponse (D) est fausse. L'affacturage consiste à escompter des créances accounts receivable
à recevoir.

P1 - 107 -- Qu'est-ce que le risque résiduel ? P1 - 107 -- What is residual risk?


A. L'impact du risque. A. Impact of risk.
B. Le risque qui est maîtrisé. B. Risk that is under control.
C. Le risque qui n'est pas géré. C. Risk that is not managed.
D. Le risque sous-jacent dans un environnement. D. Underlying risk in the environment.
La réponse (A) est fausse. L'impact du risque est la conséquence du risque. Answer (A) is incorrect. The impact of risk is its consequence.
La réponse (B) est fausse. Le risque maîtrisé est le risque géré. Answer (B) is incorrect. Risk that is under control is managed risk.
La réponse (C) est juste. Le risque résiduel est le risque qui persiste Answer (C) is correct. Residual risk is that risk left over after
après la mise en œuvre de tous les contrôles et de toutes les all controls and risk management techniques have been
techniques de gestion du risque. applied.
La réponse (D) est fausse. Le risque sous-jacent est le risque absolu. Answer (D) is incorrect. The underlying risk is the absolute risk.

Restitution

© IFACI – CIA1 V.5.1 page 65/73


Acquérir une certification
> Préparation au CIA, partie 1
[Enoncé #21] Le service marketing d'un gros distributeur affecte un chef de [Fact Pattern #21] The marketing department for a major retailer
produit différent à chaque ligne de produits. Les chefs de produit sont assigns separate product managers for each product line. Product
chargés de commander les produits et d'en déterminer le prix de vente au managers are responsible for ordering products and determining
détail. C'est le chef du marketing qui fixe le budget d'achat de chacun des retail pricing. Each product manager’s purchasing budget is set by
chefs de produit. Les produits sont livrés à une centrale de répartition où les the marketing manager. Products are delivered to a central
marchandises sont triées pour être envoyées aux 52 grands magasins du distribution center where goods are segregated for distribution to
groupe. Étant donné que les marchandises reçues sont enregistrées à la the company’s 52 department stores. Because receipts are
centrale de répartition, il n'y a pas de fonction réception dans les magasins. recorded at the distribution center, the company does not maintain
Les chefs de produit sont évalués sur la base du chiffre d'affaires et des a receiving function at each store. Product managers are evaluated
marges brutes dégagés sur leur ligne de produits. Beaucoup de produits sont on a combination of sales and gross profit generated from their
saisonniers et les directeurs de magasins sont en droit de demander que product lines. Many products are seasonal and individual store
certains produits soient retirés des rayons pour laisser la place à ceux de la managers can require that seasonal products be removed to make
saison suivante. space for the next season's products.
P1 - 108 -- (Voir énoncé 21) Les demandes d'achat sortant du cadre du P1 - 108 -- (Refers to Fact Pattern #21) Requests for purchases
budget initial doivent être validées par le directeur commercial. Cette beyond those initially budgeted must be approved by the marketing
procédure : manager. This procedure:
I. Devrait permettre l'allocation la plus efficiente des ressources limitées de I. Should provide for the most efficient allocation of scarce
l'organisation. organizational resources.
II. Est une procédure de contrôle de détection. II. Is a detective control procedure.
III. Est inutile, car chaque chef de produit est évalué sur la base du bénéfice III. Is unnecessary because each product manager is evaluated on
dégagé. profit generated.
A. I uniquement. A. I only.
B. III uniquement. B. III only.
C. II et III uniquement. C. II and III only.
D. I, II et III. D. I, II, and III.
La réponse (A) est juste. L'organisation doit allouer deux ressources Answer (A) is correct. The organization has two scarce
limitées : (a) son budget d'achat (limité par sa capacité financière) et (b) resources to allocate: (a) its purchasing budget (constrained
l'espace disponible dans les points de vente. Elle a donc besoin d'un by financing ability) and (b) space available in retail stores.
mécanisme lui permettant d'allouer ces deux ressources de manière à Thus, there is a need for a mechanism to allocate these two
maximiser le rendement global pour l'organisation. Il s'agit ici du scarce resources to maximize the overall return to the
mécanisme approprié. organization. This is the proper mechanism.
La réponse (B) est fausse. L'évaluation du chef de produit sur la base de la Answer (B) is incorrect. The gross profit evaluation is effective in
marge brute est efficace mais ne répond pas aux deux importantes evaluating the manager but does not address the two major
contraintes énoncées dans la proposition I. constraints identified in statement I.
La réponse (C) est fausse. La proposition II désigne un contrôle préventif, Answer (C) is incorrect. The item II is a preventive control, not a
non un contrôle de détection. L'évaluation du chef de produit sur la base de la detective control. The gross profit evaluation is effective in
marge brute est efficace mais ne répond pas aux deux importantes evaluating the manager but does not address the two major
contraintes énoncées dans la proposition I. constraints identified in statement I.
La réponse (D) est fausse. La proposition II désigne un contrôle préventif, Answer (D) is incorrect. The item II is a preventive control, not a
non un contrôle de détection. L'évaluation du chef de produit sur la base de la detective control. The gross profit evaluation is effective in
marge brute est efficace mais ne répond pas aux deux importantes evaluating the manager but does not address the two major
contraintes énoncées dans la proposition I. constraints identified in statement I.

P1 - 109 -- Lequel des objectifs suivants optimise les stratégies de gestion du P1 - 109 -- Which of the following goals sets risk management
risque ? strategies at the optimum level?
A. Minimiser les coûts. A. Minimize costs.
B. Maximiser la part de marché. B. Maximize market share.
C. Minimiser les pertes. C. Minimize losses.
D. Maximiser la valeur pour les actionnaires. D. Maximize shareholder value.
La réponse (A) est fausse. Ce n'est pas une approche complète de la gestion Answer (A) is incorrect. This is not a comprehensive approach to
du risque. risk management.
La réponse (B) est fausse. Voir réponse A. Answer (B) is incorrect. See answer "a".
La réponse (C) est fausse. Voir réponse A. Answer (C) is incorrect. See answer "a".
La réponse (D) est juste. C'est une approche complète, en liaison avec Answer (D) is correct. This is a comprehensive approach and
les stratégies de gestion du risque dans toute l'entreprise. will relate to risk management strategies across the
enterprise.

Restitution

© IFACI – CIA1 V.5.1 page 66/73


Acquérir une certification
> Préparation au CIA, partie 1
P1 - 110 -- Quelle est la raison principale de la planification ? P1 - 110 -- What is a primary reason for planning?
A. Une réaction aux besoins des salariés. A. Reacting to employee needs.
B. L’incertitude environnementale B. Environmental uncertainty.
C. Une réaction à la concurrence C. Reacting to competition.
D. Une politique de l’entreprise D. Organizational policy.
La réponse (A) est fausse. La planification aide un directeur à éviter d'avoir Answer (A) is incorrect because planning helps a manager avoid
besoin de réagir. Un des objectifs de la planification est d’anticiper, évitant reacting. Anticipating and avoiding problems is a purpose of
ainsi des problèmes. planning.
La réponse (B) est juste. La planification est influencée et limitée Answer (B) is correct. Planning is influenced and limited
(contrainte) par des facteurs externes (environnementaux), dont les (constrained) by external (environmental) factors, the effects
effets ne peuvent pas être déterminés avec certitude. Les tendances du of which cannot be determined with certainty. Market trends,
marché, les changements sociaux, les actions des concurrents, et la social change, competitors' actions, and governmental
réglementation gouvernementale figurent parmi les facteurs regulation are among these environmental factors.
environnementaux. Answer (C) is incorrect because planning helps a manager avoid
La réponse (C) est fausse. La planification aide un directeur à éviter le besoin reacting. Anticipating and avoiding problems is a purpose of
de réagir. Un des objectifs de la planification est d’anticiper, ainsi évitant des planning.
problèmes. Answer (D) is incorrect because organizational policies may require
La réponse (D) est fausse. Il est possible que les politiques de l’entreprise certain types of planning documents, but they cannot force
nécessitent certains types de documents de planification, mais elles ne managers to openly adopt planning as a useful tool.
peuvent pas obliger les responsables d’adopter ouvertement la planification
comme un outil utile.

P1 - 111 -- Tous les éléments suivants font partie du système de contrôle P1 - 111 -- All of the following would be part of a factory's control
d'une usine destiné à éviter les rejets d'effluents liquides non conformes aux system to prevent release of waste water that does not meet
normes sauf : discharge standards except:
A. Procéder à des analyses chimiques de l'eau avant le rejet pour détecter A. Performing chemical analysis of the water, prior to discharge, for
les composants énumérés dans l'autorisation. components specified in the permit.
B. Spécifier (dans la politique d'entreprise, par une formation ou par B. Specifying (by policy, training, and advisory signs) which
l'affichage de recommandations) les substances qu'il est permis d'évacuer substances may be disposed of via sinks and floor drains within the
par un évier ou les orifices d'évacuation dans le sol dans l'enceinte de l'usine. factory.
C. Laver périodiquement les éviers et les orifices d'évacuation dans le sol à C. Periodically flushing sinks and floor drains with a large volume of
grande eau afin que les substances polluantes soient suffisamment diluées. clean water to ensure pollutants are sufficiently diluted.
D. Mettre en place un programme de maintenance préventive pour le D. Establishing a preventive maintenance program for the factory's
système de prétraitement de l'usine. pretreatment system.
La réponse (A) est fausse. (comme B et D). Pris individuellement, chacun de Answer (A) is incorrect (like B and D). Each of these individual
ces contrôles, ainsi que d'autres probablement, aide la direction à parvenir à controls, and probably others as well, help management achieve its
son objectif, à savoir éviter le rejet d'effluents liquides non conformes aux objective of preventing the release of waste water that does not
plafonds autorisés ou à d'autres normes. Ces trois contrôles ont chacun une meet permit limits or other conditions. These three controls each
approche différente du risque. Les résultats des analyses constituent les approach the risk in different ways. Analytical results are the criteria
critères de décision sur les rejets ; le fait de ne pas évacuer les substances for the decision to discharge; keeping pollutants out of the waste
polluantes dans les eaux usées permet de réduire les concentrations et le water will help reduce concentrations and the degree of
degré de prétraitement nécessaire, et les pannes de matériel sont moins pretreatment needed; and equipment breakdown is less likely to
probables si un programme de maintenance préventive est en place. occur if a preventive maintenance program is in place.
La réponse (B) est fausse. (comme A et D). Pris individuellement, chacun de Answer (B) is incorrect (like A and D). Each of these individual
ces contrôles, ainsi que d'autres probablement, aide la direction à parvenir à controls, and probably others as well, help management achieve its
son objectif, à savoir éviter le rejet d'effluents liquides non conformes aux objective of preventing the release of waste water that does not
plafonds autorisés ou à d'autres normes. Ces trois contrôles ont chacun une meet permit limits or other conditions. These three controls each
approche différente du risque. Les résultats des analyses constituent les approach the risk in different ways. Analytical results are the criteria
critères de décision sur les rejets ; le fait de ne pas évacuer les substances for the decision to discharge; keeping pollutants out of the waste
polluantes dans les eaux usées permet de réduire les concentrations et le water will help reduce concentrations and the degree of
degré de prétraitement nécessaire, et les pannes de matériel sont moins pretreatment needed; and equipment breakdown is less likely to
probables si un programme de maintenance préventive est en place. occur if a preventive maintenance program is in place.
La réponse (C) est juste. Une dilution périodique ne peut pas toujours Answer (C) is correct. Periodic dilution may not always
prévenir les rejets de substances polluantes qui dépassent les limites prevent the release of pollutants which exceed the discharge
autorisées. limits.
La réponse (D) est fausse. (comme A et B). Pris individuellement, chacun de Answer (D) is incorrect (like A and B). Each of these individual
ces contrôles, ainsi que d'autres probablement, aide la direction à parvenir à controls, and probably others as well, help management achieve its
son objectif, à savoir éviter le rejet d'effluents liquides non conformes aux objective of preventing the release of waste water that does not
plafonds autorisés ou à d'autres normes. Ces trois contrôles ont chacun une meet permit limits or other conditions. These three controls each
approche différente du risque. Les résultats des analyses constituent les approach the risk in different ways. Analytical results are the criteria
critères de décision sur les rejets ; le fait de ne pas évacuer les substances for the decision to discharge; keeping pollutants out of the waste
polluantes dans les eaux usées permet de réduire les concentrations et le water will help reduce concentrations and the degree of
degré de prétraitement nécessaire, et les pannes de matériel sont moins pretreatment needed; and equipment breakdown is less likely to
probables si un programme de maintenance préventive est en place. occur if a preventive maintenance program is in place.

Restitution

© IFACI – CIA1 V.5.1 page 67/73


Acquérir une certification
> Préparation au CIA, partie 1
P1 - 112 -- Un responsable de l'audit interne prévoit de procéder à des P1 - 112 -- A chief audit executive plans to make changes that may
changements qui peuvent être mal perçus par le personnel d'audit. Quel est be perceived negatively by the audit staff. The best way to reduce
le meilleur moyen d'atténuer cette résistance : resistance would be to:
A. Élaborer complètement sa nouvelle approche avant de la présenter au A. Develop the new approach fully before presenting it to the audit
personnel d'audit. staff.
B. Demander au directeur général (DG) d'approuver ces changements et B. Ask the chief executive officer (CEO) to approve the changes
d'assister à la réunion du personnel du service lors de laquelle ils seront and have the CEO attend the departmental staff meeting when
présentés. they are presented.
C. Présenter l'idée générale au personnel et l'associer à l'élaboration des C. Approach the staff with the general idea and involve them in the
changements. development of the changes.
D. Demander aux clients de l'activité d'audit interne de soutenir ces D. Get the internal audit activity's clients to support the changes.
changements.
La réponse (A) est fausse. Elaborer le plan puis le présenter au personnel Answer (A) is incorrect. Developing the plan then presenting it to
d'audit ne permettra pas d'atténuer les résistances au changement. the audit staff would not help reduce their resistance to change.
La réponse (B) est fausse. Faire participer le DG ne permettra pas forcément Answer (B) is incorrect. Involving the CEO will not necessarily
d'atténuer la résistance du personnel au changement. reduce the audit staff's resistance to change.
La réponse (C) est juste. Associer le personnel à l'élaboration des Answer (C) is correct. Involving the staff in the change from
changements dès les premières phases atténuera ses résistances. the beginning will reduce their resistance to change.
La réponse (D) est fausse. Faire participer les clients de l'activité d'audit Answer (D) is incorrect. Involving the internal audit activity's clients
interne ne permettra pas forcément d'atténuer la résistance du personnel au will not necessarily reduce the audit staff's resistance to change.
changement.

P1 - 113 -- Le changement organisationnel doit être considéré en tenant P1 - 113 -- Organizational change must be considered in the light
compte de la résistance que peut lui opposer le personnel. Cette résistance : of potential employee resistance. Resistance
A. Peut survenir alors même que le personnel bénéficiera de ce changement. A. May occur even though employees will benefit from the change.
B. Sera la plus vive lorsque les groupes informels sont les plus faibles. B. Will be greatest when informal groups are weakest.
C. Sera insignifiante si le personnel ne redoute pas de pertes financières. C. Will be insignificant if no economic loss by employees is
D. Est essentiellement axée sur les menaces perçues sur les besoins expected.
psychologiques. D. Is centered mostly on perceived threats to psychological needs.
La réponse (A) est juste. La résistance au changement peut être Answer (A) is correct. Resistance to change may be caused
provoquée par la crainte des ajustements personnels qui peuvent se by fear of the personal adjustments that may be required.
révéler nécessaires. Le personnel peut être véritablement inquiet de la Employees may have a genuine concern about the
pertinence du changement, percevoir un manque d'égard envers ses usefulness of the change, perceive a lack of concern for
sentiments, craindre le résultat, s'inquiéter d'une dévalorisation de son workers' feelings, fear the outcome, worry about downgrading
statut professionnel et redouter l'abandon des anciennes procédures of job status, and resent deviations from past procedures for
(en particulier si les nouvelles sont moins participatives que les implementing change (especially if new procedures are less
anciennes). Des ajustements sociaux peuvent également se révéler participative than the old). Social adjustments also may be
nécessaires, qui enfreignent les normes comportementales des required that violate the behavioral norms of informal groups
groupes informels ou dérangent le statut quo social au sein des or disrupt the social status quo within groups. Economic
groupes. Les ajustements économiques peuvent entraîner des pertes adjustments may involve potential economic loss or
économiques ou une insécurité due aux menaces perçues envers les insecurity based on perceived threats to jobs. In general, any
emplois. En général, toute dégradation perçue de la situation perceived deterioration in the work situation that is seen as a
professionnelle qui est considérée comme une menace sur les besoins threat to economic, social, and/or psychological needs will
économiques, sociaux et/ou psychologiques produira de la résistance. produce resistance. The various adjustments required are
Les divers ajustements nécessaires risquent davantage de se heurter à most likely to be resisted when imposed unilaterally by higher
une résistance lorsqu'ils sont imposés de manière unilatérale par une authority. However, employees who share in finding solutions
autorité supérieure. Cependant, les salariés qui participent à la to the problems requiring change are less likely to resist
recherche de solutions aux problèmes nécessitant un changement because they will have some responsibility for the change.
sont moins susceptibles de s'opposer car ils ont une certaine Answer (B) is incorrect because strong informal groups are likely to
responsabilité dans le changement. offer more resistance.
La réponse (B) est fausse, car si les groupes informels sont puissants, ils Answer (C) is incorrect because resistance arises from threats to a
risquent d'opposer davantage de résistance. complex pattern of economic, social, and psychological needs.
La réponse (C) est fausse. La résistance provient des menaces qui pèsent Answer (D) is incorrect because resistance arises from threats to a
sur un schéma complexe de besoins économiques, sociaux et complex pattern of economic, social, and psychological needs.
psychologiques.
La réponse (D) est fausse. La résistance provient des menaces qui pèsent
sur un schéma complexe de besoins économiques, sociaux et
psychologiques.

Restitution

© IFACI – CIA1 V.5.1 page 68/73


Acquérir une certification
> Préparation au CIA, partie 1
P1 - 114 -- Le contrôle qui permet le plus probablement de s'assurer que les P1 - 114 -- The control that would most likely ensure that payroll
chèques de paye ont été établis pour les montants autorisés seulement checks are written only for authorized amounts is to:
consiste à :
A. Conduct periodic floor verification of employees on the payroll.
A. Procéder à des vérifications périodiques, dans les ateliers, du personnel B. Require the return of undelivered checks to the cashier.
inscrit dans les effectifs. C. Require supervisory approval of employee time cards.
B. Demander que les chèques non remis à leur destinataire soient renvoyés D. Periodically witness the distribution of payroll checks.
au trésorier.
C. Demander à la hiérarchie de valider les fiches de présence des salariés.
D. Assister périodiquement à la distribution des chèques de paye.
La réponse (A) est fausse. Les salariés peuvent être correctement inscrits Answer (A) is incorrect. Employees may be properly included on
dans les effectifs sans que les montants versés soient pour autant autorisés. payroll, but the amounts paid may be unauthorized.
La réponse (B) est fausse. Les chèques non remis à leur destinataire Answer (B) is incorrect. Undelivered checks provide no evidence
n'apportent aucune preuve de la validité des montants. regarding the validity of the amounts.
La réponse (C) est juste. Le supérieur hiérarchique du salarié serait le Answer (C) is correct. The employee's supervisor would be in
mieux placé pour veiller à ce que le montant correct soit payé. the best position to ensure payment of the proper amount.
La réponse (D) est fausse. Assister à la distribution des chèques ne permet Answer (D) is incorrect. Witnessing a payroll distribution would not
pas de vérifier que ce sont les montants corrects qui sont payés. assure that amounts paid are authorized.

P1 - 115 -- Laquelle des observations suivantes indiquera le plus P1 - 115 -- Which of the following observations by an auditor is
probablement à l'auditeur l'existence de carences dans les contrôles sur la most likely to indicate the existence of control weaknesses over
préservation des actifs ? safeguarding of assets?
I. En raison de son emplacement, un service ne peut pas servir I. A service department's location is not well suited to allow
correctement les autres unités. adequate service to other units.
II. Les antécédents des salariés occupant des postes sensibles ne sont pas II. Employees hired for sensitive positions are not subjected to
contrôlés. background checks.
III. Les cadres n'ont pas accès aux rapports présentant les performances III. Managers do not have access to reports that profile overall
globales de l'entreprise par rapport à celles des autres organisations de performance in relation to other benchmarked organizations.
référence. IV. Management has not taken corrective action to resolve past
IV. La direction n'a pas pris de mesures correctives pour remédier aux engagement observations related to inventory controls.
lacunes observées lors de missions précédentes dans les contrôles des
stocks. A. I and II only.
B. I and IV only.
A. I et II uniquement. C. II and III only.
B. I et IV uniquement. D. II and IV only.
C. II et III uniquement.
D. II et IV uniquement.
La réponse (A) est fausse. La proposition I décrit un symptôme de carences Answer (A) is incorrect. The item I is a symptom of weak controls
dans les contrôles mis en place pour la réalisation des buts et objectifs de for achieving organizational goals and objectives, but not for
l'organisation, mais non pour la préservation des actifs. safeguarding of assets.
La réponse (B) est fausse. La proposition I décrit un symptôme de carences Answer (B) is incorrect. The item I is a symptom of weak controls
dans les contrôles mis en place pour la réalisation des buts et objectifs de for achieving organizational goals and objectives, but not for
l'organisation, mais non pour la préservation des actifs. safeguarding of assets.
La réponse (C) est fausse. La proposition III décrit un symptôme de carences Answer (C) is incorrect. The item III is a symptom of weak controls
dans les contrôles mis en place pour la réalisation des buts et objectifs de for achieving organizational goals and objectives, but not for
l'organisation, mais non pour la préservation des actifs. safeguarding of assets.
La réponse (D) est juste. La proposition II décrit un symptôme de Answer (D) is correct. The item II is a symptom of weak
carences dans les contrôles mis en place pour la préservation des controls for achieving organizational goals and objectives,
actifs.Le fait que la direction n'ait pas pris de mesures correctives à la but not for safeguarding of assets. Management's failure to
suite des observations des missions précédentes, qui portaient sur la take corrective action on past engagement observations,
préservation des actifs, constitue une carence en la matière. which related to safeguarding of assets, is a weakness related
to safeguarding of assets.

Restitution

© IFACI – CIA1 V.5.1 page 69/73


Acquérir une certification
> Préparation au CIA, partie 1
P1 - 116 -- Laquelle des mesures suivantes permettrait de réduire au P1 – 116 -- Which of the following would minimize defects in
maximum les défauts sur les produits finis provoqués par la mauvaise qualité finished goods caused by poor quality raw materials?
des matières premières ?
A. Documented procedures for the proper handling of work-in-
A. Établir des procédures documentées décrivant pour le traitement correct process inventory.
de la production en cours. B. Required material specifications for all purchases.
B. Imposer des spécifications matérielles pour tous les achats. C. Timely follow-up on all unfavorable usage variances.
C. Effectuer un suivi rapide de tous les écarts défavorables sur quantité. D. Determination of the amount of spoilage at the end of the
D. Déterminer le volume de rejets à la fin du processus de fabrication. manufacturing process.
La réponse (A) est fausse. Cette mesure ne permettrait pas de faire en sorte Answer (A) is incorrect. This would not ensure that raw materials
que les matières premières soient de qualité satisfaisante. are of sufficient quality.
La réponse (B) est juste. Les spécifications pour les matériaux achetés Answer (B) is correct. Specifications for materials purchased
procurent un moyen objectif de déterminer que les matériaux provide an objective means of determining that the materials
répondent aux niveaux minimum de qualité requis pour la production. meet the minimum quality level required for production.
La réponse (C) est fausse. Cette procédure permettrait uniquement de veiller Answer (C) is incorrect. This would only help ensure that raw
à ce que les matières premières soient utilisées dans les quantités materials are used in the proper quantities.
appropriées. Answer (D) is incorrect. This would only permit proper
La réponse (D) est fausse. Cette méthode permettrait uniquement de determination of spoilage after raw materials have been used in
déterminer les rejets après utilisation des matières premières pour la production.
production.

P1 - 117 -- La filiale d'une multinationale comporte une unité de virements de P1 - 117 -- Appropriate internal control for a multinational
fonds. Lequel des contrôles internes suivants est impératif ? corporation's branch office that has a monetary transfer unit
requires that:

A. La personne qui procède aux virements ne fait pas les rapprochements A. The individual who initiates wire transfers not reconcile the bank
bancaires. statement.
B. C'est le responsable de la filiale qui reçoit l'ensemble des virements. B. The branch manager receive all wire transfers.
C. Les taux de change doivent être calculés séparément par deux employés C. Foreign currency rates be computed separately by two different
différents. employees.
D. Ce sont les cadres de l'entreprise qui avalisent l'embauche des employés D. Corporate management approve the hiring of monetary transfer
de l'unité des virements. unit employees.

La réponse (A) est juste. Il est nécessaire que le rapprochement Answer (A) is correct. Independent reconciliation of bank
bancaire soit indépendant pour que le contrôle interne soit satisfaisant. accounts is necessary for good internal control.
La réponse (B) est fausse. Cette considération n'est pas importante pour le Answer (B) is incorrect. This is not an important internal control
contrôle interne. consideration.
La réponse (C) est fausse. La conversion au moyen des taux de change Answer (C) is incorrect. Foreign currency translation rates are not
n'est pas calculée, mais vérifiée. Le fait que deux employés du même computed, but instead verified. Having two employees in the same
département effectuent la même tâche n'améliorera pas significativement le department perform the same task will not significantly enhance
contrôle interne. internal control.
La réponse (D) est fausse. Cette considération n'est pas importante pour le Answer (D) is incorrect. This is not an important internal control
contrôle interne. consideration.

P1 - 118 -- Laquelle des propositions suivantes ne fait pas partie de la P1 - 118 -- Which of the following is not implied by the definition of
définition du contrôle ? control?
A. Mesurer les avancées en direction des objectifs. A. Measurement of progress toward goals.
B. Révéler les écarts par rapports aux plans. B. Uncovering of deviations from plans.
C. Attribuer les responsabilités concernant les écarts. C. Assignment of responsibility for deviations.
D. Indiquer la nécessité d'une mesure correctrice. D. Indication of the need for corrective action.

La réponse (A) est fausse. La mesure des avancées en direction des buts est Answer (A) is incorrect because measurement of progress toward
sous-entendue dans la définition du contrôle. goals is implied by the definition of control.
La réponse (B) est fausse. L'identification des écarts par rapport aux plans Answer (B) is incorrect because uncovering of deviations from
est sous-entendue dans la définition du contrôle. plans is implied by the definition of control.
La réponse (C) est juste. Pour l'encadrement, le contrôle consiste Answer (C) is correct. The basic process of control for
fondamentalement à définir des normes, mesurer les performances à managers consists of establishing standards, measuring
l'aune de ces références et corriger les écarts afin de garantir la performance against standards, and correcting for deviations
réalisation des objectifs de l'entreprise. Ainsi, l'attribution des to ensure accomplishment of enterprise goals. Thus,
responsabilités concernant les écarts observés ne fait pas partie du assigning responsibility for deviations found is not a part of
contrôle. the controlling function.
La réponse (D) est fausse. Le fait d'indiquer la nécessité d'une mesure Answer (D) is incorrect because indication of the need for
correctrice est sous-entendu dans la définition du contrôle. corrective action is implied by the definition of control.

Restitution

© IFACI – CIA1 V.5.1 page 70/73


Acquérir une certification
> Préparation au CIA, partie 1
P1 - 119 -- Quelle méthode de départementalisation se caractérise par un P1 - 119 -- Dual reporting is most characteristic of which method of
double reporting ? departmentation?
A. La départementalisation géographique. A. Territorial.
B. La départementalisation par fonction. B. Functional.
C. La départementalisation par produit. C. Product.
D. La départementalisation matricielle. D. Matrix.
La réponse (A) est fausse. La départementalisation géographique n'impose Answer (A) is incorrect because departmentation by territory does
pas d'enfreindre le principe de l'unité de commandement. not require violation of the unity of command principle.
La réponse (B) est fausse. La départementalisation par fonction n'impose pas Answer (B) is incorrect because departmentation by function does
d'enfreindre le principe de l'unité de commandement. not require violation of the unity of command principle.
La réponse (C) est fausse. La départementalisation par produit n'impose pas Answer (C) is incorrect because departmentation by product does
d'enfreindre le principe de l'unité de commandement. not require violation of the unity of command principle.
La réponse (D) est juste. Une organisation matricielle se compose Answer (D) is correct. A matrix organization consists of a
d'une équipe de projet constituée de personnes issues de différentes project team formed with people from various functional
zones fonctionnelles au sein de l'organisation. Ces spécialistes rendent areas within the organization. These specialists report
simultanément compte au gestionnaire du projet et au responsable de simultaneously to the project manager and the managers of
leur département fonctionnel. À la fin du projet, l'équipe est dissoute. their functional departments. At the end of the project, the
team is disbanded.

P1 - 120 -- Lorsqu'on détermine l'éventail de contrôle approprié, il faut avant P1 - 120 -- When determining the appropriate span of control, the
tout prendre en compte : most important consideration is
A. L'ensemble des politiques et procédures en vigueur. A. The set of policies and procedures currently in effect.
B. L'éventail de contrôle type dans les autres organisations. B. The typical span of control used by other organizations.
C. Les préférences des créanciers de l'organisation. C. The preference of the organization's creditors.
D. Le fait que tous les services seront évalués, qu'ils soient ou non D. That all departments will be evaluated, whether or not they will
concernés. be affected.
La réponse (A) est juste. Les facteurs les plus importants à prendre à Answer (A) is correct. The most important factors to consider
compte sont les préférences et qualifications du personnel et de are the employees' and manager's preferences and skills, the
l'encadrement, la culture de l'organisation, les tâches concernées, organization's culture, the tasks involved, physical location of
l'emplacement du service, ainsi que les politiques et procédures the department, and established policies and procedures.
établies. Answer (B) is incorrect because, although examining similar
La réponse (B) est fausse, car même s'il peut être utile d'examiner ce que entities may be useful, it is not one of the most important
font des entités analogues, ce n'est pas le facteur le plus important à prendre considerations.
en compte. Answer (C) is incorrect because this consideration would only be
La réponse (C) est fausse. La prise en compte de ce facteur n'est nécessaire necessary if required by an agreement with the creditors.
que si un accord passé avec les créanciers l'impose. Answer (D) is incorrect because whether all departments will be
La réponse (D) est fausse. Une décision portant sur l'éventail de contrôle ne evaluated is not normally considered in a span-of-control decision.
tient habituellement pas compte de la question de savoir si tous les services
seront évalués.

P1 - 121 -- Pendant une mission d’audit, l’auditeur a éprouvé une certaine P1 - 121 -- During an internal audit, the auditor experienced
difficulté pour obtenir les informations dont il avait besoin auprès d’un salarié. difficulty obtaining required information from a specific employee.
Comme cette situation a perduré pendant une semaine, l’auditeur a When this situation continued for one week, the auditor requested
demandé un rendez-vous avec le salarié afin d’identifier le problème et de a private meeting with the employee for the purpose of identifying
résoudre la difficulté grâce à une discussion ouverte. Quelle technique de the problem and resolving the difficulty through open discussion.
gestion de conflit utilisait l’auditeur ? Which conflict management technique was the auditor applying?
A. La résolution des problèmes. A. Problem solving.
B. L’élargissement des ressources B. Expansion of resources.
C. La commande autoritaire C. Authoritative command.
D. La modification de la variable humaine. D. Altering the human variable.
La réponse (A) est juste. La technique de gestion de conflit qui implique Answer (A) is correct. The conflict management technique
l'organisation de réunions face à face est la résolution des problèmes. that involves face-to-face meetings is problem solving.
La résolution des problèmes est une façon d’affronter le conflit et d’en Problem solving is a means of confronting the conflict and
faire disparaître les causes. L’accent est mis sur les faits et les removing its causes. The emphasis is on facts and solutions,
solutions, non pas sur les personnalités ou l'affectation de la not personalities and assignment of blame.
responsabilité. Answer (B) is incorrect because expansion of resources addresses
La réponse (B) est fausse. L’élargissement des ressources s’adresse aux conflicts that arise from scarcity.
conflits qui sont dus à la pénurie de ressources. Answer (C) is incorrect because the auditor is not using formal
La réponse (C) est fausse. L’auditeur n’utilise pas une autorité formelle. authority.
La réponse (D) est fausse. L’auditeur n’utilise pas des techniques Answer (D) is incorrect because the auditor is not using behavioral
comportementales afin de modifier des attitudes ou un comportement. techniques to change attitudes and behavior.

Restitution

© IFACI – CIA1 V.5.1 page 71/73


Acquérir une certification
> Préparation au CIA, partie 1
P1 - 122 -- Laquelle des activités suivantes représente le risque le plus P1 - 122 -- Which of the following activities represents the greatest
important pour une organisation industrielle qui vient de subir une fusion et risk to a post-merger manufacturing organization and is therefore
qui est par conséquent fortement susceptible d'être auditée ? most likely be the subject of an internal auditing engagement?
A. Regrouper les caisses à montant fixe. A. Combining imprest funds.
B. Regrouper les fonctions achats. B. Combining purchasing functions.
C. Regrouper les fonctions juridiques. C. Combining legal functions.
D. Regrouper les fonctions marketing. D. Combining marketing functions.
La réponse (A) est fausse. Les caisses à montant fixe représentent Answer (A) is incorrect because imprest funds are typically
généralement un montant négligeable. immaterial in amount.
La réponse (B) est juste. Selon le CRIPP, “l'auditeur interne doit Answer (B) is correct. According to IPPF, "When planning the
procéder à une évaluation préliminaire des risques liés à l'activité engagement, the internal auditor should identify and assess
soumise à l'audit. Les objectifs de la mission doivent être déterminés risks relevant to the activity under review. The engagement
en fonction des résultats de cette évaluation”. “Les objectifs d'une objectives should reflect the results of the risk assessment."
mission de conseil doivent porter sur les processus de management In a consulting engagement, the objectives should address
des risques, de contrôle et de gouvernement d'entreprise dans la limite risks, controls, and governance processes to the extent
convenue avec le client” (CRIPP). Les fonctions achats représentent agreed upon with the client (IPPF). Purchasing functions
l'exposition la plus forte au risque de perte des éléments listés et sont represent the greatest exposure to loss of the items listed and
par conséquent fortement susceptibles d'être évaluées. L'exposition are therefore most likely to be evaluated. The financial
financière est ici généralement supérieure à celle qui prévaut par exposure in the purchasing function is ordinarily greater than
exemple dans les fonctions juridiques ou de marketing. Après une in, for example, the legal and marketing functions. After a
fusion, le risque est accru par la difficulté qu'il y a à combiner les merger, risk is heightened because of the difficulty of
systèmes différents des deux organisations. Ainsi, la probabilité qu'un combining the disparate systems of the two organizations.
audit soit mené est augmentée. Thus, the likelihood of an engagement is increased.
La réponse (C) est fausse. Les fonctions juridiques ne représentent Answer (C) is incorrect because legal functions do not typically
généralement pas un risque de perte aussi important que les fonctions represent a risk of loss as great as the purchasing functions.
achats. Answer (D) is incorrect because marketing functions do not
La réponse (D) est fausse. Les fonctions marketing n'entraînent typically represent a risk of loss as great as the purchasing
généralement pas un risque de perte aussi important que les fonctions functions.
achats.
P1 - 123 -- Aux États-Unis, la loi Sarbanes-Oxley s'applique dans tous les cas P1 - 123 -- The Sarbanes-Oxley Act applies to all of the following
suivants, sauf except
A. Une entreprise locale (américaine) dont le capital est fortement dilué et A. A domestic corporation with widely held stock that is traded on a
dont les titres sont cotés sur une grande place boursière. major stock exchange.
B. Une entreprise étrangère dont le capital est fortement dilué aux États-Unis B. A foreign corporation with widely held stock in the United States
et dont les titres sont cotés sur une grande place boursière. that is traded on a major stock exchange.
C. Une société de personnes cotée en Bourse. C. A publicly traded partnership.
D. Une société en nom collectif qui opère dans plusieurs États des États- D. A general partnership that practices in more than one state.
Unis.
La réponse (A) est fausse. Une entreprise locale dont le capital est fortement Answer (A) is incorrect because a domestic corporation with widely
dilué doit respecter les lois fédérales sur les titres. held stock is required to follow federal securities laws.
La réponse (B) est fausse. Les actions de l'entreprise étrangère s'échangent Answer (B) is incorrect because the stock of the foreign company is
sur une grande place boursière, et doivent donc respecter les lois fédérales traded on a major stock exchange, and therefore required to follow
sur les titres. federal securities laws.
La réponse (C) est fausse. Une société de personnes cotée en Bourse Answer (C) is incorrect because a publicly traded partnership is
s'apparente à une société classique. Elle émet des parts de capital et ses similar to a corporation because it often issues shares of stock and
titres s'échangent sur une grande place boursière. Une société de personnes is traded on a major stock exchange. Therefore, a publicly traded
cotée en Bourse doit par conséquent respecter les lois fédérales sur les titres. partnership is required to follow federal securities laws.
La réponse (D) est juste. La loi Sarbanes-Oxley Act s'applique aux Answer (D) is correct. The Sarbanes-Oxley Act applies to
émetteurs de titres cotés en Bourse soumis aux lois fédérales sur les issuers of publicly traded securities subject to federal
titres, y compris les entreprises étrangères. Une société en nom securities laws, including foreign companies. A general
collectif n'émet pas de parts de capital et n'est donc pas tenue par les partnership dœs not issue stock, and therefore, federal
lois fédérales sur les titres. securities laws do not apply.

Restitution

© IFACI – CIA1 V.5.1 page 72/73


Acquérir une certification
> Préparation au CIA, partie 1
[Enoncé #22] Le responsable d’une ligne de production a autorité pour [Fact Pattern #22] The manager of a production line has the
commander et recevoir des pièces de rechange pour tous les équipements authority to order and receive replacement parts for all machinery
qui requièrent une maintenance périodique. L’auditeur interne reçoit le that require periodic maintenance. The internal auditor received an
renseignement anonyme suivant : le responsable a commandé une quantité anonymous tip that the manager ordered substantially more parts
de pièces nettement plus élevée que nécessaire à un membre de sa famille than were necessary from a family member in the parts supply
travaillant dans la fourniture de pièces détachées. Les pièces non business. The unneeded parts were never delivered. Instead, the
nécessaires n’ont jamais été livrées. En revanche, le responsable a traité les manager processed receiving documents and charged the parts to
documents de réception et porté les pièces aux comptes de maintenance de machinery maintenance accounts. The payments for the
l’équipement. Le paiement des pièces non livrées a été envoyé au undelivered parts were sent to the supplier, and the money was
fournisseur, et l’argent a été réparti entre le responsable et son parent. divided between the manager and the family member.
P1-124 -- (Voir énoncé 22) Lequel des contrôles internes suivants aurait le P1-124 -- (Refers to Fact Pattern #22) Which of the following
plus vraisemblablement empêché cette fraude ? internal controls would have most likely prevented this fraud from
occurring?
A. La définition, durant le processus d'adjudication, des niveaux de dépenses A. Establishing predefined spending levels for all vendors during
préétablis pour tous les fournisseurs. the bidding process.
B. La séparation des fonctions de réception et d'autorisation d'achat de B. Segregating the receiving function from the authorization of parts
pièces. purchases.
C. La comparaison du connaissement des pièces de rechange et du bon de C. Comparing the bill of lading for replacement parts to the
commande avalisé. approved purchase order.
D. L'utilisation du système d'inventaire de l'entreprise afin de vérifier D. Using the company's inventory system to match quantities
l'adéquation des quantités commandées aux quantités reçues. requested with quantities received.

La réponse (A) est fausse. Des niveaux de dépense prédéfinis inclueraient Answer (A) is incorrect. Predefined spending levels would probably
probablement déjà les montants frauduleux et ne feraient que limiter already include the fraudulent amounts and would only limit the
l'ampleur de la fraude. size of the fraud.
La réponse (B) est juste. La solution qui permettrait le plus Answer (B) is correct. Additional authorization would be the
probablement d'empêcher cette fraude consiste à exiger une most likely choice in preventing the fraud.
autorisation complémentaire. Answer (C) is incorrect. The bill of lading would agree with the
La réponse (C) est fausse. Le connaissement serait conforme à la purchase order. The quantity received (verified by a third party)
commande. Il faudrait comparer la quantité reçue (vérifiée par un tiers) à la should be compared to both the bill of lading and the purchase
fois au connaissement et au bon de commande. order.
La réponse (D) est fausse. Le rapprochement informatique permettrait Answer (D) is incorrect. The computer matching would only verify
seulement de vérifier s'il y a eu des écritures frauduleuses. the fraudulent paperwork.

P1-125 -- Laquelle des entrées frauduleuses suivantes est la plus susceptible P1-125 -- Which of the following fraudulent entries is most likely to
d'être utilisée pour masquer le vol d'un actif ? be made to conceal the theft of an asset?
A. Débiter des charges et créditer l'actif. A. Debit expenses, and credit the asset.
B. Débiter l'actif et créditer un autre compte d'actif. B. Debit the asset, and credit another asset account.
C. Débiter les produits et créditer l'actif. C. Debit revenue, and credit the asset.
D. Débiter un autre compte d'actif et créditer l'actif. D. Debit another asset account, and credit the asset.
La réponse (A) est juste. La plupart des fraudeurs tenteront de Answer (A) is correct. Most fraud perpetrators would attempt
dissimuler leur vol en l'imputant sur un compte de charge. to conceal their theft by charging it against an expense
La réponse (B) est fausse. Débiter le compte correspondant à l'actif volé irait account.
dans la mauvaise direction si le but est de masquer un vol d'actif. Answer (B) is incorrect. Debiting the stolen asset account would be
La réponse (C) est fausse. Une entrée qui diminuerait les produits serait going in the wrong direction to conceal an asset theft.
inhabituelle et attirerait l'attention. Answer (C) is incorrect. An entry decreasing revenue would be
La réponse (D) est fausse. Cette entrée ne permettrait pas de masquer la unusual and would stand out.
fraude de manière permanente. Elle reporterait simplement un solde Answer (D) is incorrect. This entry would not permanently conceal
déséquilibré sur un autre compte. the fraud. It would simply shift the unreconcilible balance to another
asset account.

Restitution

© IFACI – CIA1 V.5.1 page 73/73

Vous aimerez peut-être aussi